SlideShare a Scribd company logo
1 of 57
ĐẠI HỌC THÁI NGUYÊN
TRƢỜNG ĐẠI HỌC KHOA HỌC
---------------------------
Tải tài liệu tại sividoc.com
Viết đề tài giá sinh viên – ZALO:0973.287.149-TEAMLUANVAN.COM
VŨ PHÚ BÌNH
PHƢƠNG TRÌNH DIOPHANTINE DẠNG
x2
− Dy2
= ±4
LUẬN VĂN THẠC SĨ TOÁN HỌC
THÁI NGUYÊN - 2018
ĐẠI HỌC THÁI NGUYÊN
TRƢỜNG ĐẠI HỌC KHOA HỌC
---------------------------
Tải tài liệu tại sividoc.com
Viết đề tài giá sinh viên – ZALO:0973.287.149-TEAMLUANVAN.COM
VŨ PHÚ BÌNH
PHƢƠNG TRÌNH DIOPHANTINE DẠNG
x2
− Dy2
= ±4
Chuyên ngành: Phƣơng pháp Toán sơ cấp
Mã số: 8460113
LUẬN VĂN THẠC SĨ TOÁN HỌC
NGƯỜI HƯỚNG DẪN KHOA HỌC
PGS.TS. Nông Quốc Chinh
THÁI NGUYÊN - 2018
i
Viết đề tài giá sinh viên – ZALO:0973.287.149-TEAMLUANVAN.COM
Mnc lnc
L i nói đau 1
Chương 1 Phương trình Diophantine x2
− Dy2
= ±1 2
1.1 Liên phân so và giản phân . . . . . . . . . . . . . . . . . . . . . 2
1.1.1 Liên phân so hǎu hạn và giản phân . . . . . . . . . . . . 2
1.1.2 Liên phân so vô hạn . . . . . . . . . . . . . . . . . . . . 6
1.2 Phương trình Diophantine x2
− Dy2
= ±1..........................................13
1.2.1 Phương trình Pell dạng x2
− dy2
= 1.......................................14
1.2.2 Úng dụng liên phân so
√
D vào phương trình Pell x2
−
Dy2
= 1 ............................................................................ 21
1.2.3 Phương trình Pell dạng x2
− dy2
= −1 ...................................27
Chương 2 Phương trình Diophantine dạng x2
− Dy2
= ±4 37
2.1 Cau trúc nghi m của ho phương trình x2
− Dy2
= ±4.......................37
2.2 Phương trình Diophantine dạng x2
− Dy2
= 4....................................42
2.3 Phương trình Diophantine dạng x2
− Dy2
= −4.................................45
2.4 M®t so áng dụng trong toán phő thông.................................................48
2.4.1 Tìm so nguyên tà h thác ràng bu®c.......................................48
2.4.2 Xap xỉ hǎu t của căn b c 2................................................. 48
2.4.3 Tőng của nhǎng so nguyên liên tiep nhau.................................49
2.4.4 Tam giác Pythagoras ................................................................49
2.4.5 Tam giác Heron ........................................................................50
Ket lu n 52
Tài li u tham khảo 53
ii
Viết đề tài giá sinh viên – ZALO:0973.287.149-TEAMLUANVAN.COM
L i nói đau
Xét phương trình có dạng
f(x1, x2, ..., xn) = 0 (1)
với n ≥ 2 và f (x1, x2, ..., xn) là m®t đa thác nguyên m®t ho c nhieu bien được
goi là phương trình nghi m nguyên hay phương trình Diophantine, nó được goi
theo tên nhà toán hoc Hy Lạp ở the kỉ thá 3 sau công nguyên. Phương trình
Diophantine là m®t trong nhǎng dạng toán lâu đời nhat của Toán hoc và nh n
được nhieu sự quan tâm nghiên cáu của nhieu nhà toán hoc. Tà Euclid, Dio-
phantus, qua Fibonacci, Pell roi đen Fermat, Euler, Lebesgue... và thời hi n đại
là Gelfold, Matiasevic, Shenzel, Serpinsky... Phương trình Diophantine đã trải
qua m®t lịch sả phát trien lâu dài.
Thông qua vi c giải các phương trình Diophantine, các nhà toán hoc đã
tìm ra được nhǎng tính chat thú vị của so nguyên, so hǎu t , so đại so. Giải
phương trình Diophantine đã đưa đen sự ra đời của Liên phân so, Lý thuyet
đường cong elliptic, Lý thuyet xap xỉ Diophantine, Th ng dư bình phương, So
hoc modular...
Các bài toán ve phương trình Diophantine không có quy tac giải tőng
quát, ho c neu có cũng chỉ là đoi với các dạng đơn giản. Moi phương trình
với dạng riêng của nó đòi hỏi m®t cách giải đ c trưng phù hợp. Chính vì v y,
phương trình Diophantine van thường xuyên xuat hi n dưới các hình thác khác
nhau và luôn được đánh giá là khó do tính không mau mực của nó. M®t dạng
đ c bi t của phương trình Diophante là x2
− Dy2
= N rat được quan tâm và
có rat nhieu ket quả xung quanh dạng phương trình này. Gan đây m®t ket quả
thú vị của A. Tekcan ve phương trình x2
− Dy2
= ±1 và x2
− Dy2
= ±4 đã
được công bo. Mục đích của lu n văn là trình bày lại các ket quả ve cau trúc
1
Viết đề tài giá sinh viên – ZALO:0973.287.149-TEAMLUANVAN.COM
nghi m của các phương trình x2
− Dy2
= ±1 và x2
− Dy2
= ±4.
Lu n văn gom 2 chương:
Chương 1: Chúng tôi giới thi u các ket quả ve liên phân so, giản phân và cau
trúc nghi m của phương trình Diophantine x2
− Dy2
= ±1.
Chương 2: Chúng tôi trình bày lại cau trúc nghi m của phương trình Diophan-
tine x2
− Dy2
= ±4 và m®t so áng dụng trong toán phő thông.
Lu n văn này được thực hi n và hoàn thành vào tháng 5 năm 2018 tại
trường Đại hoc Khoa hoc - Đại hoc Thái Nguyên. Qua đây, tác giả xin bày tỏ
lòng biet ơn sâu sac tới PGS.TS. Nông Quoc Chinh, người đã t n tình hướng
dan tác giả trong suot quá trình làm vi c đe hoàn thành lu n văn này. Tác giả
xin gải lời cảm ơn chân thành đen Khoa Toán - Tin, Trường Đại hoc Khoa hoc
- Đại hoc Thái Nguyên, đã tạo moi đieu ki n đe giúp tác giả hoc t p và hoàn
thành lu n văn cũng như chương trình thạc sĩ. Tác giả cũng xin gải lời cảm
ơn tới t p the lớp cao hoc, khóa 05/2016 - 05/2018 đã đ®ng viên giúp đơ tác
giả trong quá trình hoc t p và hoàn thành lu n văn này. Đong thời tác giả xin
gải lời cảm ơn tới Ban giám hi u, các đong nghi p tại trường THPT Nguyen
Khuyen, huy n Vĩnh Bảo, Hải Phòng và gia đình bạn bè đã tạo đieu ki n tot
nhat cho tác giả trong suot quá trình hoc t p và hoàn thành lu n văn.
Tác gia
Vũ Phú Bình
2
Viết đề tài giá sinh viên – ZALO:0973.287.149-TEAMLUANVAN.COM
i=0 i=0
Chương 1
Phương trình Diophantine
x2 − Dy2 = ±1
Trong chương này, chúng tôi trình bày m®t so ket quả ve liên phân so, m®t
so cách giải phương trình Diophantine dạng x2
− Dy2
= ±1 và áng dụng của
nó. Các ket quả trong chương này được viet theo các tài li u [1] và [2].
1.1 Liên phân so và giản phân
1.1.1 Liên phân so hfiu hạn và giản phân
Định nghĩa 1.1.1. Cho {ai}∞
i=0 và {bi}∞
i=0 là dãy các so thực.
(i) Bieu thác có dạng
b0
a0 +
b1
(1.1)
a1 +
2 + ...
được goi là m®t liên phân so của hai dãy so {ai}∞
i=0 và {bi}∞
i=0.
(ii) Dãy các bieu thác u = a , u = a +
b0
, u b0
= a + , . . . , được
0 0 1 0
a1
2
a +
b1
1
a2
goi là các giản phân của hai dãy so {ai}∞
i=0 và {bi}∞
i=0.
(iii) Phan tả un xác định như trên được goi là giản phân thú n của hai dãy
so {ai}∞
i=0 và {bi}∞
i=0.
Chú j 1.1.2. (i) Neu n là hǎu hạn và b0 = b1 = . . . = bn = 1 ta kí hi u liên
phân so của hai dãy so {ai}n
và {bi}n
là [a0; a1, . . . , an] .
(ii) Neu a0 ∈ Z và a1, ..., an là các so nguyên dương thì ta nói [a0; a1, ..., an] là
a
0
3
Viết đề tài giá sinh viên – ZALO:0973.287.149-TEAMLUANVAN.COM
m®t liên phân so hũu hạn có đ® dài n.
(iii) M®t liên phân so hǎu hạn là m®t so hǎu t .
Với hai dãy so thực {ai}∞
i=0 và {bi}∞
i=0 ta xét hai dãy so {pn}∞
n=−1 và {qn}n
∞
=−1
như sau:
p−1 = 1, p0 = a0, . . . , pn+1 = an+1pn + bnpn−1.
q−1 = 0, q0 = 1, . . . , qn+1 = an+1qn + bnqn−1.
Khi đó moi quan h giǎa giản phân thá n của hai dãy so {ai}∞
i=0 và {bi}∞
i=0 với
thương thá n của hai dãy so {pn}∞
n=−1 và {qn}n
∞
=−1 được the hi n trong bő đe
sau.
Bo đe 1.1.3. Với các kí hi u và giả thiet như trên ta có giản phân un
moi n ≥ 0.
=
pn
qn
với
Chúng minh. Ta cháng minh đȁng thác trên là đúng bang quy nạp theo n. Th t
v y, với n = 0 và n = 1 thì hien nhiên ket quả là đúng. Giả sả quy nạp đúng
cho n, nghĩa là ta có u =
pn
. Thay a trong bieu thác u bởi a bn
+ ta thu
n
qn
n
n n an+1
được un+1. Theo định nghĩa ta có pn, qn không phụ thu®c vào bn và an+1 nên tà
công thác truy hoi
ta có
pn
=
anpn−1 + bn−1pn−2
qn anqn−1 + bn−1qn−2
un+1 =
(an
bn
+ )p
an+1
bn
n−1 + bn−1 pn−2
(an +
n+1)
qn−1 + bn−1qn−2
=
(anan+1 + bn)pn−1 + an+1bn−1pn−2
(anan+1 + bn)qn−1 + an+1bn−1qn−2
=
an+1(anpn−1 + bn−1pn−2) + bnpn−1
an+1(anqn−1 + bn−1qn−2) + bnqn−1
=
an+1pn + bnpn−1
an+1qn + bnqn−1
=
pn+1
.
qn+1
Bő đe được cháng minh.
a
4
Viết đề tài giá sinh viên – ZALO:0973.287.149-TEAMLUANVAN.COM
i=0
i=0 i=0
Bő đe 1.1.3 cho ta m®t công thác tính các giản phân qua thương của các
dãy so. M nh đe tiep theo chỉ ra rang moi so hǎu t đeu bieu dien được dưới
dạng m®t liên phân so hǎu hạn và bieu dien đó là duy nhat. Trước tiên ta nhac
lại thu t toán Euclid tìm ước chung lớn nhat của hai so nguyên.
Chú j 1.1.4. (i) Cho các so nguyên a, b ∈ Z, b > 0. Khi đó như đã biet chúng
ta có the tìm được ước chung lớn nhat của a và b bang cách thác hi n thu t
toán Euclid như sau:
a = a0b + r1, 0 < r1 < b
b = a1r1 + r2, 0 < r2 < r1,
r1 = a2r2 + r3, 0 < r3 < r1,
. . . ,
rn−2 = an−1rn−1 + rn, 0 < rn < rn−1,
rn−1 = anrn,
quá trình này phải dàng và sau hǎu hạn bước ta có gcd(a, b) = rn.
(ii) Tà thu t toán trên ta thu được hai dãy so nguyên hǎu hạn là {ai}n
và
b0 = b1 = . . . = bn = 1.
Khi đó các giản phân của {ai}n
1
và {bi}n
là
u0 = a0 = [a0], u1 = a0 +
1
= [a0; a1], . . . , un = ... = [a0; a1, a2, . . . , an].
(iii) Tà thu t toán trên ta cũng thu được các dãy truy hoi là
p0 = a0, p1 = a1p0 + 1, . . . , pn = anpn−1 + pn−2
và
q0 = 1, q1 = a1, . . . , qn = anqn−1 + qn−2.
Ta có tính chat quan trong của so hǎu t the hi n trong m nh đe sau:
M nh đe 1.1.5. Mői so hũu ty đeu được bieu dien dưới dạng m®t liên phân so
hũu hạn.
a
5
Viết đề tài giá sinh viên – ZALO:0973.287.149-TEAMLUANVAN.COM
Chúng minh. Cho a/b là m®t so hǎu t với b > 0. Theo thu t toán tìm ước
chung lớn nhat và công thác giản phân ta có
a 1
b
= a0 +
b
r1
1
= a0 + 1
. . .
= a0 +
a1 +
a1 +
r1
r2
1
1
.
1
a2 +
1
a3 +
n−2
. . .
+
1
an−1 +
a
V y moi so hǎu t a/b đeu viet được thành m®t liên phân so hǎu hạn là
a/b = [a0; a1, . . . , an].
M nh đe được cháng minh.
Như chúng ta đã biet bieu dien của m®t so hǎu t dưới dạng phân so không
là duy nhat. Tuy nhiên m nh đe tiep theo chỉ ra rang bieu dien của m®t so hǎu
t thành liên phân so là duy nhat.
M nh đe 1.1.6. Bieu dien so hũu ty thành m®t liên phân so hũu hạn dạng
[a0; a1, . . . , an] là duy nhat.
Chúng minh. Cho a/b là m®t so hǎu t và giả sả
a
[a0; a1, . . . , an] =
b
= [b0; b1, . . . , bm].
Ta can cháng minh m = n và ai = bi, với moi i = 0, 1, . . . , n. Th t v y, với
n = 0 ta có a0 = [b0; b1, . . . , bm]. Vì b0 là phan nguyên của a0 và a0 là so nguyên
nên m = 0 và a0 = b0. Giả sả quy nạp đúng cho n − 1, nghĩa là ket lu n trên
là đúng cho moi liên phân so hǎu hạn có đ® dài nhỏ hơn n. Tà bieu thác
a
[a0; a1, . . . , an] =
b
= [b0; b1, . . . , bm]
a
n
6
Viết đề tài giá sinh viên – ZALO:0973.287.149-TEAMLUANVAN.COM
ta suy ra a0 = b0, vì đeu là phan nguyên của cùng m®t so hǎu t . Khi đó ta có
a
[0; a1, . . . , an] =
b
− a0 = [0; b1, . . . , bn].
Do đó [a1; a2, . . . , an] = [b1; b2, . . . , bm]. Theo giả thiet quy nạp ta có n−1 = m−1
và ai = bi, với moi i = 1, . . . , n.
Ví dn 1.1.7. Xét so hǎu t 187/4, ta có
187 = 46.4 + 3,
4 = 1.3 + 1,
3 = 3.1.
Do đó
187
4
= [46; 1, 3].
1.1.2 Liên phân so vô hạn
Trong mục này chúng tôi t p trung trình bày các kien thác ve liên phân so
vô hạn. Trong đó chúng tôi trình bày lại m®t tính chat tot của liên phân so vô
hạn đó là moi so vô t đeu viet được dưới dạng m®t liên phân so vô hạn. Các
ket quả trong mục này được viet theo các tài li u [1].
Định nghĩa 1.1.8. (i) Liên phân so vô hạn là m®t bieu thác có dạng
1
q0 +
q1 +
q2+
1
...
1
+
qs
...
(1.2)
trong đó, q0 là m®t so nguyên, qs với s = 1, 2, ... là các so nguyên dương và được
kí hi u là [q0; q1, . . . , qs, . . .].
(ii) Phan tả qs được goi là so thương hụt hay so hạng thá s của liên phân
so.
7
Viết đề tài giá sinh viên – ZALO:0973.287.149-TEAMLUANVAN.COM
3
4
Chú j 1.1.9. (i) Cho α ∈
/ Q là m®t so thực. So [α] được định nghĩa là so
nguyên sao cho [α] ≤ α < [α] + 1. Đ t
1
α0 = α = a0 +
1
, với a0 = [α] ∈ Z, α1 > 1.
Vì α ∈
/ Q nên α1 không là so nguyên. Khi đó ta có
1
α1 = a1 +
2
, với a1 = [α1] ∈ Z, α2 > 1.
Tiep tục quá trình trên đen bước thá n + 1 ta được
1
αn = an +
αn+1
, với an = [αn] ∈ Z, αn+1 > 1.
Vì α ∈
/ Q nên αn+1 không là so hǎu t nên quá trình này kéo dài vô hạn. Do
đó α = [a0; a1, . . . , ] là m®t bieu dien của so vô t qua liên phân so vô hạn. Đ t
πn = [a0; a1, . . . , an], với moi n = 0, 1, . . . . Khi đó các πn được goi là các liên
phân so hũu hạn của liên phân so vô hạn α.
(ii) Tà cách tìm các so ai và αi ta suy ra bieu dien của m®t so vô t α =
[a0; a1, . . . , ] là duy nhat.
Tà Chú ý 1.1.9 ta suy ra h quả quan trong sau:
H quả 1.1.10. Mői so vô ty đeu bieu dien m®t cách duy nhat dưới dạng m®t
liên phân so vô hạn.
Chúng minh. Theo Chú ý 1.1.9 (i) thì moi so vô t đeu có bieu dien thành liên
phân so vô hạn. Tiep theo giả sả [a0; a1, . . . , ] = α = [b0; b1, . . . , ] là hai bieu
dien của so vô t α. Theo Chú ý 1.1.9 (ii) ta suy ra đieu can cháng minh.
Ví dn 1.1.11. Cho α =
√
3 = 1 +
1
. Suy ra a = √
1 1
= 1 + . Tà đó ta
a1
1
có
2 1
3 − 1 a2
Do đó
a2 = √
3 − 1
= 2 +
a
.
1 1
M®t cách tương tự
a3 = √
3 − 1
= 1 +
a
.
1 3
a2n = √
3 − 1
, a2n+1 = √
2 − 1
với moi n ≥ 1. Vì the
√
3 = [1, 1, 2, 1, 2, 1, 2 . . .].
α
α
8
Viết đề tài giá sinh viên – ZALO:0973.287.149-TEAMLUANVAN.COM
3
— −
≥
√ p
. .
. .
q
∀k : α − <
. qk
. qkqk+1 qkqk
2
k
Ví dn 1.1.12. Cho α =
√
2 = 1 +
1
. Suy ra a = √
1 1
= 2 + . Tà đó ta
a1
1
có
1 1
2 − 1 a2
a2 = √
2 − 1
= 2 +
a
.
M®t cách tương tự an
1
=
2 − 1 với moi n ≥ 1. Vì the
√
2 = [1, 2, 2, 2 . . .].
Tiep theo là m®t so tính chat của liên phân so vô hạn được dùng cho phan
cháng minh sau.
Chú j 1.1.13. Với hai dãy so nguyên dương {ai}∞
i=0 và bi = 1, i = 0, 1, . . . ta
xét các dãy truy hoi sau
p0 = a0, p1 = a1p0 + 1, pn = anpn−1 + pn−2,
q0 = 1, q1 = a1,n = anqn−1 + qn−2.
Khi đó các tính chat sau là đúng:
(i) πi
=
pi
, với moi i = 0, 1, . . . .
qi
(ii) pn và qn là nguyên to cùng nhau, nghĩa là πn là phân so toi giản.
(iii) α =
pn−1αn + pn−2
, với moi n ≥ 2.
(iv) lim
k→∞
qn 1αn + qn 2
πk = α.
n+1
(v) πn — πn−1 =
(−1)
, với moi n 1.
qn−1qn
Bo đe 1.1.14. Cho
√
d là so vô tí. Khi đó, ton tại vô so c¾p so nguyên dương
(p, q) thóa mãn:
(i) . d − . <
.
. q.
. q2
√
(ii) .p2
− dq2. < 1 + 2 d.
Chúng minh. (i) Theo tính chat của liên phân so vô hạn ta có
pk 1
. qk
. qkqk+1
với α ∈ R. Theo cách xác định {qk} ta có
qk+1 = akqk + qk−1 > ak−1qk−1 + qk−2 = qk.
Vì the
pk 1 1 1
∀k : α − < < = .
√
1
9
Viết đề tài giá sinh viên – ZALO:0973.287.149-TEAMLUANVAN.COM
√ p 1
q
√
2 2
y
. q. . .
q
.
p + q
√
d
.
=
.
p − q
√
d + 2q
√
d
.
≤
.
q
√
d − p
.
+
.
2q
√
d
.
y 2y2
y
−
y
Chon
(ii) Th t v y, ta có
p = pk ; q = qk ; α =
√
d.
. d − . < ⇒ 0 <
.
q
√
d − p
.
<
1
⇒ q > 0.
. .
<
.
1
+ 2q
√
d ≤ q + 2
.
q
√
d
.
= 1 +
.
2
√
d
.
q.
.
Suy ra .p − dq . < 1 + 2 d. Ta có đieu can cháng minh.
Bo đe 1.1.15. Giả sủ d và n là so nguyên sao cho d > 0 và d không là so chính
phương, |n| <
√
d. Khi đó neu x2
− dy2
= n thì
x
là m®t giản phân của liên
phân so của
√
d với x, y ∈ Z và y > 0.
Chúng minh. Đe cháng minh bő đe, ta xét hai trường hợp. Trường hợp thá
nhat, với n > 0 ta có x2
− dy2
= n hay (x + y
√
d)(x − y
√
d) = n, suy ra
x − y
√
d > 0, v y x > y
√
d. Do đó
x
−
√
d > 0. Hơn nǎa, vì 0 < n <
√
d nên
x √ x − y
√
d x2
− dy2
d = =
y y y
n x + y
√
d
n
=
y x + y
√ <
2y
√
d
√
d 1
<
2y2
√
d
=
2y2
.
Do 0 <
x
−
√
d <
1 x
ta suy ra được là giản phân của liên phân so của
√
d.
Trường hợp thá 2, với n < 0 chia hai ve của x2
− dy2
= n cho −d ta được
y2
−
1
x2
= −
n
.
d d
n y
L p lu n tương tự như trên, neu −
d
> 0 thì là giản phân của liên phân so
x
của
1
√
d
. Do đó
1 x 1
= là giản phân của liên phân so của
y 1
x √
d
=
√
d.
y
d
Mà
q2
y
10
Viết đề tài giá sinh viên – ZALO:0973.287.149-TEAMLUANVAN.COM
k
k
k+1
d − s
k+1
= k
+ k k
k
+ 2aksk − aktk
V y 2aksk − a2
tk ∈ Z và tk| d − s2
theo giả thuyet quy nạp nên tk+1 ∈ Z.
Bo đe 1.1.16. Cho liên phân so
√
d = [a ;a1 , a2 , a3 , ..., an , ...] và các dãy so
{sk}, {tk} được xác đ nh bang h thúc:
s0 = 0 ; t0 = 1
d − s2
sk+1 = aktk − sk ; tk+1 =
k+1
tk
với k ≥ 0.
Khi đó các phát bieu sau là đúng:
(i) sk, tk ∈ Z và tk =
/ 0;
(ii) tk| d − s2
;
sk +
√
d
(iii) xk =
k k ≥ 0 trong đó xk = [a0; a1, ..., ak].
Chúng minh. Ta cháng minh (i) và (ii) bang quy nạp theo k. Với k = 0 thì
khȁng định là hien nhiên. Giả sả với k > 0 và sk, tk được xác định như trên thì
sk, tk ∈ Z , tk 0 và tk| d − s2
.
Ta can cháng minh (i) và (ii) đúng với k + 1. Th t v y,
sk+1 = aktk − sk ∈ Z.
M t khác tk+1
thuan). Ta có
0, vì neu tk+1 = 0 thì d = s2
⇒ d là so chính phương (mâu
tk+1 =
=
2
k+1
tk
d − (aktk − sk)2
tk
d − s2
2aktksk − a2
t2
tk tk
d − s2
2
k
Suy ra
k
tk+1| d − s2
.
t
k
0
= .
t
11
Viết đề tài giá sinh viên – ZALO:0973.287.149-TEAMLUANVAN.COM
d − 2
s
0 1 2 3 k k+1
(
sk+1qkpk + qk−1tk+1pk = p ,
k
k k
(iii) Ta cháng minh bang quy nạp theo k. Với k = 0, luôn đúng. Giả sả với
sk +
√
d
k > 0 ta có xk =
ta có
. Ta can cháng minh (iii) đúng với k + 1. Th t v y,
tk
1
xk+1 =
k
=
s
ak
tk
+
√
d − a t
k k k
tk
= √
d − sk+1
tk sk+1 +
√
d
k+1
s +
√
d
Bo đe 1.1.17. Neu
pk
qk
=
k+1
tk+1
là giản phân của liên phân so của
√
d thì
p2
− dq2
= (−1)k+1
tk+1 , k = 0, 1, 2, ... và tk+1 > 0.
Chúng minh. Giả sả
√
d = [a ; a , a , a , ...a , x ] . Khi đó, ta có
√
d =
xk+1pk + pk−1
và xk+1
s +
√
d
= k+1
. V y
tk+1
sk+1 +
√
d
.q + q
xk+1qk + qk−1
!
√
d =
sk+1 +
√
d
.p + p .
Suy ra
tk+1
k k−1
tk+1
k k−1
√
d (sk+1qk + qk−1tk+1 − pk) = sk+1pk + tk+1pk−1 − dqk.
Do ve phải là so hǎu tỉ và
√
d là so vô tỉ nên
sk+1qk + qk−1tk+1 = pk,
sk+1pk + tk+1pk−1 = dqk.
Lan lượt nhân 2 ve của h thác với pk và −qk ta có
2
k
−sk+1pkqk − tk+1pk−1qk = −dq2
.
−
(
x
=
12
Viết đề tài giá sinh viên – ZALO:0973.287.149-TEAMLUANVAN.COM
2 2
∀ ≥
k k
k k
k k k
1
d <
C®ng ve theo ve ta được
p2
− dq2
= tk+1 (qk−1pk − qkpk−1) .
M t khác, vì qk−1pk − qkpk−1 = (−1)k+1
nên ta có
p2
− dq2
= (−1)k+1
tk+1.
Vì
p2k
q2k
<
√
d <
p2k+1
q2k+1
với moi k ≥ 0 nên neu k chȁn ta có
pk
<
√
d ⇒ p < q
√
d ⇒ p2
− dq2
< 0.
qk
Neu k lẻ ta có √
d <
pk
k k k k
⇒ q
√
d < p ⇒ p − dq > 0.
Ta xét bieu thác
qk
k k k k
p2
− dq2
(−1)k+1
tk+1 tk+1
k k
= = − (k ≥ 1) .
p2
−1 − dq2
−1 (−1)k
t tk
Neu k lẻ thì (k − 1) chȁn nên
p2
− dq2
> 0 ; p2
— dq < 0 ⇒ −
tk+1
< 0 ⇒
tk+1 > 0 ∀k ≥ 1.
k k k−1 k−1
tk tk
Neu k chȁn thì (k − 1) lẻ nên
p2
− dq2
< 0 ; p2
— dq > 0 ⇒ −
tk+1
< 0 ⇒
tk+1 > 0 ∀k ≥ 1.
k k k−1 k−1
tk tk
V y neu t0 = 1 > 0 thì
d − s2 h√ i √
Mà
t2
t1
> 0 ta suy ra t2 > 0. Tương tự ta có tk
> 0 k 0 mà
tk+1
tk
> 0 nên
suy ra tk+1 > 0 ∀ k ≥ 0.
Bo đe 1.1.18. So tự nhiên n thóa mãn
√
d = [a ;a1 ,a2 , ..., an ,...] = [a0 ;a1 ,a2 , ..., an−1 , an]
được goi là chieu dài chu kì của liên phân so của
√
d. Khi đó t
khi n| j.
= 1 khi và chí
2
2
0
0
t1 =
t0
= d − a2 > 0 a0 = d .
j
13
Viết đề tài giá sinh viên – ZALO:0973.287.149-TEAMLUANVAN.COM
s t +
√ √
t d
= s t+ t d,
1 kn+1
Chúng minh. Cho
√
d = [a0; a1, a2, ..., an]. Ta có xkn+1 = x1 với k ≥ 0. Khi đó
s +
√
d s +
√
d
kn+1
=
1
,
tkn+1 t1
khȁng định này tương đương với
kn+1 1 1 1 kn+1 kn+1
nghĩa là
(tkn+1 − t1)
√
d = skn+1t1 − s1tkn+1.
Ta có tkn+1 = t1, skn+1 = s1. M t khác,
t1 = d − s2
= d − s2 = tkntkn+1 = tknt1.
Suy ra tkn = 1. V y ket quả nh n được là tj = 1 , j = kn nên n|j. Ngược lại,
cho j là so nguyên dương thỏa mãn tj = 1. Khi đó, tà Bő đe 1.1.16 ta có
Vì sj ∈ Z nên ta có
s +
√
d
xj = j
= sj
tj
+
√
d.
[xj] =
h
sj +
√
d
i
= sj +
h√
d
i
= sj + a0.
Theo cách xác định của xj+1 ta có
xj = [xj] +
1
xj+1
= sj + a0 +
x
1
.
j+1
M t khác,
√
d = x 1
= a + và
√
d = x − s 1 1
= a + . Suy ra a + =
0 0
x1
1
j j 0
xj+1
0
x1
a0 +
xj+1
. Do đó x1 = xj+1. Đieu đó có nghĩa dãy a1, a2, ..., aj (j ∈ Z) l p lại
√
trong sự bieu dien liên phân so của
dài n chu kì của liên phân so
√
d.
d. Suy ra j = kn. V y j là b®i của chieu
1.2 Phương trình Diophantine x2
− Dy2
= ±1
Như chúng ta đã biet phương trình Pell là phương trình có dạng x2
− dy2
= n,
trong đó d và n là so nguyên cho trước. Khi n = ±1 đã có nhieu công trình liên
14
Viết đề tài giá sinh viên – ZALO:0973.287.149-TEAMLUANVAN.COM
quan đen vi c tìm nghi m nguyên của phương trình này. Trong mục này chúng
tôi trình bày lại phương pháp giải và cau trúc nghi m của m®t so phương trình
Pell đã biet. Các ket quả trong mục này được viet theo các tài li u [4].
1.2.1 Phương trình Pell dạng x2
− dy2
= 1
Xét phương trình
x2
− dy2
= 1 (1.3)
trong đó d là so nguyên cho trước.
Định lj 1.2.1. Neu d là so chính phương (d = m2
) thì phương trình (1.3)
không có nghi m nguyên dương.
Chúng minh. Vì d = m2
, m > 0 nên x2
− m2
y2
= 1, đieu này tương đương với
(x − my)(x + my) = 1.
Giả sả phương trình (1.3) có nghi m nguyên dương. Khi đó ta có (x − my) ∈ Z
và (x + my) ∈ Z≥0. Do đó
x + my = 1,
x − my = 1.
Vì x, y, m là các so nguyên dương nên x = 1 và y = 0. Đieu này là mâu thuan.
V y phương trình (1.3) không có nghi m nguyên dương.
M nh đe 1.2.2. Neu d là so nguyên âm thì phương trình (1.3) không có nghi m
nguyên dương.
Chúng minh. Đ t d = −dJ
với dJ
> 0. Khi đó phương trình (1.3) được viet lại
dưới dạng
x2
+ dJ
y2
= 1.
Neu dJ
= 1, thì x2
+ y2
= 1. Suy ra x = 1, y = 0 ho c x = 0, y = 1.
Neu dJ
> 1, thì x = 1 và y = 0.
V y phương trình (1.3) không có nghi m nguyên dương.
15
Viết đề tài giá sinh viên – ZALO:0973.287.149-TEAMLUANVAN.COM
√
2 2
h
i
√
√
S
S
x1 ≡ x2(mod |k|)
1 1 2 2
Định lý 1.2.1 và M nh đe 1.2.2 cho ta m®t so đieu ki n đe phương trình trên
không có nghi m nguyên dương. Định lý tiep theo chỉ ra đieu ki n can và đủ đe
phương trình có nghi m nguyên dương.
Định lj 1.2.3. Phương trình (1.3) có nghi m nguyên dương khi và chí khi d là
so nguyên dương và không phải là so chính phương.
Chúng minh. Theo Định lý 1.2.1 và M nh đe 1.2.2 ta có phương trình (1.3) có
nghi m nguyên dương thì d là so nguyên dương và không chính phương. Ngược
lại, theo Bő đe 1.1.14 ta có ton tại vô so c p so (x, y) nguyên dương sao cho
.x − dy . < 1 + 2 d
Suy ra
−1 − 2
√
d < x2
− dy2
< 1 + 2
√
d (1.4)
Tà tính vô hạn của các c p so nguyên dương (x, y) nên ton tại k ∈ Z thỏa mãn
k ∈ −1 − 2 d, 1 + 2 d sao cho có vô so c p so nguyên dương (x, y) thỏa
mãn
Khi đó ta xét t p
x2
− dy2
= k.
H =
,
(x, y) : x2
− dy2
= k, với k ∈
h
−1 − 2
√
d, 1 + 2
√
d
i,
và
Hi,j = {(x, y) ∈ H : x ≡ i (mod |k| ) và y ≡ j (mod |k|) , 0 ≤ i, j ≤ |k|} .
Vì có vô so (x, y) nên suy ra |H| = +∞. Ta có 0 ≤ i, j ≤ |k| nên so c p
(i, j) là hǎu hạn. Tà định nghĩa của hai t p H và Hi,j suy ra H = Hi,j và
|H| = | Hi,j| = +∞. Do so c p (i, j) là hǎu hạn nên ton tại (i, j) sao cho
|Hi,j| = +∞.
Xét Hi,j thỏa mãn |Hi,j| = +∞. Khi đó ton tại (x1, y1) /= (x2, y2) sao cho
(xk, yk) ∈ Hi,j (k = 1, 2) và thỏa mãn
y1 ≡ y2(mod |k|)
x2
− dy2
= x2
− dy2
= k.
(1.5)
16
Viết đề tài giá sinh viên – ZALO:0973.287.149-TEAMLUANVAN.COM
(
−
1 1 2 2
2 2
1 1 2 2
Ta có
(x1 − y1
√
d)(x2 + y2
√
d) = x1x2 − dy1y2 +
√
d (x1y2 − x2y1) . (1.6)
Theo công thác (1.5) ta có
(
x1x2 − dy1y2 ≡ x2
− dy2
≡ 0 mod (|k|)
x1y2 − x2y1 ≡ x2y2 − x2y2 ≡ 0 mod (|k|) .
Do đó ton tại (u, v) ∈ Z thỏa mãn
x1x2 − dy1y2 = ku,
x1y2 − x2y1 = kv.
Ket hợp (1.5), (1.6), (1.7) và (1.8) ta được
(x1 y1
√
d)(x2 + y2
√
d) = k(u + v
√
d),
(x1 + y1
√
d)(x2 − y2
√
d) = k(u − v
√
d).
Nhân hai ve của h phương trình ta có
(1.8)
(x2
− x2
− u2
− ).
Vì (x2
− dy2
)(x2
− dy2
) = k2
suy ra u2
− dv2
= 1. Cuoi cùng ta can cháng minh
u và v là nhǎng so nguyên dương. Th t v y, vì u2
= 1 + dv2
suy ra u ≥ 1. Vì ta
chỉ xét nghi m dương nên ta có u > 0. Giả sả v = 0 ta có u2
= 1 suy ra u = 1.
V y
(x1 − y1
√
d)(x2 + y2
√
d) = k.
Theo (1.5) ta có x2
− dy2
= (x1 − y1
√
d)(x1 + y1
√
d) = k, đieu này suy ra
1 1
(x + y
√
d) = (x + y
√
d). (1.9)
Giả sả x — y
√
d = 0 ⇒
√
d =
x1
và do đó
√
d là so hǎu tỉ, đieu này là mâu
1 1
√ y1 √ √
thuan. V y x1 − y1 d /= 0. Tà (1.9) ta có x1 − x2 = (y2 − y1) d. Vì d so vô
tỉ và xi, yi ∈ Z+
nên
x1 − x2 = 0,
y1 − y2 = 0.
(
(1.7)
dy2
)( dy2
) = k2
( dv2
1 1 2 2
17
Viết đề tài giá sinh viên – ZALO:0973.287.149-TEAMLUANVAN.COM
(
(
j j
j j
Khȁng định này tương đương với
x1 = x2
y1 = y2
nghĩa là (x1, y1 = (x2, y2). Đieu này mâu thuan và do đó v > 0. V y ton tại c p
so nguyên dương (u, v) sao cho u2
− dv2
= 1.
Định lý tiep theo cho ta cau trúc nghi m của phương trình thông qua giản
phân của liên phân so.
Định lj 1.2.4. Cho
pk
qk
là giản phân của liên phân so của
√
d với n là chieu
dài của nó. Khi đó các phát bieu sau là đúng:
(i) Neu n chȁn, khi đó tat cả các nghi m dương của phương trình (1.3) được
cho bới:
x = pkn−1
y = qkn−1
(k ≥ 1) ;
(ii) Neu n lé, khi đó tat cả các nghi m dương của phương trình (1.3) được
cho bới:
x = p2kn−1
y = q2kn−1
(k ≥ 1) .
Chúng minh. Theo Bő đe 1.1.15 ta có x0 , y0 là nghi m của phương trình x2
−
dy2
= 1. Trong đó x0 = pj và y0 = qj
với
pj
qj
là giản phân của liên phân so của
√
d. Theo Bő đe 1.1.17 thì:
p2
− dq2
= (−1)j+1
tj+1 j = 0, 1, 2, ... và tj+1 > 0.
Do p2
− dq2
= 1 suy ra tj+1 = 1 và j + 1 là so chȁn. Theo bő đe 1.1.17 thì
n| (j + 1) suy ra j + 1 = nk
′
k
′
∈ Z . V y j = k
′
n − 1(k
′
, n ≥ 1). (i) Neu n
chȁn và (j + 1) chȁn nên
∀k
′
≥ 1 ; k
′
= k (k ≥ 1) ⇒ j = kn − 1.
V y nghi m của phương trình (1.3) là:
x = pkn−1 và y = qkn−1 (k ≥ 1) .
(
18
Viết đề tài giá sinh viên – ZALO:0973.287.149-TEAMLUANVAN.COM
−
(ii) Neu n lẻ và (j + 1) chȁn nên kJ
chȁn và đ t kJ
= 2k. Khi đó k ≥ 1 suy
ra j = 2kn − 1. V y nghi m của phương trình (1.3) là:
x = p2kn−1 và y = q2kn−1 với k ≥ 1.
Tiep theo ta xét m®t ví dụ minh hoa cho Định lý 1.2.4.
Ví dn 1.2.5. Tìm nghi m nhỏ nhat của phương trình x2
− 13y2
= 1.
Th t v y, ta có
√
13 = 3; 1, 1, 1, 1, 6 với chieu dài chu kì 5 là so lẻ. Nên ta có
các giản phân sau:
ak 3 1 1 1 1 6 1 1 1 6 1
pk 1 3 4 7 11 18 119 137 256 393 649
qk 0 1 1 2 3 5 33 38 71 109 180
Ta có tả so và mau so của giản phân
p10k−1
là dạng nghi m của phương trình
đã cho. Với k = 1 ta có g9 =
p9
q9
q10k 1
649
=
180
nên (649, 180) là c p nghi m bé
nhat của phương trình đã cho. Đe tìm các c p nghi m còn lại ta lan lượt the
k = 2, 3, 4, . . . vào công thác
p10k−1
q10k−1
và dựa vào bảng các giản phân.
Chú j 1.2.6. Nghi m (x, y) của phương trình Pell (1.3) được goi là nghi m
cực tieu neu x < u với (u, v) là nghi m bat kỳ khác của phương trình (1.3).
Nghi m cực tieu còn được goi là nghi m cơ bản.
Định lý tiep theo cho ta m®t công cụ hǎu hi u đe tìm nghi m của phương
trình Pell (1.3) thông qua nghi m cơ bản.
Định lj 1.2.7. Cho (a, b) là nghi m cơ bản của phương trình (1.3). Với mői
n = 0, 1, 2, ... các dãy {xn} và {yn} được xác đ nh như sau
x0 = 1 ; x1 = a ; xn+2 = 2axn+1 − xn,
y0 = 0 ; y1 = b ; yn+2 = 2ayn+1 − yn.
Khi đó, với moi n ∈ N ta có (xn, yn) là nghi m dương của phương trình (1.3).
(
19
Viết đề tài giá sinh viên – ZALO:0973.287.149-TEAMLUANVAN.COM
1 2
n n
yn = d1λn
+ d2λn
.
Chúng minh. Áp dụng lí thuyet ve dãy so ta có phương trình đ c trưng của dãy
trên là
λ2
− 2aλ + 1 = 0. (1.10)
Phương trình (1.10) có hai nghi m là
λ1 = a −
√
∆
′
và λ2 = a +
√
∆
′
∆
′
= a2
− 1 .
Vì (a, b) là m®t c p nghi m của phương trình x2
−dy2
= 1, nên ta có a2
−1 = db2
.
V y hai nghi m của phương trình (1.10) là
λ1 = a − b
√
d và λ2 = a + b
√
d.
Tà đó theo lý thuyet dãy so, thì:
(
xn = c1λn
+ c2λn
,
1 2
Bây giờ ta xác định c1, c2; d1, d2 tà các đieu ki n x0 = 1, x2 = a; y0 = 0, y1 = b.
Xét h phương trình sau
(
x0 = 1
⇔ c1 + c2 = 1,
√ √
x1 = a
Giải h trên ta có
c1 a − b d + c2 a + b d = a.
xn =
và
yn =
a + b
√
d
n
+ a − b
√
d
n
a + b
√
d
n
− a − b
√
d
n
2
√
d
.
Vì v y xn và yn là các so nguyên dương. Tà đó ta có
xn
x
+ yn
— y
√
d = a + b
√
d
n
,
√
d = a − b
√
d
n
.
Vì v y
x2
− dy2
= xn + yn
√
d xn − yn
√
d = a2
− db2
= 1.
2
n n
20
Viết đề tài giá sinh viên – ZALO:0973.287.149-TEAMLUANVAN.COM
1 1 1 1
n n
n n
1 − 1 = 0 < r + s
√
d − r − s
√
d = 2s
√
d
⇒ 2
s > 0.
n
1
Do đó
Do đó (xn, yn) là nghi m nguyên dương của phương trình Pell (1.3) với moi
n = 0, 1, 2, ...
Định lj 1.2.8. Cho (x1, y1) là nghi m cơ bản của phương trình x2
− dy2
= 1.
Với mői nghi m dương (x, y) bat kì của phương trình thì ton tại n ≥ 1 sao cho
x + y
√
d = x + y
√
d = x + y
√
d
n
.
n 1 1
Chúng minh. Ta dùng phương pháp cháng minh phản cháng. Giả sả u, v là
nghi m dương của phương trình mà không đạt được dạng x1 + y1
√
d . Tác
n
là u + v
√
d x1
+ y1√
d
n
. Giả sả
u + v
√
d > x1
+ y1
√
d
n
.
Vì x + y
√
d > 1 nên lũy thàa của x + y
√
d sě lớn tùy ý. Đieu đó có nghĩa là:
x1 + y1√
d
n
< u + v
√
d < x
+ y1
√
d
n+1
.
Suy ra
V y
xn + yn
√
d < u + v
√
d < xn + yn
√
d x1 + y1
√
d .
1 < u + v
√
d xn − yn
√
d < x1 + y1
√
d vì x2
− dy2
= 1.
So nguyên r, s được xác định bởi r + s
√
d = u + v
√
d xn − yn
√
d , hay
r + s
√
d = (uxn − dvyn) + (sxn − uyn)
√
d.
Suy ra r = uxn − dvyn ; s = sxn − uyn. Ta có
r2
− ds2
= x2
− dy2
u2
− dv2
= 1.
Do đó r, s là nghi m của phương trình (1.3). Ta cháng minh rang r, s > 0. Th t
v y, vì
1 < r + s
√
d < x + y
√
d, (1.11)
1
mà r + s
√
d r − s
√
d = 1 nên 0 < r−s
√
d < 1. Suy ra −1 < − r − s
√
d .
1 + 0 = 1 < r + s
√
d + r − s
√
d = 2r r >
1
1
21
Viết đề tài giá sinh viên – ZALO:0973.287.149-TEAMLUANVAN.COM
1 1
1 1
( (
( (
V y r, s là nghi m dương của phương trình (1.3) và x1 < r ; y1 < s. Suy ra
x + y
√
d < r + s
√
d.
Đieu này mâu thuan với (1.11). Do đó ton tại so n sao cho moi u, v là nghi m
nguyên dương của phương trình (1.3) đeu có dạng:
u + v
√
d = x + y
√
d
n
.
Ví dn 1.2.9. Tìm ba nghi m đau tiên của phương trình: x2
− 23y2
= 1
Th t v y, trước tiên ta xác định nghi m cơ bản của phương trình (*). Bang
phương pháp the y = 1, 2, 3, ... vào bieu thác 23y2
+ 1 ta có
Với y = 1 thì x2
= 23 + 1 = 24 loại.
Với y = 2 thì x2
= 23.4 + 1 = 92 loại.
Với y = 3 thì x2
= 23.32 + 1 = 208 loại.
Với y = 4 thì x2
= 23.42 + 1 = 369 loại.
Với y = 5 thì x2
= 23.52 + 1 = 576 nên x = 24.
V y nghi m cơ bản là (24, 5). Ta có
xn+2 = 2axn+1 − xn
yn+2 = 2ayn+1 − yn
⇒
x2 = 2ax1 − x0
y2 = 2ay1 − y0
⇒
x2 = 2.24.24 − 1 = 1151
y2 = 2.24.5 − 0 = 240
V y nghi m thá hai của phương trình đã cho là (1151, 240). Tương tự, ta tìm
nghi m thá ba như trên:
x3 = 2ax2 − x1
y3 = 2ay2 − y1
⇒
x3 = 2.24.1151 − 24 = 55224
y3 = 2.24.240 − 5 = 11515.
V y nghi m thá ba của phương trình (*) là (55224, 11515).
1.2.2 Ứng dnng liên phân so
√
D vào phương trình Pell x2
− Dy2
= 1
Trong phan này, chúng tôi trình bày lại m®t so ket quả ve áp dụng liên phân
so của
√
D trong đó D = {k2
+ 1, k2
− 1, k2
+ 2, k2
− 2, k2
+ k, k2
− k}, (với k
là so nguyên dương tùy ý) đe mô tả cau trúc nghi m nguyên của phương trình
(1.3). Các ket quả trong mục này được viet theo tài li u [4].
(
22
Viết đề tài giá sinh viên – ZALO:0973.287.149-TEAMLUANVAN.COM
(
D =
−
Định lj 1.2.10. Cho k ≥ 1 là so nguyên và D = k2
+ 1. Khi đó các phát bieu
sau là đúng:
(i) Liên phân so của
√
D là
√ 1; 2 khi k = 1,
k; 2k khi k > 1.
(ii) (x1, y1) = (2k2
+ 1, 2k) là nghi m cơ bản. T¾p {(xn, yn)}, thóa mãn
xn
= [k; 2k, ..., 2k] (1.12)
`
2n−1
˛¸
lan
x
với n ≥ 2 là nghi m của phương trình x2
− (k2
+ 1)y2
= 1 và ta có:
(a) Hai nghi m liên tiep (xn, yn) và (xn+1, yn+1) thóa mãn
xn+1 = (2k2
+ 1)xn + (2k3
+ 2k)yn, yn+1 = 2kxn + (2k2
+ 1)yn, với n ≥ 1.
(b) Các nghi m (xn, yn) thóa mãn các moi quan h sau đây
xn = (4k2
+ 1)(xn−1 + xn−2) − xn−3,
yn = (4k2
+ 1)(yn−1 + yn−2) − yn−3 với n ≥ 4.
Chúng minh. (i) Cho D = k2
+ 1. Neu k = 1 ta có
√
2 = [1; 2]. Với k > 1 ta có
√
k2 + 1 = k + (
√
k2 + 1 k) = k +
1
1
√
k2 + 1 − k
1 1
= k + √
k2 + 1 + k
= k +
2k + (
√
k2 + 1 − k)
.
V y ta có
√
D = [k; 2k].
(ii) Cho
√
D = [a0; a1, ..., al] là liên phân so có đ® dài tuan hoàn l. T p
A−2 = 0, A−1 = 1, Ak = akAk−1 + Ak−2
và B−2 = 1, B−1 = 0, Bk = akBk−1 + Bk−2 với so nguyên không âm k. Khi đó
C =
Ak
k
Bk là giản phân thá k của
√
D, và nghi m cơ bản của phương trình (1.3)
là (x1, y1) = (Al−1, Bl−1) neu l là so chȁn và (x1, y1) = (A2l−1, B2l−1) neu l là
so lẻ. Hơn nǎa, theo cháng minh trên ta có
√
D = [k; 2k]. V y A0 = k, A1 =
yn
23
Viết đề tài giá sinh viên – ZALO:0973.287.149-TEAMLUANVAN.COM
n+1
n n
2k2
+ 1, B0 = 1, B1 = 2k. Do đó (x1, y1) = (2k2
+ 1, 2k) là nghi m cơ bản. Bây
giờ, giả sả rang (xn, yn) là nghi m của phương trình x2
− (k2
+ 1)y2
= 1. thì
x2
− (k2
+ 1)y2
= 1. Áp dụng (1.12) ta có
n n
xn+1
yn+1
1
= k + 1
2k + xn
k +
yn
(2k2
+ 1)x)n + (2k3
+ 2k)yn
Khi đó ta có
=
2kxn
.
+ (2k2 + 1)yn
2
n+1 — (k2
+ 1)y2 = [(2k2
+ 1)x)n + (2k3
+ 2k)yn]2
−(k2
+ 1)[2kxn + (2k2
+ 1)yn]2
= x2
− (k2
+ 1)y2
= 1.
V y (xn+1, yn+1) là nghi m của phương trình x2
− (k2
+ 1)y2
= 1.
(a) Theo cháng minh phan 2 ta có xn+1 = (2k2
+ 1)xn + (2k3
+ 2k)yn và
yn+1 = 2kxn + (2k2
+ 1)yn với n ≥ 1.
(b) Ta cháng minh
xn = (4k2
+ 1)(xn−1 + xn−2) − xn−3 (1.13)
bang phương pháp quy nạp. Áp dụng (1.12) ta có
x1 = 2k2
+ 1, x2 = 8k4
+ 8k2
+ 1, x3 = 32k6
+ 48k4
+ 18k2
+ 1
và x4 = 128k8
+ 256k6
+ 160k4
+ 32k2
+ 1. Ta kiem tra (1.13) đúng với n = 4.
Th t v y
x4 = (4k2
+ 1)(x3 + x2) − x1
= (4k2
+ 1)(32k6
+ 48k4
+ 18k2
+ 1 + 8k4
+ 8k2
+ 1) − 2k2
+ 1 .
= 128k8
+ 256k6
+ 160k4
+ 32k2
+ 1.
Ta giả sả rang công thác (1.13) đúng với n − 1, tác là
xn−1 = (4k2
+ 1)(xn−2 + xn−3) − xn−4.
Ta phải cháng minh (1.13) đúng với n. Th t v y, theo cháng minh trên ta có
xn+1 = (2k2
+ 1)xn + (2k3
+ 2k)yn. Do đó
xn−1 = (2k2
+ 1)xn−2 + (2k3
+ 2k)yn−2,
x
24
Viết đề tài giá sinh viên – ZALO:0973.287.149-TEAMLUANVAN.COM
xn−2 = (2k2
+ 1)xn−3 + (2k3
+ 2k)yn−4,
xn−3 = (2k2
+ 1)xn−4 + (2k3
+ 2k)yn−3.
V y công thác (1.13) đúng với moi n ≥ 4.
Tiep theo chúng tôi nhac lại m®t so ket quả tương tự cho các trường hợp
khác của D mà không trình bày cháng minh của các định lý đó vì các cháng
minh được cháng minh như trong cách cháng minh Định lý 1.2.10.
Định lj 1.2.11. Cho k ≥ 1 là so nguyên và D = k2
+ 2. Khi đó các phát bieu
sau là đúng:
(i) Liên phân so của
√
D là
√
D = [k; k, 2k].
(ii) (x1, y1) = (k2
+ 1, k) là nghi m cơ bản. T¾p {(xn, yn)}, thóa mãn
xn
= [k; k; 2k, ..., k, 2k, k] (1.14)
`
n−1
˛¸
lan
x
với n ≥ 2 là nghi m của phương trình x2
− (k2
+ 2)y2
= 1 và ta có:
(a) Hai nghi m liên tiep (xn, yn) và (xn+1, yn+1) thóa mãn
xn+1 = (k2
+ 1)xn + (k3
+ 2k)yn, yn+1 = kxn + (k2
+ 1)yn với n ≥ 1.
(b) Các nghi m (xn, yn) thóa mãn các moi quan h sau đây
xn = (2k2
+ 1)(xn−1 + xn−2) − xn−3,
yn = (2k2
+ 1)(yn−1 + yn−2) − yn−3 với n ≥ 4.
Trường hợp D = {k2
− 1, k2
− 2, k2
+ k, k2
− k} được the hi n trong các định
lý sau:
Định lj 1.2.12. Cho k ≥ 2 là so nguyên và D = k2
− 1.
(i) Liên phân so của
√
D là
√
D = [k − 1; 1, 2k − 2].
yn
25
Viết đề tài giá sinh viên – ZALO:0973.287.149-TEAMLUANVAN.COM
— − −
` ˛¸ x
n
(ii) (x1, y1) = (k, 1) là nghi m cơ bản. T¾p {(xn, yn)}, trong đó
xn
= [k; 1; 2k 2, ..., 1, 2k 2, 2k 1] (1.15)
`
n−2
˛¸
lan
x
với n ≥ 2 là nghi m của phương trình x2
− (k2
− 1)y2
= 1 và ta có:
(a) Hai nghi m liên tiep (xn, yn) và (xn+1, yn+1) thóa mãn
xn+1 = kxn + (k2
− 1)yn, yn+1 = xn + kyn với n ≥ 1.
(b) Các nghi m (xn, yn) thóa mãn các moi quan h sau đây
xn = (2k − 1)(xn−1 + xn−2) − xn−3,
yn = (2k − 1)(yn−1 + yn−2) − yn−3 với n ≥ 4.
Định lj 1.2.13. Cho k ≥ 2 là so nguyên và D = k2
− 2. Khi đó các phát bieu
sau là đúng:
(i) Liên phân so của
√
D là
√
D =
(
1; 2 khi k = 2
(ii) (x1, y1) = (k2
− 1, k) là nghi m cơ bản. T¾p {(xn, yn)}, trong đó
xn
y
= [k − 1; 1, k − 2, 1, 2k − 2, ..., 1, k − 2, 1, 2k − 2, 1, k − 1]. (1.16)
n−1 lan
với n ≥ 2 là nghi m của phương trình x2
− (k2
− 2)y2
= 1 và ta có:
(a) Hai nghi m liên tiep (xn, yn) và (xn+1, yn+1) thóa mãn
xn+1 = (k2
− 1)xn + (2k3
− 2k)yn, yn+1 = kxn + (k2
− 1)yn với n ≥ 1.
(b) Các nghi m (xn, yn) thóa mãn các moi quan h sau đây
xn = (2k2
− 3)(xn−1 + xn−2) − xn−3,
yn = (2k2
− 3)(yn−1 + yn−2) − yn−3 với n ≥ 4.
k − 1; 1, k − 2, 2k − 2 khi k > 2.
yn
26
Viết đề tài giá sinh viên – ZALO:0973.287.149-TEAMLUANVAN.COM
k; k, 2k khi
k > 1.
D =
(
— − −
Định lj 1.2.14. Cho k ≥ 1 là so nguyên và D = k2
+ k. Khi đó các phát bieu
sau là đúng:
(i) Liên phân so của
√
D là
√
D =
(
1; 2 khi k = 1,
(ii) (x1, y1) = (2k + 1, 2) là nghi m cơ bản. T¾p {(xn, yn)}, trong đó
xn
= [k; k, 2k, ..., k, 2k, 2]. (1.17)
`
n−1
˛¸
lan
x
với n ≥ 2 là nghi m của phương trình x2
− (k2
+ k)y2
= 1 và ta có:
(a) Hai nghi m liên tiep (xn, yn) và (xn+1, yn+1) thóa mãn
xn+1 = (2k + 1)xn + (2k2
+ 2k)yn, yn+1 = 2xn + (2k + 1)yn với n ≥ 1.
(b) Các nghi m (xn, yn) thóa mãn các moi quan h sau đây
xn = (4k + 1)(xn−1 + xn−2) − xn−3,
yn = (4k + 1)(yn−1 + yn−2) − yn−3 với n ≥ 4.
Định lj 1.2.15. Cho k ≥ 2 là so nguyên và D = k2
− k. Khi đó các phát bieu
sau là đúng
(i) Liên phân so của
√
D là
√ 1; 2 khi k = 2,
k − 1; 2, 2k − 2 khi k > 2.
(ii) (x1, y1) = (2k − 1, 2) là nghi m cơ bản. T¾p {(xn, yn)}, trong đó
xn
= [k 1; 2, 2k 2, ..., 2, 2k 2, 2] (1.18)
`
n−1
˛¸
lan
x
với n ≥ 2 là nghi m của phương trình x2
− (k2
− k)y2
= 1 và ta có:
(a) Hai nghi m liên tiep (xn, yn) và (xn+1, yn+1) thóa mãn
xn+1 = (2k − 1)xn + (2k2
− 2k)yn, yn+1 = 2xn + (2k − 1)yn với n ≥ 1.
(b) Các nghi m (xn, yn) thóa mãn các moi quan h sau đây
xn = (4k − 3)(xn−1 + xn−2) − xn−3,
yn = (4k − 3)(yn−1 + yn−2) − yn−3 với n ≥ 4.
yn
yn
27
Viết đề tài giá sinh viên – ZALO:0973.287.149-TEAMLUANVAN.COM
0 0
0 0 0 + 1 .d
1.2.3 Phương trình Pell dạng x2
− dy2
= −1
Trong phan này chúng tôi sě trình bày lại m®t so ket quả ve cau trúc nghi m
của phương trình
x2
− dy2
= −1 (1.19)
trong đó d là so nguyên tùy ý cho trước. Đ c bi t, chúng tôi trình bày lại moi
liên h giǎa nghi m của phương trình x2
− dy2
= 1 với nghi m của phương trình
(1.19).
Định lj 1.2.16. Phương trình (1.19) không có nghi m nguyên dương khi d =
m2
với m > 0 là so nguyên (túc là d là so chính phương).
Chúng minh. Cho m là so nguyên dương và d = m2
thì phương trình (1.19) có
dạng
x2
− m2
y2
= −1,
khȁng định này tương đương với
(my + x) (my − x) = 1.
Tà tính nguyên dương của x và y nên suy ra:
my + x = 1,
my − x = 1.
Suy ra x = 0. Đieu này là mâu thuan vì x > 0. V y phương trình Pell (1.19)
không có nghi m nguyên dương, khi d là so chính phương.
Định lj 1.2.17. Phương trình (1.19) không có nghi m khi d có ước nguyên to
p = 4k + 3.
Chúng minh. Giả sả khi d có ước nguyên to dạng p = 4k + 3, mà phương trình
van có nghi m
(x0, y0) : x2
− dy2
= −1.
Đieu này tương đương với x2
+1 = dy2
. Tà đó suy ra: x2 . . Vì p = 4k + 3
nên theo lý thuyet chia het suy ra 1 chia het cho p. Đieu này là mâu thuan, v y
giả thuyet là sai và phương trình (1.19) không có nghi m trong trường hợp này.
(
28
Viết đề tài giá sinh viên – ZALO:0973.287.149-TEAMLUANVAN.COM
d ≡ 1 (mod4)
Định lj 1.2.18. Cho d là so nguyên to, khi đó phương trình (1.19) có nghi m
nguyên dương khi và chí khi d không có dạng 4k + 3.
Chúng minh. Giả sả phương trình (1.19) có nghi m. Khi đó theo định lý 1.2.11
thì d không có dạng 4k + 3. Ta cháng minh đieu ngược lại. Giả sả d /= 4k + 3
.
Ta xét các trường hợp sau:
d ≡ 2 (mod4)
d ≡ 0 (mod4)
Do d là so nguyên to nên chỉ còn 2 trường hợp:
d ≡ 2 (mod4) ,
d ≡ 1 (mod4) .
Trường hợp 1: d ≡ 2 (mod4) hay d có dạng d = 4k + 2 nên d chia het cho 2. Vì
d là so nguyên to nên d = 2. V y phương trình (1.19) có nghi m nguyên dương
(1, 1).
Trường hợp 2: d ≡ 1 (mod4) hay d = 4k + 1. Xét phương trình Pell (1.3)
x2
− dy2
= 1 là phương trình liên ket với phương trình (1.19). Goi (a, b) là
nghi m dương nhỏ nhat của phương trình (1.3), khi đó ta có:
a2
− 1 = db2
. (1.20)
Ta xét hai khả năng sau:
Neu a chȁn, thì ve trái của (1.20) lẻ và do d = 4k + 1 là so lẻ nên b lẻ. Do đó
b2
≡ 1 (mod4) . Suy ra
a2
− 1 ≡ 1.1 (mod4) nên a2
≡ 2 (mod4) .
Đieu này mâu thuan với giả thiet (a chȁn nên a2
≡ 0 (mod4)). Vì the không
xảy ra khả năng này.
Neu a lẻ, khi đó l p lu n tương tự như trên ta có b chȁn. Giả sả
a = 2a1 + 1,
b = 2b1.
Thay vào (1.20) ta được:
(2a1 + 2) 2a1 = d.4b2
nên (a1 + 1) a1 = db2
. (1.21)
1 1
"
(
29
Viết đề tài giá sinh viên – ZALO:0973.287.149-TEAMLUANVAN.COM
(
(
(
(
a1 + 1 = dv2
,
0 0 0 0 0 0 0 0
Vì d là so nguyên to và (a1, a1 + 1) = 1 nên tà (1.21) suy ra:
a1 = u2
.
a1 = du2
a1 + 1 = v2
Trong đó u.v = b1 với u, v là so nguyên dương. Neu
a1 + 1 = dv2
a1 = u2
,
suy ra u2
−dv2
= −1. Trong trường hợp này (u, v) là nghi m nguyên dương của
phương trình (1.19). Neu
a1 = du2
a1 + 1 = v2
,
suy ra v2
− du2
= 1. Lúc này (v, u) là nghi m của phương trình (1.3). Do (a, b)
là nghi m cơ bản của phương trình (1.3) nên ta có v ≥ a. Tà đó:
a1 + 1 = v2
≥ v ≥ a = 2a1 + 1 nên a1 ≥ 2a1.
Bat đȁng thác thu được mâu thuan vì a1 là so nguyên dương. Do đó trường hợp
này không the xảy ra.
Định lj 1.2.19. Goi (a, b) là nghi m cơ bản của phương trình liên ket với
phương trình Pell (1.19). Khi đó phương trình Pell (1.19) có nghi m khi và chí
khi h
có nghi m nguyên dương.
a = x2
+ dy2
b = 2xy
(1.22)
Chúng minh. Giả sả (x0, y0) là nghi m nguyên dương của h (1.22). Vì (a, b) là
nghi m của phương trình x2
− dy2
= 1 nên a2
− db2
= 1. Tà đó theo h ta có
1 = x2
+ dy2 2
− d(2x y )2
nên 1 = x2
− dy2 2
⇒ x2
− dy2
= ±1.
(
30
Viết đề tài giá sinh viên – ZALO:0973.287.149-TEAMLUANVAN.COM
0 0
0 0
0 0 0 0 0 0
0 0 0 0 0 0 0 0
Neu x2
− dy2 = 1 thì (x0, y0) là nghi m nguyên dương của phương trình Pell
liên ket. Do (a, b) là nghi m nguyên dương nhỏ nhat của phương trình này nên
x0 ≥ a = x2
+ dy2
> x0. Đieu này là đieu mâu thuan.
Neu x2
− dy2
= −1. Khi đó (x0, y0) là nghi m nguyên dương của phương trình
0 0
(1.19).
Ngược lại, giả sả phương trình (1.19) có nghi m nguyên dương. Khi đó goi
(x0, y0) là nghi m nguyên dương nhỏ nhat của nó. Ta sě cháng minh rang
(x0, y0) chính là nghi m của h (1.22). Th t v y, đ t u = x2
+ dy2
; v = 2x0y0.
0 0
Vì x2
− dy2
= −1 ta có
0 0
u2
− dv2
= x2
+ dy2 2
− d(2x y )2
= x2
− dy2 2
= 1.
V y (u, v) là m®t nghi m của phương trình Pell liên ket của phương trình (1.19).
Tà tính nhỏ nhat của (a, b) suy ra u ≥ a ; v ≥ b. Ta sě cháng minh u = a; v = b
khi đó (x0, y0) là nghi m của h (1.22). Giả sả ngược lại, tác là u > a, v > b.
Ta có:
0 < a − b
√
d < a − b
√
d a + b
√
d = a2
− db2
= 1 nên 0 < a − b
√
d < 1.
Suy ra
V y
a − b
√
d x0 + y0
√
d < x0 + y0
√
d.
(ax0 − bdy0) + (ay0 − bx0)
√
d < x0 + y0
√
d. (1.23)
Tà a < u, b < v ta có
a + b
√
d < u + v
√
d = x0 + y0
√
d .
2
Suy ra
— (ax0 − bdy0) + (ay0 − bx0)
√
d = a + b
√
d −x0 + y0
√
d
< x + y
√
d
2
−x + y
√
d = x + y
√
d dy2
− x2
.
Vì x2
− dy2
= −1 nên
0 0
— (ax0 − bdy0) + (ay0 − bx0)
√
d < x0 + y0
√
d dy2
− x2
0 0
= x0 + y0
√
d.
(1.24)
31
Viết đề tài giá sinh viên – ZALO:0973.287.149-TEAMLUANVAN.COM
0
√ √
0 0 0 0
0 0 0 0
0
⇔ y2
> −b2
.
0 0 0
0
Đ t s = ax0 − bdy0; t = ay0 − bx0. Khi đó (1.23), (1.24) có dạng như sau
s + t
√
d < x + y
√
d,
0
Ta có
−s + t
√
d < x0 + y0
√
d.
s2
− dt2
= (ax0 − bdy0)2
− d(ay0 − bx0)2
= a2
x2
+ b2
d2
y2
− da2
y2
− db2
x2
(1.25)
= a2
x2
− dy2
+ b2
d dy2
− x2
.
Do (x0, y0) là nghi m của phương trình (1.19) nên x2
− dy2
= −1. V y tà (1.25)
0 0
ta có các đȁng thác
s2
− dt2
= −a2
+ db2
= − a2
− db2
. (1.26)
Lại do (a, b) là nghi m của phương trình Pell liên ket, nên tà (1.26) ta có
s2
− dt2
= −1.
Giả sả s = 0 ta suy ra dt2
= 1. Do tính nguyên dương của d và t nên d = t = 1.
Đieu đó vô lý vì d không là so chính phương. Do đó s khác 0. Tiep theo ta can
cháng minh t > 0. Th t v y:
t > 0 ⇔ ay0 − bx0 > 0 ⇔ ay0 > bx0
⇔ a2
y2
> b2
x2
⇔ 1 + db2
y2
> b2
dy2
− 1
V y t > 0. Do s khác 0 nên chỉ có hai trường hợp xảy ra:
Neu s > 0. Khi đó (s, t) là nghi m nguyên dương của phương trình (1.19), mà
(x0, y0) là nghi m dương bé nhat của phương trình này nên:
s ≥ x0 và t ≥ y0 nên s + t
√
d ≥ x0 + y0
√
d.
Tà đó ta suy ra mâu thuan nên không xảy ra trường hợp s > 0.
Neu s < 0 thì (−s, t) là nghi m nguyên dương của phương trình x2
− dy2
= −1.
Bang cách l p lu n tương tự trường hợp trên, ta dan đen −s+t d ≥ x0 +y0 d.
Tà đó ta cũng dan đen mâu thuan. V y giả thuyet cháng minh phản cháng
(u, v) /= (a, b) là sai. Đieu đó có nghĩa là (u, v) = (a, b).
32
Viết đề tài giá sinh viên – ZALO:0973.287.149-TEAMLUANVAN.COM
1 2
√ √
3 2
⇒ −2b dc
= u + 3duv − u a + b d
1
−
−
1 1
(
x0 = u; x1 = u3
+ 3duv2
; xn+2 = 2axn+1 − xn,
1 2
b = 2xy
Định lj 1.2.20. Cho (a, b) là nghi m cơ bản của phương trình Pell liên ket
(1.3) của phương trình (1.19) và h
(
a = x2
+ dy2
có nghi m (u, v) duy nhat. Xét hai dãy so nguyên dương {xn} và {yn} sau đây:
y0 = v; y1 = dv3
+ 3u2
v; yn+2 = 2ayn+1 − yn.
Khi đó (xn, yn) là nghi m của phương trình (1.19).
n = 0, 1, 2, 3...
Chúng minh. Theo lý thuyet ve dãy so, thì phương trình đ c trưng của dãy là
λ2
− 2aλ + 1 = 0. (1.28)
Phương trình (1.28) có hai nghi m là
λ1 = a −
√
∆
′
và λ2 = a +
√
∆
′
, ∆
′
= a2
− 1.
Do (a, b) là nghi m của phương trình (1.3) nên a2
− db2
= 1, tà đó ta có:
∆
′
= db2
. V y hai nghi m của phương trình đ c trưng (1.28) là
λ1 = a − b
√
d và λ2 = a + b
√
d.
Theo lý thuyet dãy so thì xn = c1λn
+ c2λn
. Bây giờ ta xác định c1; c2 tà đieu
1 2
ki n x0 = u; x1 = u3
+ 3duv2
. Xét h phương trình sau:
c1 a − b
√
c + c = u,
d + c2 a + b
√
d = u3
+ 3duv2
.
(1.29)
Giải h (1.29) ta có c2 = u − c1 và
c a b
√
d + (u c ) a + b
√
d = u3
+ 3duv2
⇒ c1 a − b
√
d − a − b
√
d = u3
+ 3duv2
− u a + b
√
d .
⇒ −2b
√
dc1 = u3
+ 3duv2
− u u2
+ dv2
+ 2uv
√
d
⇒ −2b
√
dc1 = 2duv2
− 2u2
v
√
d
⇒ −2uv
√
dc1 = uv
√
d v
√
d − u
⇒ c =
u − v
√
d
⇒ c = u −
u − v
√
d
=
u + v
√
d
.
(1.27)
2 2 2
33
Viết đề tài giá sinh viên – ZALO:0973.287.149-TEAMLUANVAN.COM
√
√
2 √ 2 √
n n
−
b = 2xy
V y
x =
u − v
√
d
a − b
√
d
n
+
u + v
√
d
a + b
√
d
n
=
u − v d
u2
+ dv2
− 2uv
√
d
n
+
u + v d
u2
+ dv2
+ 2uv
√
d
n
2 2
u + v d
2n+1
+ u v d
2n+1
= ).
2
(1.30)
Tương tự ta có
yn =
u + v
√
d
2n+1
− u − v
√
d
2n+1
2
√
d
. (1.31)
Tà (1.30) và (1.31), ta có:
xn + yn
√
d = u + v
√
d
2n+1
,
xn — yn √
d = u − v
√
d
2n+1
.
Nhân đȁng thác trên ta có
x2
− dy2
= u2
− dv2 2n+1
.
Theo cháng minh của Định lý 1.2.4, thì do (u, v) là nghi m của h (1.27) nên
u2
− dv2
= −1. Do đó x2
− dy2
= −1. V y (xn, yn) là nghi m nguyên dương của
n n
phương trình (1.19).
Định lj 1.2.21. Cho (a, b) là nghi m của phương trình Pell liên ket (1.19). H
(
a = x2
+ dy2
có nghi m duy nhat (u, v). Goi (x, y) là nghi m của phương trình Pell (1.19)
Khi đó, ton tại so tự nhiên n sao cho
x + y
√
d = u + v
√
d
2n+1
= x + yn
√
d.
Chúng minh. Xét so sau đây
x + y
√
d u + v
√
d .
n
(1.32)
n
34
Viết đề tài giá sinh viên – ZALO:0973.287.149-TEAMLUANVAN.COM
(
Ta có
x + y
√
d u + v
√
d = (xu + dyv) + (yu + xv)
√
d = s + t
√
d,
trong đó s = xu + dyv, t = yu + xv. M t khác
s2
− dt2
= (xu + dyv)2
− d(yu + xv)2
= x2
u2
+ d2
y2
v2
− dy2
u2
− dx2
v2
= x2
− dy2
u2
− dv2
= (−1) (−1) = 1.
V y (s, t) là nghi m của phương trình (1.3). Do (a, b) là nghi m bé nhat của nó,
nên theo Định lý 1.2.8 thì ton tại n ≥ 1 sao cho s + t
√
d = a + b
√
d
n+1
và
x + y
√
d u + v
√
d = a + b
√
d
n+1
. (1.33)
M t khác
a + b
√
d = u2
+ dv2
+ 2uv
√
d = u + v
√
d
2
,
thay vào (1.33) ta được
x + y
√
d u + v
√
d = u + v
√
d
2n+2
,
hay
x + y
√
d = u + v
√
d
2n+1
= x + yn
√
d. (1.34)
Đieu đó có nghĩa là moi nghi m (x, y) là nghi m của phương trình (1.19) và
được bieu dien ở dạng (1.34).
Ví dn 1.2.22. Tìm 3 nghi m đau tiên của phương trình x2
− 5y2
= −1.
Th t v y, trước tiên ta xác định nghi m cơ bản của phương trình Pell liên
ket x2
− 5y2
= 1 là (9, 4). Giải h phương trình sau
x2
+ 5y2
= 9
2xy = 4
ta được nghi m nguyên duy nhat là (2, 1). V y ta có x0 = 2 ; y0 = 1 đây là
nghi m bé nhat của phương trình đã cho.
n
35
Viết đề tài giá sinh viên – ZALO:0973.287.149-TEAMLUANVAN.COM
(
(
j j
j j
√
Khi đó
x1 = 23
+ 3.5.2.12
= 38
y1 = 5.13
+ 3.22
.1 = 17
là nghi m thá hai của phương trình.
Ta có
x2 = 2.9.38 − 2 = 682
y2 = 2.9.17 − 1 = 305
là nghi m thá ba của phương trình.
V y (2, 1); (38, 17) và (682, 305) là 3 nghi m đau tiên của phương trình.
Định lj 1.2.23. Giả sủ d là so nguyên dương không chính phương,
pk
là giản
qk
phân của liên phân so của d với k = 0, 1, 2, 3, ... và n là chieu dài chu kì của
liên phân so của
√
d. Khi đó các phát bieu sau là đúng:
(i) Neu n chȁn thì phương trình Pell (1.19) vô nghi m.
(ii) Neu n lé thì tat cả các nghi m của phương trình (1.19) được cho bới công
thúc:
x = p(2k−1)n−1
y = q(2k−1)n−1
k ≥ 1.
Chúng minh. Theo Bő đe 1.1.16 ta có (x0, y0) là nghi m của phương trình (1.19).
Trong đó x0 = pj
với
pj
qj
là giản phân của liên phân so của
√
d. Áp dụng Bő đe
1.1.17 ta có
p2
− dq2
= (−1)j+1
tj+1 (j = 0, 1, 2, ...; tj+1 > 0)
Do p2
− dq2
= −1 nên j + 1 là so lẻ và tj+1 = 1. Vì n là chieu dài của liên phân
so của
√
d nên theo Bő đe 1.1.18 ta có
n| (j + 1) nên j + 1 = n.j
′
j
′
∈ Z , suy ra j = n.j
′
− 1 (j ≥ 0)
Với n là so lẻ. Do j + 1 so lẻ nên njJ
lẻ và vì n lẻ nên jJ
lẻ. V y jJ
= 2k − 1,
k ≥ 1. Tà đó ta có j = n(2k − 1) − 1. V y nghi m của phương trình (1.19) có
dạng
x = p(2k−1)n−1
y = q(2k−1)n−1
k ≥ 1,
(
(
36
Viết đề tài giá sinh viên – ZALO:0973.287.149-TEAMLUANVAN.COM
với n là so chȁn. Do j + 1 là so lẻ nên njJ
là so lẻ mà n chȁn nên vô lý. Do đó
không ton tại j’ thỏa mãn. V y phương trình (1.19) vô nghi m.
Ví dn 1.2.24. Tìm hai nghi m đau tiên của phương trình x2
− 13y2
= −1.
Với k = 1 thì x0 = p4 = 18 ; y0 = q4 = 5 là nghi m bé nhat của phương trình.
Với k = 2 thì x1 = p14 = 23382 ; y1 = q14 = 6485 là nghi m thá hai của phương
trình. V y (18, 5) và (23382, 6485) là hai nghi m của phương trình.
Định lý tiep theo cho ta m®t moi liên h cau trúc nghi m của hai phương
trình Pell liên ket.
Định lj 1.2.25. Neu (a, b) là nghi m của phương trình (1.3) và (u, v) là nghi m
của phương trình (1.19) thì (au+dbv, av+bu) là nghi m của phương trình (1.19).
Ví dn 1.2.26. Ta có (99, 70) là nghi m của phương trình x2
− 2y2
= 1 và (7, 5)
là nghi m của phương trình x2
− 2y2
= −1. Khi đó
(99.7 + 2.70.5, 99.5 + 70.7) = (1393, 985)
là nghi m của phương trình x2
− 2y2
= −1.
37
Viết đề tài giá sinh viên – ZALO:0973.287.149-TEAMLUANVAN.COM
Chương 2
Phương trình Diophantine dạng
x2 − Dy2 = ±4
Chương này chúng tôi t p trung trình bày lại các ket quả ve cau trúc nghi m
nguyên của phương trình x2
− Dy2
= ±4 và áng dụng của phương trình x2
−
Dy2
= ±1 và x2
− Dy2
= ±4 vào m®t so bài toán phő thông. Các ket quả của
chương này được viet theo tài li u [3].
2.1 Cau trúc nghi m của ho phương trình x2
− Dy2
= ±4
Xét phương trình Diophantine
u2
− Dv2
= ±4, (2.1)
trong đó D là so nguyên, không là so chính phương.
Giả sả phương trình√
(2.1) có nghi m và cho u, v là hai so nguyên thỏa
mãn (2.1). Khi đó
√
u + v
2
D
được goi là nghi m của phương trình (2.1). Neu
x + y
2
D
là nghi m của phương trình Diophantine
u2
− Dv2
= 4 (2.2)
thì so
u + v
√
D x + y
√
D u + v
√
D
=
1 1
2 2 2
cũng là nghi √
m của phương trình (2.1). Nghi m này được goi là liên ket với
nghi m
u + v
2
D
. T p tat cả các nghi m liên ket với nhau tạo thành m®t lớp
nghi m của phương trình (2.1).
38
Viết đề tài giá sinh viên – ZALO:0973.287.149-TEAMLUANVAN.COM
(
(
0
2
Đieu ki n can và đủ đe hai nghi m
u + v
√
D
2
uJ
+ vJ
√
D
,
2 thu®c cùng m®t lớp
nghi m là so
vuJ
− uJ
v
2
là so nguyên.
Định lj 2.1.1. Phương trình (2.1) ho¾c vô nghi m ho¾c có vô so nghi m.
Chúng minh. Ta đi cháng minh rang neu phương trình (2.1) có nghi m thì sě
có vô so nghi m. Giả sả (x, y) là m®t nghi m của phương trình (2.1). Goi (a, b)
là m®t nghi m của phương trình (1.3), tác là a2
− db2
= 1. Khi đó ta có
x2
− dy2
= ±4,
a2
− db2
= 1.
Nhân tàng ve hai đȁng thác trên, ta nh n được
x2
− dy2
a2
− db2
= ±4 ⇔ (xa + dyb)2
− d(xb + ya)2
= ±4. (2.3)
Đ t
xJ
= xa + dyb,
yJ
= xb + ya.
Khi đó thay vào (2.3), ta có xJ2
− dyJ2
= ±4. Tà đó ta có (xJ
, yJ
) là nghi m của
phương trình (2.1). Rõ ràng x < xJ
, y < yJ
. Do đó giả sả có nghi m ban đau
(x1, y1). Xét h thác
xn+1 = xna + dynb,
yn+1 = xnb + yna.
H thác này theo cháng minh trên cho ta lớp nghi m của phương trình (2.2).
Định lj 2.1.2. Giả sủ phương trình Pell (2.1) có nghi m và goi (x0, y0) là
nghi m nguyên dương nhó nhat của nó. Khi đó ta có
y2
≤ max ±4b ;
4a2
,
d
ớ đây (a, b) là nghi m dương nhó nhat của phương trình Pell (1.3).
Chúng minh. Đ t
u = x0a − dy0b,
v = y0a − x0b.
(
(
∓
39
Viết đề tài giá sinh viên – ZALO:0973.287.149-TEAMLUANVAN.COM
0
0
⇔ − ⇔
(
0
0 0 0 0
= x2
a2
− db2
− dy2
a2
− db2
0 0
0 0 0 0
0 0
⇔ x2
1 + db2
> db2
x2
∓ 4 ⇔ x2
> ∓4db2
⇔ ±4 + dy0 > −4 a — 1 ⇔ dy0 > ∓4a ⇔ y0 > .
d
2
0 0
0
khi đó:
u2
− dv2
= (x0a − dy0b)2
− d(y0a − x0b)2
= a2
x2
+ d2
b2
y2
− 2abdx0y0 − da2
y2
− db2
x2
+ 2abdx0y0
= a2
− db2
x2
− dy2
= ±4.
Giả sả ket lu n của bài toán không đúng, tác là
y2
> ±4b2
,
y2
> max ±4b2
;
4a2
.
d
Tà đó,
y2
>
4a2
.
d
Ta sě cháng minh rang u > 0, v > 0. Th t v y,
v > 0 y0a x0b > 0 y0a > x0b
⇔ a2
y2
> b2
x2
⇔ 1 + db2
y2
> b2
±4 + dy2
⇔ y2
> ±4b2
.
V y v > 0. Tương tự ta có
u > 0 ⇔ x0a − dy0b > 0 ⇔ x0a > dy0b ⇔ x2
a2
> d2
b2
y2
0 0 0
2 2 2 2 2 ∓4a2
Như v y, ta có u > 0 và v > 0 mà u2
− dv2
= ±4 nên (u, v) là nghi m của
(2.1). Tà h
với ȁn là (x0, y0), suy ra
u = x0a − dy0b
v = −x0b + y0a
x0 = au + dbv,
y0 = bu + av.
Tà đó ta có x0 > u, y0 > v. Như the ta đã xây dựng được nghi m (u, v) của
phương trình Pell (2.2), mà nghi m này còn nhỏ hơn (x0, y0). Đieu này là mâu
thuan vì (x0, y0) là nghi m bé nhat của (2.1). V y giả thuyet phản cháng sai,
nên
y2
≤ max ±4b ;
4a2
.
d
∓
0
(
∓
∓
40
Viết đề tài giá sinh viên – ZALO:0973.287.149-TEAMLUANVAN.COM
i
0
0 0
1. Neu v0 ≤ max ±4b ;
1 1
Định lj 2.1.3. Giả sủ (2.1) có nghi m và (α1, β1) ; (α2, β2) ; ... ; (αm, βm) là
tat cả các nghi m của (2.1) thóa mãn bat đȁng thúc:
β2
≤ max ±4b2
;
∓4a2
d
Xét m dãy {xn,i, yn,i} i = 1, m xác đ nh như sau:
x0,i = αi; y0,i = βi
xn+1,i = xn,ia + dyn,ib
yn+1,i = xn,ib + yn,ia,
ớ đây (a, b) là nghi m dương bé nhat của phương trình Pell (1.3) tương úng với
(2.1). Khi đó các dãy {xn,i, yn,i} i = 1, m sẽ vét cạn het nghi m của phương
trình (2.1).
Chúng minh. Theo cháng minh trong Định lý 2.1.1, các so hạng của các dãy
trên đeu là nghi m của phương trình Pell (2.1). Ta đi cháng minh đieu ngược
lại. Giả sả (u0, v0) là m®t nghi m dương bat kì của (2.1). Ta phải cháng minh
sự ton tại i, k sao cho u0 = xk,i ; v0 = yk,i. Chỉ có hai khả năng sau xảy ra
2 2 ∓4a2
sao cho (u0, v0) = (αi, βi) = (x0,i, y0,i). Ta chỉ chon k = 0.
2. Neu v2
> max ±4b2
;
∓4a2
d
. Đ t: u1 = au0 − dbv0 ; v1 = av0 − bu0.
Ta thay rang u2
− dv2
= ±4. Ngoài ra, do
v2
> ±4b2
và v2
>
4a2
,
d
nên l p lu n tương tự như cách cháng minh trong định lý 2.1.2 ta có u1 >
0; v1 > 0. Tà đó suy ra (u1, v1) là nghi m của phương trình (2.1). Tà h
u1 = au0 − dbv0,
v1 = av0 − bu0.
Suy ra
u0 = au1 + dbv1,
v0 = bu1 + av1.
d
∓
(
(
. Theo giả thuyet ton tại i ∈ {1, 2, ..., m}
41
Viết đề tài giá sinh viên – ZALO:0973.287.149-TEAMLUANVAN.COM
1
k
2
2
Vì v y suy ra
v1 < v0 (u0, v0, u1, v1, a, b, d > 0) .
Neu v2
> max ±4b2
;
∓4a2
d
thì bang l p lu n trên ta lại xây dựng được
nghi m (u2, v2) của (2.2) với u2 < u1 ; v2 < v1. Quá trình ay cá tiep tục và
phải ket thúc ở bước thá k mà sau khi có nghi m (uk, vk) của (2.2), thì
v2
≤ max 4b2
;
4a2
.
d
Khi đó, ton tại i ∈ {1, 2, ..., m} : (uk, vk) = (αi, βi) = (x0,i, y0,i). Do
uk−1 = auk + dbvk,
vk−1 = buk + avk.
Nên ta có
(u ,v ) = (x , y ) vì
(
uk−1 = auk + dbvk = ax0,i + dby0,i = x1,i
k−1 k−1 1,i 1,i vk−1 = buk + avk = bx0,i + ay0,i = y1,i
(u ,v ) = (x , y ) vì
(
uk−2 = auk−1 + dbvk−1 = ax1,i + dby1,i = x2,i
k−2 k−2
...
2,i 2,i
vk−2 = buk−1 + avk−1 = bx1,i + ay1,i = y2,i
(u0, v0) = (xk,i, yk,i) .
V y Định lý đã được cháng minh.
Ví dn 2.1.4. Giải phương trình:
x2
− 5y2
= −4.
Xét phương trình Pell liên ket với nó có dạng:
x2
− 5y2
= 1.
Phương trình liên ket có nghi m dương nhỏ nhat là (a, b) = (9, 4). Khi đó:
max −4.4 ;
4.92
5
4.81
= = 64, 8.
5
So nguyên dương β lớn nhat thỏa mãn
β ≤ max 4b2
;
−4a2
d
= 64, 8
là β = 8. Xét phương trình x2
− 5y2
= −4, ta có:
(
−
42
Viết đề tài giá sinh viên – ZALO:0973.287.149-TEAMLUANVAN.COM
i
1. Neu y = 1 thì x = 1,
2. Neu y = 2 thì x = 4,
3. Neu y = 3, 4, 7, 8 thì x không là so nguyên.
4. Neu y = 5 thì x = 11.
Như v y bang cách thả trực tiep như the, ta thay phương trình đã cho có ba
nghi m (1, 1); (4, 2) và (11, 5) mà thỏa đieu ki n:
β2
≤ max 4b2
;
4a2
.
d
Theo định lý, phương trình đã cho có 3 dãy nghi m sau
x0,1 = 1; y0,1 = 1; xn+1,1 = 9xn,1 + 20yn,1; yn+1,1 = 4xn,1 + 9yn,1,
x0,2 = 4; y0,2 = 2; xn+1,2 = 9xn,2 + 20yn,2; yn+1,2 = 4xn,2 + 9yn,2,
x0,3 = 11; y0,3 = 5; xn+1,3 = 9xn,3 + 20yn,3; yn+1,3 = 4xn,3 + 9yn,3.
Ba dãy này đã vét het nghi m của phương trình.
Định lý tiep theo của mục này cho ta m®t cách hǎu hi u đe tìm nghi m của
phương trình (2.1) thông qua m®t nghi m biet trước và nghi m của phương
trình Pell (1.3).
Định lj 2.1.5. Cho (u, v) là nghi m nguyên dương của phương trình (2.1) và
(a, b) là nghi m của phương trình Pell (1.3) liên ket với phương trình (2.1). Khi
đó (ua+dbv, ub+va) là nghi m của phương trình (2.1). Các nghi m của phương
trình (2.1) khác bi t nhau với sự khác bi t của (u, v) và (a, b).
2.2 Phương trình Diophantine dạng x2
− Dy2
= 4
Xét phương trình Diophantine (2.2)
u2
− Dv2
= 4,
trong đó D là các so nguyên và D không là so chính phương.
−
43
Viết đề tài giá sinh viên – ZALO:0973.287.149-TEAMLUANVAN.COM
!
— Dv
n n
2n−1 2n−1
!
!
Định lj 2.2.1. Cho (x1, y1) là nghi m cơ bản của phương trình (2.2) và cho
un
!
=
x1 Dy1
!n
1
!
(2.4)
vn y1 x1 0
với n ≥ 1. Khi đó nghi m nguyên của phương trình (2.2) là (xn, yn) được cho
bới công thúc
(x , y ) =
un
,
vn
. (2.5)
Chúng minh. Ta đi cháng minh định lý bang phương pháp quy nạp theo n. Với
n = 1, theo (2.4) ta có (u1, v1) = (x1, y1). Và vì (x1, y1) là nghi m cơ bản của
phương trình (2.2). Giả sả xn−1, yn−1 là nghi m của phương trình (2.2), tác là
u2
− Dv2
x2
n−1
+ y2
n−1
= n−1 n−1
22n−4
= 4.
Ta sě cháng minh nghi m của phương trình (2.2) là (xn, yn) được cho bởi công
thác (2.5). Th t v y, tà phương trình (2.4) ta có
un
!
=
x1
Dy1
!n
1
!
vn y1 x1 0
=
x1
Dy1
!
x1 Dy1
!n−1
1
!
y1 x1 y1 x1 0
=
x1 Dy1
y1 x1
un−1
vn−1
Hơn nǎa,
=
x1un−1 + Dy1vn−1
.
y1un−1 + x1vn−1
x2
− Dy2 u2
= n−1
2
n−1
n n
22n−2
(x1un−1 + Dy1vn−1)2
− D(y1nn−1 + x1vn−1)2
22n−2
x2u2 + 2x1un−1Dy1vn−1 + D2
y2
v2
= 1 n−1
22n−2
1 n−1
D(y2
u2
+ 2y1un−1x1vn−1 + x2
v2
)
— n−1
22n−2
1 n−1
x2
(u2
− Dv2
) − Dy2
(u2
− Dv2
)
= 1 n−1 n−1 1 n−1 n−1
22n−2
(x2
− Dy2
)(u2
− Dv2
)
= 1 1 n−1 n−1
.
22n−2
1
=
44
Viết đề tài giá sinh viên – ZALO:0973.287.149-TEAMLUANVAN.COM
−
−
Vì (x1, y1) là nghi m cơ bản của phương trình (2.2) và theo giả thiet quy nạp
ta có
2 2 4.2n−2
xn + Dyn =
2n−2
= 4.
Do đó, (xn, yn) là nghi m của phương trình (2.2). Vì n là tùy ý nên tat cả các
nghi m nguyên của phương trình (2.2) là (xn, yn).
H quả 2.2.2. Cho (x1, y1) là nghi m cơ bản của phương trình (2.2) thì
x =
x1xn−1 + Dy1yn−1
, y
n
2
n
và
=
y1xn−1 + x1yn−1
2
xn xn 1
. . = −2y1
với moi n ≥ 2.
. yn yn−1
.
Chúng minh. Ta có un = x1un−1 + Dy1vn−1, vn = y1un−1 + x1vn−1 và un−1 =
2n−2
xn−1,vn−1 = 2n−2
yn−1. V y ta có bien đői tương đương sau
un = x1un−1 + Dy1vn−1,
2n−1
xn = x12n−2
xn−1 + Dy12n−2
yn−1,
2n−1
xn = 2n−2
(x1xn−1 + Dy1yn−1),
x =
x1xn−1 + Dy1yn−1
,
n
2
và
vn = y1un−1 + x1vn−1,
2n−1
yn = y12n−2
xn−1 + x12n−2
yn−1,
2n−1
yn = 2n−2
(y1xn−1 + x1yn−1),
Hơn nǎa
y =
y1xn−1 + x1yn−1
.
n
2
xn xn 1
. . = xnyn−1 — xn−1yn
. yn yn−1
=
x1xn−1 + Dy1yn−1
y
2 n−1 — xn−1
y1xn−1 + x1yn−1
2
−y1(x2
− Dy2
)
= n−1 n−1
2
=
−4y1
2
= −2y1.
.
45
Viết đề tài giá sinh viên – ZALO:0973.287.149-TEAMLUANVAN.COM
— Dv
2n+1 2n+1
22n 22n
H quả 2.2.3. Cho (x1, y1) là nghi m cơ bản của phương trình (2.2). Khi đó,
(xn, yn) thóa mãn các bieu thúc sau
xn = (x1 − 1)(xn−1 + xn−2) − xn−3,
yn = (x1 − 1)(yn−1 + yn−2) − yn−3.
2.3 Phương trình Diophantine dạng x2
− Dy2
= −4
Xét phương trình Diophantine
u2
− Dv2
= −4, (2.6)
trong đó D là các so nguyên và D không là so chính phương.
Định lj 2.3.1. Cho (x1, y1) là nghi m cơ bản của phương trình (2.6) và cho
u2n+1
!
=
x1 Dy1
!2n+1
1
!
(2.7)
v2n+1 y1 x1 0
với n ≥ 0. Khi đó nghi m nguyên của phương trình (2.6) là (x2n+1, y2n+1) được
cho bới công thúc
(x , y ) =
u2n+1
,
v2n+1
. (2.8)
Chúng minh. Sả dụng phương pháp quy nạp theo n ta có. Với n = 0, theo (2.7)
ta có (u1, v1) = (x1, y1). Và vì (x1, y1) là nghi m cơ bản của phương trình (2.6).
Giả sả x2n−1, y2n−1 là nghi m của phương trình (2.6), tác là
x2
+ y2 u2
= 2n−1
2
2n−1 = −4.
2n−1 2n−1
24(n−1)
Chúng ta phải đi cháng minh nghi m của phương trình (2.6) là (x2n+1, y2n+1)
46
Viết đề tài giá sinh viên – ZALO:0973.287.149-TEAMLUANVAN.COM
1 1
1 1
1 1
được cho bởi công thác (2.8). Th t v y, tà phương trình (2.7) ta có
u2n+1
!
=
x1
Dy1
!2n+1
1
!
v2n+1 y1 x1 0
=
x1
Dy1
!2
x Dy1
!2n−1
1
!
y1 x1 y1 x1 0
=
x1
Dy1
!2
u2n−1
!
y1 x1 v2n−1
(x2
+ Dy2
)u2n−1 + Dx1y1v2n−1
!
2x1y1u2n−1 + (x2
+ Dy2
)v2n−1
Hơn nǎa,
u2
− Dv2
2
2n+1
2n+1 =
2n+1 2n+1
22n
((x2
+ Dy2
)u2n−1 + Dx1y1v2n−1)2
= 1 1
22n
D(2x1y1u2n−1 + (x2
+ Dy2
)v2n−1)2
22n
(x2
+ Dy2
)2
u2
+ 4x1u2n−1Dy1v2n−1 + 4D2
x2
y2
v2
= 1 1 2n−1
22n
1 1 2n−1
D(4x2
y2
u2
+ 4x1y1(x2
+ Dy2
)u2n−1v2n−1 + (x2
+ Dy2
)2
v2
)
— 2n−1 1 1
22n
1 1 n−1
(x2
+ Dy2
)2
(u2
— Dv2
) − 4Dx2
y2
(u2
— Dv2
)
= 1 1 2n−1 2n−1
22n
1 1 2n−1 2n−1
(x2
− Dy2
)2
(u2
− Dv2
)
= 1 1 n−1 n−1
.
22n
Vì (x1, y1) là nghi m cơ bản của phương trình (2.6) và theo giả thiet quy nạp
ta có
2 2 (−4)2
(−4).24n−4
xn−1 + Dyn−1 =
24n
= −4.
Do đó, (x2n+1, y2n+1) là nghi m của phương trình (2.6). Vì n là tùy ý nên tat
cả các nghi m nguyên của phương trình (2.6) là (x2n+1, y2n+1).
H quả 2.3.2. Cho (x1, y1) là nghi m cơ bản của phương trình (2.6). Khi đó
(x2
+ Dy2
)x2n−1 + Dx1y1y2n−1
x2n+1 = 1 1
,
4
2x1y1x2n−1 + (x2
+ Dy2
)y2n−1
y2n+1 = 1 1
4
1
1
2
x — Dy
= .
1
−
47
Viết đề tài giá sinh viên – ZALO:0973.287.149-TEAMLUANVAN.COM
−
.
−
1 1
1 1
1 1
1 1
1 1
1 1
1 1
1 1
1 1
và
x2n+1 x2n 1
. = 2x1y1
với moi n ≥ 1.
Chúng minh. Ta có
. y2n+1 y2n−1
.
u2n+1 = (x2
+ Dy2
)u2n−1 + Dx1y1v2n−1,
v2n+1 = 2x1y1u2n−1 + (x2
+ Dy2
)v2n−1
và u2n−1 = 22n−2
x2n−1,v2n−1 = 22n−2
y2n−1. V y chúng ta có bien đői tương
đương sau
u2n+1 = (x2
+ Dy2
)u2n−1 + Dx1y1v2n−1
22n
x2n+1 = (x2
+ Dy2
)22n−2
x2n−1 + Dx1y122n−2
y2n−1
22n
x2n+1 = 22n−2
((x2
+ Dy2
)x2n−1 + 2Dx1y1y2n−1)
(x2
+ Dy2
)x2n−1 + Dx1y1y2n−1
x2n+1 = 1 1
,
4
và
v2n+1 = 2x1y1u2n−1 + (x2
+ Dy2
)v2n−1
22n
y2n+1 = 2x1y122n−2
x2n−1 + (x2
+ Dy2
)22n−2
x2n−1
22n
y2n+1 = 22n−2
(2x1y1x2n−1 + (x2
+ Dy2
)y2n−1)
2x1y1x2n−1 + (x2
+ Dy2
)y2n−1
Và hơn nǎa
y2n+1 = 1 1
.
4
x2n+1 x2n 1
. . = x2n+1 y2n−1 — x2n−1 y2n+1
. y2n+1 y2n−1
.
(x2
+ Dy2
)x2n−1 + Dx1y1y2n−1
= 1 1
4
y2n−1
−x2n−1
2 x1y1x2n−1 + (x2
+ Dy2
)y2n−1
4
−2x1y1(x2
− Dy2
)
= 2n−1 2n−1
4
=
−2x1y1(−4)
= 2x
4
y1.
1
48
Viết đề tài giá sinh viên – ZALO:0973.287.149-TEAMLUANVAN.COM
1
1
−
≈
−
≈ ≈
408 408
−
H quả 2.3.3. Cho (x1, y1) là nghi m cơ bản của phương trình (2.6). Khi đó,
(x2n+1, y2n+1) thóa mãn các bieu thúc sau
x2n+1 = (x2
+ 1)(x2n−1 + x2n−3) − x2n−5
y2n+1 = (x2
+ 1)(y2n−1 + y2n−3) − y2n−5.
2.4 M t so fíng dnng trong toán pho thông
2.4.1 Tìm so nguyên tfi h thfíc ràng bu c
Ví dn 2.4.1. Xác định t ∈ Z thỏa mãn 10t + 12 = x2
và 5t + 4 = y2
Th t v y, ta có:
x2
− 12
t =
10
y2
4
t =
5
x2
12 =
10
y2
4 2
5
⇒ x
— 12 = 2y − 8
Suy ra x2
− 2y2
= 4. Giải phương trình Pell ta xác định được t.
2.4.2 Xap xỉ hfiu t của căn b c 2
Ta không the viet
√
d
x
y
với x, y ∈ Z vì
√
d là so vô tỉ. Tuy nhiên ta có
x2
− dy2
= ±N ⇒
x
2
N
= d +
y2
≈ d.
Vì the neu nghi m y của phương trình Pell sao cho y2
càng lớn thì xap xỉ hǎu
tỉ của
√
d càng tot. Ta xét m®t so ví dụ minh hoa.
Ví dn 2.4.2. Nghi m thá tư của phương trình x2
2y2
= 1 là (x, y) = (577, 408)
và
577
≈ 1.4142156. Trong khi đó,
√
2 = 1.4142135. V y
577
≈
√
d.
Ví dn 2.4.3. Cho phương trình Pell x2
− 3y2
= 1. Ta có (1351, 780) là nghi m
của phương trình trên. Do đó
√
3
1351
1.7320512.
780
y
2
−
⇒
49
Viết đề tài giá sinh viên – ZALO:0973.287.149-TEAMLUANVAN.COM
4
#
⇔ 2 m2
+ m + 1 = n2
2
2
2
2 4
2 2
2.4.3 Tong của nhfing so nguyên liên tiep nhau
Ví dn 2.4.4. Cho 1 + 2 = 3; 1 + 2 + ... + 14 = 15 + ... + 20. Tìm k, l thỏa mãn
1 + 2 + . . . + k = (k + 1) + . . . + l. Ta có
(k + 1) k
1 + 2 + . . . + k = (k + 1) + . . . + l ⇔
2
=
(k + 1 + l) (l − k)
2
⇔ 2k2
+ 2k = l2
+ l
⇔ 2
"
k +
1
−
1
#
=
1 1
2
l + −
2 4
⇔ 2
h
(2k + 1)2
− 1
i
= (2l + 1)2
− 1
⇔ (2l + 1) − 2(2k + 1) = −1.
Đ t x = 2l + 1 và y = 2k + 1 ta được phương trình Pell x2
− 2y2
= −1 với
x, y > 0 và x, y là so lẻ. Ta can giải phương trình trên và tà đó tìm ra k và l.
Khi đó ta có:
1 + 2 = 3
1 + 2 + . . . + 14 = 15 + 16 + ... + 20
1 + 2 + ... + 84 = 85 + 86 + ... + 119.
2.4.4 Tam giác Pythagoras
Ví dn 2.4.5. Cho 32
+ 42
= 52
. Ta can tìm m và n sao cho m2
+(m + 1)2
= n2
.
Th t v y:
m2
+ (m + 1)2
= n2
⇔ 2m2
+ 2m + 1 = n2
⇔ 2
"
m +
1
−
1
+ 1 = n2
⇔ (2m + 1)2
− 2n2
= −1.
Đ t x = 2m + 1 và y = n ta được phương trình Pell x2
− 2y2
= −1 với
x, y > 0 và x lẻ. Ta can giải phương trình trên và tà đó tìm ra m và n. Khi đó
ta có:
50
Viết đề tài giá sinh viên – ZALO:0973.287.149-TEAMLUANVAN.COM
2
−
x 1 7 41 239 1393 8119
y 1 5 29 169 985 5741
m 0 3 20 119 696 4059
n 0 5 29 169 985 5741
202
+ 212
= 292
,
1192
+ 1202
= 1692
.
Ví dn 2.4.6. Ta can tìm m và n sao cho m2
+ n2
= (n + 1)2
. Th t v y, ta
có phương trinh tương đương với m2
= 2n + 1. Cho m = 2k + 1 vì m lẻ nên
m2
1
n =
2
= 2k2
+ 2k. Không can dùng tới phương trình Pell mà ta van có
the giải ra dựa vào bảng sau:
k 1 2 3 4
m 3 5 7 9
n 4 12 24 40
n + 1 5 13 25 41
Tà bảng trên cho ta thay
32
+ 42
= 52
; 52
+ 122
= 132
; 72
+ 242
= 252
; 92
+ 402
= 412
.
2.4.5 Tam giác Heron
Tam giác Heron là tam giác có cạnh và di n tích nguyên.
Ví dn 2.4.7. Tam giác vuông có cạnh 3, 4, 5 và có di n tích bang 6. Công thác
tính di n tích của Hêron với tam giác bat kì có cạnh là a, b, c là
S =
√
p (p − a) (p − b) (p − c) với p =
a + b + c
.
3a
Tìm tam giác Hêron với các cạnh liên tiep a − 1; a; a + 1. Khi đó p =
2
và
S2
= p (p − a) (p − a + 1) (p − a − 1)
3a a
= . .a + 2
.
a − 2
.
2 2 2 2
51
Viết đề tài giá sinh viên – ZALO:0973.287.149-TEAMLUANVAN.COM
−
Suy ra (4S)2
= 3a2
a2
4 . Tà đó ta thay rang a là so chȁn. Đ t a = 2x với
x ∈ Z ta có S2
= 3x2
x2
− 1 . Vì 3, x2
= 1 nên x2
− 1 = 3y2
(y ∈ Z). Ta
có phương trình Pell
x2
− 3y2
= 1 (a = 2x; S = 3xy) .
Giải phương trình trên ta sě tìm được a và S. Ta có bảng sau:
x 2 7 26 97 362 1351
y 2 7 26 97 362 1351
a 4 14 52 154 724 2702
S 6 84 1170 16926 226974 3161340
Tà bảng ket quả trên ta thay:
Tam giác có các cạnh là 13, 14, 15 và có di n tích là 84.
Tam giác có các cạnh là 51, 52, 53 và có di n tích là 1170.
Tam giác có các cạnh là 153, 154, 155 và có di n tích là 16926.
52
Viết đề tài giá sinh viên – ZALO:0973.287.149-TEAMLUANVAN.COM
Ket lu n
Lu n văn phương trình Diophantine x2
− Dy2
= ±4 trình bày nhǎng van đe
chính sau đây:
• Trình bày lại m®t so tính chat của liên phân so và liên phân so mở r®ng tà
đó áng dụng nghiên cáu tính chat nghi m của phương trình Pell cő đien;
• Trình bày lại các ket quả ve công thác nghi m của phương trình Diophan-
tine dạng x2
− Dy2
= ±4 trong m®t so trường hợp giải được của phương
trình;
• Trình bày m®t so áng dụng của phương trình Diophantine trong toán hoc
phő thông.
53
Viết đề tài giá sinh viên – ZALO:0973.287.149-TEAMLUANVAN.COM
Tài li u tham khảo
Tieng Vi t
[1] Nguyen Thị My Hạnh (2017), Giải phương trình Diophante y2
= Ax4
+ B,
Lu n văn Thạc sĩ Toán hoc, Trường Đại hoc Khoa hoc, Đại hoc Thái
Nguyên.
[2] Đào Thị Thương Hoài (2010), M®t vài van đe ve phương trình Diophante,
Lu n văn Thạc sĩ Toán hoc, Trường Đại hoc Khoa hoc, Đại hoc Thái
Nguyên.
Tieng Anh
[3] Ahmet Tekcan (2007), "The Pell equation X2
− D ∗ Y 2
= ±4", Applied
Mathematical Sciences, 1(8), pp. 363–369.
[4] Ahmet Tekcan (2011), "Continued Fractions Expansion of
√
D and Pell
Equation x2
− D ∗ y2
= 1", Mathematica Moravica, 15(2), pp. 19—27.
[5] B. Stolt (1952), "On the Diophantine equation U2
−D∗V 2
= ±4N", Arkiv
för Matematik, 2(2-3), pp. 251–268.
[6] B. Stolt (1955), "On the Diophantine equation U2
− D ∗ V 2
= ±4N", part
III, Arkiv för Matematik, 3(2), pp. 117–132.

More Related Content

Similar to Phương trình diophantine dạng X2 − dy2 = ±4.docx

Tich phan %28 nguyen duy khoi%29
Tich phan %28 nguyen duy khoi%29Tich phan %28 nguyen duy khoi%29
Tich phan %28 nguyen duy khoi%29
trongphuckhtn
 
Tich phan (nguyen duy khoi)
Tich phan (nguyen duy khoi)Tich phan (nguyen duy khoi)
Tich phan (nguyen duy khoi)
roggerbob
 

Similar to Phương trình diophantine dạng X2 − dy2 = ±4.docx (20)

Bat Đang Thức V I Hàm Loi B Ph N Và Ứng Dụng.docx
Bat Đang Thức V I Hàm Loi B Ph N Và Ứng Dụng.docxBat Đang Thức V I Hàm Loi B Ph N Và Ứng Dụng.docx
Bat Đang Thức V I Hàm Loi B Ph N Và Ứng Dụng.docx
 
Một số phương pháp tìm cực trị của các hàm phân thức Sinh bởi số tự nhiên.docx
Một số phương pháp tìm cực trị của các hàm phân thức Sinh bởi số tự nhiên.docxMột số phương pháp tìm cực trị của các hàm phân thức Sinh bởi số tự nhiên.docx
Một số phương pháp tìm cực trị của các hàm phân thức Sinh bởi số tự nhiên.docx
 
Bài Toán Cực Trị Với Điều Kiện Ràng Buộc Bất Đẳng Thức, Hệ Bất Đẳng Thức.docx
Bài Toán Cực Trị Với Điều Kiện Ràng Buộc Bất Đẳng Thức, Hệ Bất Đẳng Thức.docxBài Toán Cực Trị Với Điều Kiện Ràng Buộc Bất Đẳng Thức, Hệ Bất Đẳng Thức.docx
Bài Toán Cực Trị Với Điều Kiện Ràng Buộc Bất Đẳng Thức, Hệ Bất Đẳng Thức.docx
 
Hàm Đơn Đi U, Tựa Đơn Đi U Và M T So Ứng Dụng Của Phép Đơn Đi U Hóa Hàm So.docx
Hàm Đơn Đi U, Tựa Đơn Đi U Và M T So Ứng Dụng Của Phép Đơn Đi U Hóa Hàm So.docxHàm Đơn Đi U, Tựa Đơn Đi U Và M T So Ứng Dụng Của Phép Đơn Đi U Hóa Hàm So.docx
Hàm Đơn Đi U, Tựa Đơn Đi U Và M T So Ứng Dụng Của Phép Đơn Đi U Hóa Hàm So.docx
 
Tich phan %28 nguyen duy khoi%29
Tich phan %28 nguyen duy khoi%29Tich phan %28 nguyen duy khoi%29
Tich phan %28 nguyen duy khoi%29
 
Tich phan (nguyen duy khoi)
Tich phan (nguyen duy khoi)Tich phan (nguyen duy khoi)
Tich phan (nguyen duy khoi)
 
Hàm Đơn Đi›U, Tựa Đơn Đi›U Và Một Số Ứng Dụng Của Phép Đơn Đi›U Hóa Hàm Số.docx
Hàm Đơn Đi›U, Tựa Đơn Đi›U Và Một Số Ứng Dụng Của Phép Đơn Đi›U Hóa Hàm Số.docxHàm Đơn Đi›U, Tựa Đơn Đi›U Và Một Số Ứng Dụng Của Phép Đơn Đi›U Hóa Hàm Số.docx
Hàm Đơn Đi›U, Tựa Đơn Đi›U Và Một Số Ứng Dụng Của Phép Đơn Đi›U Hóa Hàm Số.docx
 
Luận văn: Phương pháp giải phương trình chứa ẩn dưới dấu căn
Luận văn: Phương pháp giải phương trình chứa ẩn dưới dấu cănLuận văn: Phương pháp giải phương trình chứa ẩn dưới dấu căn
Luận văn: Phương pháp giải phương trình chứa ẩn dưới dấu căn
 
Tổng quát về tích phân
Tổng quát về tích phân Tổng quát về tích phân
Tổng quát về tích phân
 
Luận văn: Các bài toán về hệ thức lượng trong tam giác, HOT, 9đ
Luận văn: Các bài toán về hệ thức lượng trong tam giác, HOT, 9đLuận văn: Các bài toán về hệ thức lượng trong tam giác, HOT, 9đ
Luận văn: Các bài toán về hệ thức lượng trong tam giác, HOT, 9đ
 
Luận văn: Phép biến đổi phân tuyến tính, HAY, 9đ
Luận văn: Phép biến đổi phân tuyến tính, HAY, 9đLuận văn: Phép biến đổi phân tuyến tính, HAY, 9đ
Luận văn: Phép biến đổi phân tuyến tính, HAY, 9đ
 
M T So Dạng Toán Ve Dãy So Sinh B I Các Hàm So Sơ Cap.docx
M T So Dạng Toán Ve Dãy So Sinh B I Các Hàm So Sơ Cap.docxM T So Dạng Toán Ve Dãy So Sinh B I Các Hàm So Sơ Cap.docx
M T So Dạng Toán Ve Dãy So Sinh B I Các Hàm So Sơ Cap.docx
 
M t so dạng toán Liên quan đen xác suat r i rạc và ứng dụng.docx
M t so dạng toán Liên quan đen xác suat r i rạc  và ứng dụng.docxM t so dạng toán Liên quan đen xác suat r i rạc  và ứng dụng.docx
M t so dạng toán Liên quan đen xác suat r i rạc và ứng dụng.docx
 
M T So Dạng Toán Liên Quan Đen Xác Suat R I Rạc Và Ứng Dụng.docx
M T So Dạng Toán Liên Quan Đen Xác Suat R I Rạc Và Ứng Dụng.docxM T So Dạng Toán Liên Quan Đen Xác Suat R I Rạc Và Ứng Dụng.docx
M T So Dạng Toán Liên Quan Đen Xác Suat R I Rạc Và Ứng Dụng.docx
 
Tích Ngoài Của Véc Tơ Và Ứng Dụng.docx
Tích Ngoài Của Véc Tơ Và Ứng Dụng.docxTích Ngoài Của Véc Tơ Và Ứng Dụng.docx
Tích Ngoài Của Véc Tơ Và Ứng Dụng.docx
 
M T So Dạng Toán Cực Tr± Trong L P Hàm Mũ Và Hàm Hyperbolic.docx
M T So Dạng Toán Cực Tr± Trong L P Hàm Mũ Và Hàm Hyperbolic.docxM T So Dạng Toán Cực Tr± Trong L P Hàm Mũ Và Hàm Hyperbolic.docx
M T So Dạng Toán Cực Tr± Trong L P Hàm Mũ Và Hàm Hyperbolic.docx
 
Luận văn: Bài toán trên không chính quy cho hệ phương trình vi phân hàm bậc cao
Luận văn: Bài toán trên không chính quy cho hệ phương trình vi phân hàm bậc caoLuận văn: Bài toán trên không chính quy cho hệ phương trình vi phân hàm bậc cao
Luận văn: Bài toán trên không chính quy cho hệ phương trình vi phân hàm bậc cao
 
V N Dụng Chuői Đieu Hòa Vào Giải M T So Bài Toán Dành Cho Hoc Sinh Giỏi.docx
V N Dụng Chuői Đieu Hòa Vào Giải M T So Bài Toán Dành Cho Hoc Sinh Giỏi.docxV N Dụng Chuői Đieu Hòa Vào Giải M T So Bài Toán Dành Cho Hoc Sinh Giỏi.docx
V N Dụng Chuői Đieu Hòa Vào Giải M T So Bài Toán Dành Cho Hoc Sinh Giỏi.docx
 
Một Số Dạng Toán Cực Trị Trong Lîp Hàm Mũ Và Hàm Hyperbolic.docx
Một Số Dạng Toán Cực Trị Trong Lîp Hàm Mũ Và Hàm Hyperbolic.docxMột Số Dạng Toán Cực Trị Trong Lîp Hàm Mũ Và Hàm Hyperbolic.docx
Một Số Dạng Toán Cực Trị Trong Lîp Hàm Mũ Và Hàm Hyperbolic.docx
 
Bat Phương Trình Hàm Sinh B I Các Đại Lư Ng Trung Bình B C Tùy Ý Và Các Dạng ...
Bat Phương Trình Hàm Sinh B I Các Đại Lư Ng Trung Bình B C Tùy Ý Và Các Dạng ...Bat Phương Trình Hàm Sinh B I Các Đại Lư Ng Trung Bình B C Tùy Ý Và Các Dạng ...
Bat Phương Trình Hàm Sinh B I Các Đại Lư Ng Trung Bình B C Tùy Ý Và Các Dạng ...
 

More from DV Viết Luận văn luanvanmaster.com ZALO 0973287149

More from DV Viết Luận văn luanvanmaster.com ZALO 0973287149 (20)

Ảnh Hưởng Của Marketing Quan Hệ Đến Lòng Trung Thành Của Khách Hàng.Tình Huốn...
Ảnh Hưởng Của Marketing Quan Hệ Đến Lòng Trung Thành Của Khách Hàng.Tình Huốn...Ảnh Hưởng Của Marketing Quan Hệ Đến Lòng Trung Thành Của Khách Hàng.Tình Huốn...
Ảnh Hưởng Của Marketing Quan Hệ Đến Lòng Trung Thành Của Khách Hàng.Tình Huốn...
 
Phát triển nguồn nhân lực tại Uỷ ban nhân dân huyện Trà Bồng, tỉnh Quảng Ngãi...
Phát triển nguồn nhân lực tại Uỷ ban nhân dân huyện Trà Bồng, tỉnh Quảng Ngãi...Phát triển nguồn nhân lực tại Uỷ ban nhân dân huyện Trà Bồng, tỉnh Quảng Ngãi...
Phát triển nguồn nhân lực tại Uỷ ban nhân dân huyện Trà Bồng, tỉnh Quảng Ngãi...
 
Báo cáo tốt Nghiệp tài chính hợp nhất tại tổng công ty Indochina gol...
Báo cáo tốt Nghiệp  tài chính hợp nhất tại tổng công ty Indochina gol...Báo cáo tốt Nghiệp  tài chính hợp nhất tại tổng công ty Indochina gol...
Báo cáo tốt Nghiệp tài chính hợp nhất tại tổng công ty Indochina gol...
 
Tạo động lực thúc đẩy nhân viên làm việc tại ngân hàng TMCP Ngoại Thương Việt...
Tạo động lực thúc đẩy nhân viên làm việc tại ngân hàng TMCP Ngoại Thương Việt...Tạo động lực thúc đẩy nhân viên làm việc tại ngân hàng TMCP Ngoại Thương Việt...
Tạo động lực thúc đẩy nhân viên làm việc tại ngân hàng TMCP Ngoại Thương Việt...
 
Phát triển công nghiệp trên địa bàn Thành phố Tam Kỳ, Tỉnh Quảng Na...
Phát triển công nghiệp trên địa bàn Thành phố Tam Kỳ, Tỉnh Quảng Na...Phát triển công nghiệp trên địa bàn Thành phố Tam Kỳ, Tỉnh Quảng Na...
Phát triển công nghiệp trên địa bàn Thành phố Tam Kỳ, Tỉnh Quảng Na...
 
Giải pháp phát triển cho vay xuất nhập khẩu tại ngân hàng NN&PTNN ch...
Giải pháp phát triển cho vay xuất nhập khẩu tại ngân hàng NN&PTNN ch...Giải pháp phát triển cho vay xuất nhập khẩu tại ngân hàng NN&PTNN ch...
Giải pháp phát triển cho vay xuất nhập khẩu tại ngân hàng NN&PTNN ch...
 
Hoàn thiện công tác lập báo cáo tài chính hợp nhất tại tổng công ...
Hoàn thiện công tác lập báo cáo tài chính hợp nhất tại tổng công ...Hoàn thiện công tác lập báo cáo tài chính hợp nhất tại tổng công ...
Hoàn thiện công tác lập báo cáo tài chính hợp nhất tại tổng công ...
 
Luận Văn Thạc Sĩ Quản trị thành tích nhân viên tại Cục Hải quan TP Đà Nẵng.doc
Luận Văn Thạc Sĩ  Quản trị thành tích nhân viên tại Cục Hải quan TP Đà Nẵng.docLuận Văn Thạc Sĩ  Quản trị thành tích nhân viên tại Cục Hải quan TP Đà Nẵng.doc
Luận Văn Thạc Sĩ Quản trị thành tích nhân viên tại Cục Hải quan TP Đà Nẵng.doc
 
Hoàn thiện công tác quản lý thuế thu nhập cá nhân tại cục thuế Tỉ...
Hoàn thiện công tác quản lý thuế thu nhập cá nhân tại cục thuế Tỉ...Hoàn thiện công tác quản lý thuế thu nhập cá nhân tại cục thuế Tỉ...
Hoàn thiện công tác quản lý thuế thu nhập cá nhân tại cục thuế Tỉ...
 
Đề Tài Phát triển bền vững nông nghiệp Huyện Ba Tơ, Tỉnh Quảng Ngãi....
Đề Tài Phát triển bền vững nông nghiệp Huyện Ba Tơ, Tỉnh Quảng Ngãi....Đề Tài Phát triển bền vững nông nghiệp Huyện Ba Tơ, Tỉnh Quảng Ngãi....
Đề Tài Phát triển bền vững nông nghiệp Huyện Ba Tơ, Tỉnh Quảng Ngãi....
 
Hoàn thiện công tác bảo trợ xã hội trên địa bàn huyện Phong Điền, tỉnh Thừa T...
Hoàn thiện công tác bảo trợ xã hội trên địa bàn huyện Phong Điền, tỉnh Thừa T...Hoàn thiện công tác bảo trợ xã hội trên địa bàn huyện Phong Điền, tỉnh Thừa T...
Hoàn thiện công tác bảo trợ xã hội trên địa bàn huyện Phong Điền, tỉnh Thừa T...
 
Đề Tài Luận VănPhát triển sản phẩm du lịch tại thành phố Đà Nẵng.doc
Đề Tài Luận VănPhát triển sản phẩm du lịch tại thành phố Đà Nẵng.docĐề Tài Luận VănPhát triển sản phẩm du lịch tại thành phố Đà Nẵng.doc
Đề Tài Luận VănPhát triển sản phẩm du lịch tại thành phố Đà Nẵng.doc
 
Đào tạo nghề cho lao động thuộc diện thu hồi đất trên địa bàn Thàn...
Đào tạo nghề cho lao động thuộc diện thu hồi đất trên địa bàn Thàn...Đào tạo nghề cho lao động thuộc diện thu hồi đất trên địa bàn Thàn...
Đào tạo nghề cho lao động thuộc diện thu hồi đất trên địa bàn Thàn...
 
Tóm Tắt Luận Văn Thạc Sĩ Quản Trị Kinh Doanh Xây dựng chính sách Marketing tạ...
Tóm Tắt Luận Văn Thạc Sĩ Quản Trị Kinh Doanh Xây dựng chính sách Marketing tạ...Tóm Tắt Luận Văn Thạc Sĩ Quản Trị Kinh Doanh Xây dựng chính sách Marketing tạ...
Tóm Tắt Luận Văn Thạc Sĩ Quản Trị Kinh Doanh Xây dựng chính sách Marketing tạ...
 
Đề Tài Nghiên cứu rủi ro cảm nhận đối với mua hàng thời trang trực tuyến.docx
Đề Tài Nghiên cứu rủi ro cảm nhận đối với mua hàng thời trang trực tuyến.docxĐề Tài Nghiên cứu rủi ro cảm nhận đối với mua hàng thời trang trực tuyến.docx
Đề Tài Nghiên cứu rủi ro cảm nhận đối với mua hàng thời trang trực tuyến.docx
 
Giải pháp nâng cao động lực thúc đẩy người lao động tại công ty khai...
Giải pháp nâng cao động lực thúc đẩy người lao động tại công ty khai...Giải pháp nâng cao động lực thúc đẩy người lao động tại công ty khai...
Giải pháp nâng cao động lực thúc đẩy người lao động tại công ty khai...
 
Giải pháp phát triển dịch vụ ngân hàng điện tử tại ngân hàng đầu ...
Giải pháp phát triển dịch vụ ngân hàng điện tử tại ngân hàng đầu ...Giải pháp phát triển dịch vụ ngân hàng điện tử tại ngân hàng đầu ...
Giải pháp phát triển dịch vụ ngân hàng điện tử tại ngân hàng đầu ...
 
Giải pháp phát triển dịch vụ ngân hàng điện tử tại ngân hàng đầu ...
Giải pháp phát triển dịch vụ ngân hàng điện tử tại ngân hàng đầu ...Giải pháp phát triển dịch vụ ngân hàng điện tử tại ngân hàng đầu ...
Giải pháp phát triển dịch vụ ngân hàng điện tử tại ngân hàng đầu ...
 
Quản trị quan hệ khách hàng tại Chi nhánh Viettel Đà Nẵng – Tập đoàn Viễn thô...
Quản trị quan hệ khách hàng tại Chi nhánh Viettel Đà Nẵng – Tập đoàn Viễn thô...Quản trị quan hệ khách hàng tại Chi nhánh Viettel Đà Nẵng – Tập đoàn Viễn thô...
Quản trị quan hệ khách hàng tại Chi nhánh Viettel Đà Nẵng – Tập đoàn Viễn thô...
 
Đề Tài Đánh giá thành tích đội ngũ giảng viên trường Đại Học Phạm ...
Đề Tài Đánh giá thành tích đội ngũ giảng viên trường Đại Học Phạm ...Đề Tài Đánh giá thành tích đội ngũ giảng viên trường Đại Học Phạm ...
Đề Tài Đánh giá thành tích đội ngũ giảng viên trường Đại Học Phạm ...
 

Recently uploaded

VẤN ĐỀ 12 VI PHẠM HÀNH CHÍNH VÀ.pptx
VẤN ĐỀ 12 VI PHẠM HÀNH CHÍNH VÀ.pptxVẤN ĐỀ 12 VI PHẠM HÀNH CHÍNH VÀ.pptx
VẤN ĐỀ 12 VI PHẠM HÀNH CHÍNH VÀ.pptx
Gingvin36HC
 
Everybody Up 1 - Unit 5 - worksheet grade 1
Everybody Up 1 - Unit 5 - worksheet grade 1Everybody Up 1 - Unit 5 - worksheet grade 1
Everybody Up 1 - Unit 5 - worksheet grade 1
mskellyworkmail
 
Logic học và phương pháp nghiên cứu khoa học
Logic học và phương pháp nghiên cứu khoa họcLogic học và phương pháp nghiên cứu khoa học
Logic học và phương pháp nghiên cứu khoa học
K61PHMTHQUNHCHI
 

Recently uploaded (20)

40 ĐỀ LUYỆN THI ĐÁNH GIÁ NĂNG LỰC ĐẠI HỌC QUỐC GIA HÀ NỘI NĂM 2024 (ĐỀ 31-39)...
40 ĐỀ LUYỆN THI ĐÁNH GIÁ NĂNG LỰC ĐẠI HỌC QUỐC GIA HÀ NỘI NĂM 2024 (ĐỀ 31-39)...40 ĐỀ LUYỆN THI ĐÁNH GIÁ NĂNG LỰC ĐẠI HỌC QUỐC GIA HÀ NỘI NĂM 2024 (ĐỀ 31-39)...
40 ĐỀ LUYỆN THI ĐÁNH GIÁ NĂNG LỰC ĐẠI HỌC QUỐC GIA HÀ NỘI NĂM 2024 (ĐỀ 31-39)...
 
NGÂN HÀNG KĨ THUẬT SỐ-slide CHƯƠNG 1 B 1 2024.pptx
NGÂN HÀNG KĨ THUẬT SỐ-slide CHƯƠNG 1 B 1 2024.pptxNGÂN HÀNG KĨ THUẬT SỐ-slide CHƯƠNG 1 B 1 2024.pptx
NGÂN HÀNG KĨ THUẬT SỐ-slide CHƯƠNG 1 B 1 2024.pptx
 
VẤN ĐỀ 12 VI PHẠM HÀNH CHÍNH VÀ.pptx
VẤN ĐỀ 12 VI PHẠM HÀNH CHÍNH VÀ.pptxVẤN ĐỀ 12 VI PHẠM HÀNH CHÍNH VÀ.pptx
VẤN ĐỀ 12 VI PHẠM HÀNH CHÍNH VÀ.pptx
 
Báo cáo bài tập Quản trị Marketing Kế hoạch marketing cho ống hút cỏ của Gree...
Báo cáo bài tập Quản trị Marketing Kế hoạch marketing cho ống hút cỏ của Gree...Báo cáo bài tập Quản trị Marketing Kế hoạch marketing cho ống hút cỏ của Gree...
Báo cáo bài tập Quản trị Marketing Kế hoạch marketing cho ống hút cỏ của Gree...
 
Báo cáo tốt nghiệp Đánh giá rủi ro môi trường từ ô nhiễm hữu cơ nước thải các...
Báo cáo tốt nghiệp Đánh giá rủi ro môi trường từ ô nhiễm hữu cơ nước thải các...Báo cáo tốt nghiệp Đánh giá rủi ro môi trường từ ô nhiễm hữu cơ nước thải các...
Báo cáo tốt nghiệp Đánh giá rủi ro môi trường từ ô nhiễm hữu cơ nước thải các...
 
Báo cáo tốt nghiệp Đánh giá rủi ro môi trường ô nhiễm hữu cơ trong nước thải ...
Báo cáo tốt nghiệp Đánh giá rủi ro môi trường ô nhiễm hữu cơ trong nước thải ...Báo cáo tốt nghiệp Đánh giá rủi ro môi trường ô nhiễm hữu cơ trong nước thải ...
Báo cáo tốt nghiệp Đánh giá rủi ro môi trường ô nhiễm hữu cơ trong nước thải ...
 
Tiểu luận tổng quan về Mối quan hệ giữa chu kỳ kinh tế và đầu tư trong nền ki...
Tiểu luận tổng quan về Mối quan hệ giữa chu kỳ kinh tế và đầu tư trong nền ki...Tiểu luận tổng quan về Mối quan hệ giữa chu kỳ kinh tế và đầu tư trong nền ki...
Tiểu luận tổng quan về Mối quan hệ giữa chu kỳ kinh tế và đầu tư trong nền ki...
 
TỔNG HỢP HƠN 100 ĐỀ THI THỬ TỐT NGHIỆP THPT TOÁN 2024 - TỪ CÁC TRƯỜNG, TRƯỜNG...
TỔNG HỢP HƠN 100 ĐỀ THI THỬ TỐT NGHIỆP THPT TOÁN 2024 - TỪ CÁC TRƯỜNG, TRƯỜNG...TỔNG HỢP HƠN 100 ĐỀ THI THỬ TỐT NGHIỆP THPT TOÁN 2024 - TỪ CÁC TRƯỜNG, TRƯỜNG...
TỔNG HỢP HƠN 100 ĐỀ THI THỬ TỐT NGHIỆP THPT TOÁN 2024 - TỪ CÁC TRƯỜNG, TRƯỜNG...
 
Everybody Up 1 - Unit 5 - worksheet grade 1
Everybody Up 1 - Unit 5 - worksheet grade 1Everybody Up 1 - Unit 5 - worksheet grade 1
Everybody Up 1 - Unit 5 - worksheet grade 1
 
Báo cáo thực tập tốt nghiệp Phân tích thực trạng hoạt động bán hàng tại Công ...
Báo cáo thực tập tốt nghiệp Phân tích thực trạng hoạt động bán hàng tại Công ...Báo cáo thực tập tốt nghiệp Phân tích thực trạng hoạt động bán hàng tại Công ...
Báo cáo thực tập tốt nghiệp Phân tích thực trạng hoạt động bán hàng tại Công ...
 
Bài tập lớn môn Văn hóa kinh doanh và tinh thần khởi nghiệp Trình bày về triế...
Bài tập lớn môn Văn hóa kinh doanh và tinh thần khởi nghiệp Trình bày về triế...Bài tập lớn môn Văn hóa kinh doanh và tinh thần khởi nghiệp Trình bày về triế...
Bài tập lớn môn Văn hóa kinh doanh và tinh thần khởi nghiệp Trình bày về triế...
 
Luận Văn: HOÀNG TỬ BÉ TỪ GÓC NHÌN CẢI BIÊN HỌC
Luận Văn: HOÀNG TỬ BÉ TỪ GÓC NHÌN CẢI BIÊN HỌCLuận Văn: HOÀNG TỬ BÉ TỪ GÓC NHÌN CẢI BIÊN HỌC
Luận Văn: HOÀNG TỬ BÉ TỪ GÓC NHÌN CẢI BIÊN HỌC
 
Báo cáo tốt nghiệp Đánh giá công tác đào tạo và phát triển nguồn nhân lực tại...
Báo cáo tốt nghiệp Đánh giá công tác đào tạo và phát triển nguồn nhân lực tại...Báo cáo tốt nghiệp Đánh giá công tác đào tạo và phát triển nguồn nhân lực tại...
Báo cáo tốt nghiệp Đánh giá công tác đào tạo và phát triển nguồn nhân lực tại...
 
TỔNG HỢP HƠN 100 ĐỀ THI THỬ TỐT NGHIỆP THPT VẬT LÝ 2024 - TỪ CÁC TRƯỜNG, TRƯ...
TỔNG HỢP HƠN 100 ĐỀ THI THỬ TỐT NGHIỆP THPT VẬT LÝ 2024 - TỪ CÁC TRƯỜNG, TRƯ...TỔNG HỢP HƠN 100 ĐỀ THI THỬ TỐT NGHIỆP THPT VẬT LÝ 2024 - TỪ CÁC TRƯỜNG, TRƯ...
TỔNG HỢP HƠN 100 ĐỀ THI THỬ TỐT NGHIỆP THPT VẬT LÝ 2024 - TỪ CÁC TRƯỜNG, TRƯ...
 
Báo cáo tốt nghiệp Đánh giá rủi ro quá trình xử lí nước cấp tại Chi nhánh Cấp...
Báo cáo tốt nghiệp Đánh giá rủi ro quá trình xử lí nước cấp tại Chi nhánh Cấp...Báo cáo tốt nghiệp Đánh giá rủi ro quá trình xử lí nước cấp tại Chi nhánh Cấp...
Báo cáo tốt nghiệp Đánh giá rủi ro quá trình xử lí nước cấp tại Chi nhánh Cấp...
 
30 ĐỀ PHÁT TRIỂN THEO CẤU TRÚC ĐỀ MINH HỌA BGD NGÀY 22-3-2024 KỲ THI TỐT NGHI...
30 ĐỀ PHÁT TRIỂN THEO CẤU TRÚC ĐỀ MINH HỌA BGD NGÀY 22-3-2024 KỲ THI TỐT NGHI...30 ĐỀ PHÁT TRIỂN THEO CẤU TRÚC ĐỀ MINH HỌA BGD NGÀY 22-3-2024 KỲ THI TỐT NGHI...
30 ĐỀ PHÁT TRIỂN THEO CẤU TRÚC ĐỀ MINH HỌA BGD NGÀY 22-3-2024 KỲ THI TỐT NGHI...
 
Thực trạng ứng dụng công nghệ trong lĩnh vực giống cây trồng: Nghiên cứu điển...
Thực trạng ứng dụng công nghệ trong lĩnh vực giống cây trồng: Nghiên cứu điển...Thực trạng ứng dụng công nghệ trong lĩnh vực giống cây trồng: Nghiên cứu điển...
Thực trạng ứng dụng công nghệ trong lĩnh vực giống cây trồng: Nghiên cứu điển...
 
TỔNG HỢP HƠN 100 ĐỀ THI THỬ TỐT NGHIỆP THPT TIẾNG ANH 2024 - TỪ CÁC TRƯỜNG, ...
TỔNG HỢP HƠN 100 ĐỀ THI THỬ TỐT NGHIỆP THPT TIẾNG ANH 2024 - TỪ CÁC TRƯỜNG, ...TỔNG HỢP HƠN 100 ĐỀ THI THỬ TỐT NGHIỆP THPT TIẾNG ANH 2024 - TỪ CÁC TRƯỜNG, ...
TỔNG HỢP HƠN 100 ĐỀ THI THỬ TỐT NGHIỆP THPT TIẾNG ANH 2024 - TỪ CÁC TRƯỜNG, ...
 
Báo cáo tốt nghiệp Hoàn thiện an toàn lao động điện công ty trách nhiệm hữu h...
Báo cáo tốt nghiệp Hoàn thiện an toàn lao động điện công ty trách nhiệm hữu h...Báo cáo tốt nghiệp Hoàn thiện an toàn lao động điện công ty trách nhiệm hữu h...
Báo cáo tốt nghiệp Hoàn thiện an toàn lao động điện công ty trách nhiệm hữu h...
 
Logic học và phương pháp nghiên cứu khoa học
Logic học và phương pháp nghiên cứu khoa họcLogic học và phương pháp nghiên cứu khoa học
Logic học và phương pháp nghiên cứu khoa học
 

Phương trình diophantine dạng X2 − dy2 = ±4.docx

  • 1. ĐẠI HỌC THÁI NGUYÊN TRƢỜNG ĐẠI HỌC KHOA HỌC --------------------------- Tải tài liệu tại sividoc.com Viết đề tài giá sinh viên – ZALO:0973.287.149-TEAMLUANVAN.COM VŨ PHÚ BÌNH PHƢƠNG TRÌNH DIOPHANTINE DẠNG x2 − Dy2 = ±4 LUẬN VĂN THẠC SĨ TOÁN HỌC THÁI NGUYÊN - 2018
  • 2. ĐẠI HỌC THÁI NGUYÊN TRƢỜNG ĐẠI HỌC KHOA HỌC --------------------------- Tải tài liệu tại sividoc.com Viết đề tài giá sinh viên – ZALO:0973.287.149-TEAMLUANVAN.COM VŨ PHÚ BÌNH PHƢƠNG TRÌNH DIOPHANTINE DẠNG x2 − Dy2 = ±4 Chuyên ngành: Phƣơng pháp Toán sơ cấp Mã số: 8460113 LUẬN VĂN THẠC SĨ TOÁN HỌC NGƯỜI HƯỚNG DẪN KHOA HỌC PGS.TS. Nông Quốc Chinh THÁI NGUYÊN - 2018
  • 3. i Viết đề tài giá sinh viên – ZALO:0973.287.149-TEAMLUANVAN.COM Mnc lnc L i nói đau 1 Chương 1 Phương trình Diophantine x2 − Dy2 = ±1 2 1.1 Liên phân so và giản phân . . . . . . . . . . . . . . . . . . . . . 2 1.1.1 Liên phân so hǎu hạn và giản phân . . . . . . . . . . . . 2 1.1.2 Liên phân so vô hạn . . . . . . . . . . . . . . . . . . . . 6 1.2 Phương trình Diophantine x2 − Dy2 = ±1..........................................13 1.2.1 Phương trình Pell dạng x2 − dy2 = 1.......................................14 1.2.2 Úng dụng liên phân so √ D vào phương trình Pell x2 − Dy2 = 1 ............................................................................ 21 1.2.3 Phương trình Pell dạng x2 − dy2 = −1 ...................................27 Chương 2 Phương trình Diophantine dạng x2 − Dy2 = ±4 37 2.1 Cau trúc nghi m của ho phương trình x2 − Dy2 = ±4.......................37 2.2 Phương trình Diophantine dạng x2 − Dy2 = 4....................................42 2.3 Phương trình Diophantine dạng x2 − Dy2 = −4.................................45 2.4 M®t so áng dụng trong toán phő thông.................................................48 2.4.1 Tìm so nguyên tà h thác ràng bu®c.......................................48 2.4.2 Xap xỉ hǎu t của căn b c 2................................................. 48 2.4.3 Tőng của nhǎng so nguyên liên tiep nhau.................................49 2.4.4 Tam giác Pythagoras ................................................................49 2.4.5 Tam giác Heron ........................................................................50 Ket lu n 52 Tài li u tham khảo 53
  • 4. ii Viết đề tài giá sinh viên – ZALO:0973.287.149-TEAMLUANVAN.COM L i nói đau Xét phương trình có dạng f(x1, x2, ..., xn) = 0 (1) với n ≥ 2 và f (x1, x2, ..., xn) là m®t đa thác nguyên m®t ho c nhieu bien được goi là phương trình nghi m nguyên hay phương trình Diophantine, nó được goi theo tên nhà toán hoc Hy Lạp ở the kỉ thá 3 sau công nguyên. Phương trình Diophantine là m®t trong nhǎng dạng toán lâu đời nhat của Toán hoc và nh n được nhieu sự quan tâm nghiên cáu của nhieu nhà toán hoc. Tà Euclid, Dio- phantus, qua Fibonacci, Pell roi đen Fermat, Euler, Lebesgue... và thời hi n đại là Gelfold, Matiasevic, Shenzel, Serpinsky... Phương trình Diophantine đã trải qua m®t lịch sả phát trien lâu dài. Thông qua vi c giải các phương trình Diophantine, các nhà toán hoc đã tìm ra được nhǎng tính chat thú vị của so nguyên, so hǎu t , so đại so. Giải phương trình Diophantine đã đưa đen sự ra đời của Liên phân so, Lý thuyet đường cong elliptic, Lý thuyet xap xỉ Diophantine, Th ng dư bình phương, So hoc modular... Các bài toán ve phương trình Diophantine không có quy tac giải tőng quát, ho c neu có cũng chỉ là đoi với các dạng đơn giản. Moi phương trình với dạng riêng của nó đòi hỏi m®t cách giải đ c trưng phù hợp. Chính vì v y, phương trình Diophantine van thường xuyên xuat hi n dưới các hình thác khác nhau và luôn được đánh giá là khó do tính không mau mực của nó. M®t dạng đ c bi t của phương trình Diophante là x2 − Dy2 = N rat được quan tâm và có rat nhieu ket quả xung quanh dạng phương trình này. Gan đây m®t ket quả thú vị của A. Tekcan ve phương trình x2 − Dy2 = ±1 và x2 − Dy2 = ±4 đã được công bo. Mục đích của lu n văn là trình bày lại các ket quả ve cau trúc
  • 5. 1 Viết đề tài giá sinh viên – ZALO:0973.287.149-TEAMLUANVAN.COM nghi m của các phương trình x2 − Dy2 = ±1 và x2 − Dy2 = ±4. Lu n văn gom 2 chương: Chương 1: Chúng tôi giới thi u các ket quả ve liên phân so, giản phân và cau trúc nghi m của phương trình Diophantine x2 − Dy2 = ±1. Chương 2: Chúng tôi trình bày lại cau trúc nghi m của phương trình Diophan- tine x2 − Dy2 = ±4 và m®t so áng dụng trong toán phő thông. Lu n văn này được thực hi n và hoàn thành vào tháng 5 năm 2018 tại trường Đại hoc Khoa hoc - Đại hoc Thái Nguyên. Qua đây, tác giả xin bày tỏ lòng biet ơn sâu sac tới PGS.TS. Nông Quoc Chinh, người đã t n tình hướng dan tác giả trong suot quá trình làm vi c đe hoàn thành lu n văn này. Tác giả xin gải lời cảm ơn chân thành đen Khoa Toán - Tin, Trường Đại hoc Khoa hoc - Đại hoc Thái Nguyên, đã tạo moi đieu ki n đe giúp tác giả hoc t p và hoàn thành lu n văn cũng như chương trình thạc sĩ. Tác giả cũng xin gải lời cảm ơn tới t p the lớp cao hoc, khóa 05/2016 - 05/2018 đã đ®ng viên giúp đơ tác giả trong quá trình hoc t p và hoàn thành lu n văn này. Đong thời tác giả xin gải lời cảm ơn tới Ban giám hi u, các đong nghi p tại trường THPT Nguyen Khuyen, huy n Vĩnh Bảo, Hải Phòng và gia đình bạn bè đã tạo đieu ki n tot nhat cho tác giả trong suot quá trình hoc t p và hoàn thành lu n văn. Tác gia Vũ Phú Bình
  • 6. 2 Viết đề tài giá sinh viên – ZALO:0973.287.149-TEAMLUANVAN.COM i=0 i=0 Chương 1 Phương trình Diophantine x2 − Dy2 = ±1 Trong chương này, chúng tôi trình bày m®t so ket quả ve liên phân so, m®t so cách giải phương trình Diophantine dạng x2 − Dy2 = ±1 và áng dụng của nó. Các ket quả trong chương này được viet theo các tài li u [1] và [2]. 1.1 Liên phân so và giản phân 1.1.1 Liên phân so hfiu hạn và giản phân Định nghĩa 1.1.1. Cho {ai}∞ i=0 và {bi}∞ i=0 là dãy các so thực. (i) Bieu thác có dạng b0 a0 + b1 (1.1) a1 + 2 + ... được goi là m®t liên phân so của hai dãy so {ai}∞ i=0 và {bi}∞ i=0. (ii) Dãy các bieu thác u = a , u = a + b0 , u b0 = a + , . . . , được 0 0 1 0 a1 2 a + b1 1 a2 goi là các giản phân của hai dãy so {ai}∞ i=0 và {bi}∞ i=0. (iii) Phan tả un xác định như trên được goi là giản phân thú n của hai dãy so {ai}∞ i=0 và {bi}∞ i=0. Chú j 1.1.2. (i) Neu n là hǎu hạn và b0 = b1 = . . . = bn = 1 ta kí hi u liên phân so của hai dãy so {ai}n và {bi}n là [a0; a1, . . . , an] . (ii) Neu a0 ∈ Z và a1, ..., an là các so nguyên dương thì ta nói [a0; a1, ..., an] là a 0
  • 7. 3 Viết đề tài giá sinh viên – ZALO:0973.287.149-TEAMLUANVAN.COM m®t liên phân so hũu hạn có đ® dài n. (iii) M®t liên phân so hǎu hạn là m®t so hǎu t . Với hai dãy so thực {ai}∞ i=0 và {bi}∞ i=0 ta xét hai dãy so {pn}∞ n=−1 và {qn}n ∞ =−1 như sau: p−1 = 1, p0 = a0, . . . , pn+1 = an+1pn + bnpn−1. q−1 = 0, q0 = 1, . . . , qn+1 = an+1qn + bnqn−1. Khi đó moi quan h giǎa giản phân thá n của hai dãy so {ai}∞ i=0 và {bi}∞ i=0 với thương thá n của hai dãy so {pn}∞ n=−1 và {qn}n ∞ =−1 được the hi n trong bő đe sau. Bo đe 1.1.3. Với các kí hi u và giả thiet như trên ta có giản phân un moi n ≥ 0. = pn qn với Chúng minh. Ta cháng minh đȁng thác trên là đúng bang quy nạp theo n. Th t v y, với n = 0 và n = 1 thì hien nhiên ket quả là đúng. Giả sả quy nạp đúng cho n, nghĩa là ta có u = pn . Thay a trong bieu thác u bởi a bn + ta thu n qn n n n an+1 được un+1. Theo định nghĩa ta có pn, qn không phụ thu®c vào bn và an+1 nên tà công thác truy hoi ta có pn = anpn−1 + bn−1pn−2 qn anqn−1 + bn−1qn−2 un+1 = (an bn + )p an+1 bn n−1 + bn−1 pn−2 (an + n+1) qn−1 + bn−1qn−2 = (anan+1 + bn)pn−1 + an+1bn−1pn−2 (anan+1 + bn)qn−1 + an+1bn−1qn−2 = an+1(anpn−1 + bn−1pn−2) + bnpn−1 an+1(anqn−1 + bn−1qn−2) + bnqn−1 = an+1pn + bnpn−1 an+1qn + bnqn−1 = pn+1 . qn+1 Bő đe được cháng minh. a
  • 8. 4 Viết đề tài giá sinh viên – ZALO:0973.287.149-TEAMLUANVAN.COM i=0 i=0 i=0 Bő đe 1.1.3 cho ta m®t công thác tính các giản phân qua thương của các dãy so. M nh đe tiep theo chỉ ra rang moi so hǎu t đeu bieu dien được dưới dạng m®t liên phân so hǎu hạn và bieu dien đó là duy nhat. Trước tiên ta nhac lại thu t toán Euclid tìm ước chung lớn nhat của hai so nguyên. Chú j 1.1.4. (i) Cho các so nguyên a, b ∈ Z, b > 0. Khi đó như đã biet chúng ta có the tìm được ước chung lớn nhat của a và b bang cách thác hi n thu t toán Euclid như sau: a = a0b + r1, 0 < r1 < b b = a1r1 + r2, 0 < r2 < r1, r1 = a2r2 + r3, 0 < r3 < r1, . . . , rn−2 = an−1rn−1 + rn, 0 < rn < rn−1, rn−1 = anrn, quá trình này phải dàng và sau hǎu hạn bước ta có gcd(a, b) = rn. (ii) Tà thu t toán trên ta thu được hai dãy so nguyên hǎu hạn là {ai}n và b0 = b1 = . . . = bn = 1. Khi đó các giản phân của {ai}n 1 và {bi}n là u0 = a0 = [a0], u1 = a0 + 1 = [a0; a1], . . . , un = ... = [a0; a1, a2, . . . , an]. (iii) Tà thu t toán trên ta cũng thu được các dãy truy hoi là p0 = a0, p1 = a1p0 + 1, . . . , pn = anpn−1 + pn−2 và q0 = 1, q1 = a1, . . . , qn = anqn−1 + qn−2. Ta có tính chat quan trong của so hǎu t the hi n trong m nh đe sau: M nh đe 1.1.5. Mői so hũu ty đeu được bieu dien dưới dạng m®t liên phân so hũu hạn. a
  • 9. 5 Viết đề tài giá sinh viên – ZALO:0973.287.149-TEAMLUANVAN.COM Chúng minh. Cho a/b là m®t so hǎu t với b > 0. Theo thu t toán tìm ước chung lớn nhat và công thác giản phân ta có a 1 b = a0 + b r1 1 = a0 + 1 . . . = a0 + a1 + a1 + r1 r2 1 1 . 1 a2 + 1 a3 + n−2 . . . + 1 an−1 + a V y moi so hǎu t a/b đeu viet được thành m®t liên phân so hǎu hạn là a/b = [a0; a1, . . . , an]. M nh đe được cháng minh. Như chúng ta đã biet bieu dien của m®t so hǎu t dưới dạng phân so không là duy nhat. Tuy nhiên m nh đe tiep theo chỉ ra rang bieu dien của m®t so hǎu t thành liên phân so là duy nhat. M nh đe 1.1.6. Bieu dien so hũu ty thành m®t liên phân so hũu hạn dạng [a0; a1, . . . , an] là duy nhat. Chúng minh. Cho a/b là m®t so hǎu t và giả sả a [a0; a1, . . . , an] = b = [b0; b1, . . . , bm]. Ta can cháng minh m = n và ai = bi, với moi i = 0, 1, . . . , n. Th t v y, với n = 0 ta có a0 = [b0; b1, . . . , bm]. Vì b0 là phan nguyên của a0 và a0 là so nguyên nên m = 0 và a0 = b0. Giả sả quy nạp đúng cho n − 1, nghĩa là ket lu n trên là đúng cho moi liên phân so hǎu hạn có đ® dài nhỏ hơn n. Tà bieu thác a [a0; a1, . . . , an] = b = [b0; b1, . . . , bm] a n
  • 10. 6 Viết đề tài giá sinh viên – ZALO:0973.287.149-TEAMLUANVAN.COM ta suy ra a0 = b0, vì đeu là phan nguyên của cùng m®t so hǎu t . Khi đó ta có a [0; a1, . . . , an] = b − a0 = [0; b1, . . . , bn]. Do đó [a1; a2, . . . , an] = [b1; b2, . . . , bm]. Theo giả thiet quy nạp ta có n−1 = m−1 và ai = bi, với moi i = 1, . . . , n. Ví dn 1.1.7. Xét so hǎu t 187/4, ta có 187 = 46.4 + 3, 4 = 1.3 + 1, 3 = 3.1. Do đó 187 4 = [46; 1, 3]. 1.1.2 Liên phân so vô hạn Trong mục này chúng tôi t p trung trình bày các kien thác ve liên phân so vô hạn. Trong đó chúng tôi trình bày lại m®t tính chat tot của liên phân so vô hạn đó là moi so vô t đeu viet được dưới dạng m®t liên phân so vô hạn. Các ket quả trong mục này được viet theo các tài li u [1]. Định nghĩa 1.1.8. (i) Liên phân so vô hạn là m®t bieu thác có dạng 1 q0 + q1 + q2+ 1 ... 1 + qs ... (1.2) trong đó, q0 là m®t so nguyên, qs với s = 1, 2, ... là các so nguyên dương và được kí hi u là [q0; q1, . . . , qs, . . .]. (ii) Phan tả qs được goi là so thương hụt hay so hạng thá s của liên phân so.
  • 11. 7 Viết đề tài giá sinh viên – ZALO:0973.287.149-TEAMLUANVAN.COM 3 4 Chú j 1.1.9. (i) Cho α ∈ / Q là m®t so thực. So [α] được định nghĩa là so nguyên sao cho [α] ≤ α < [α] + 1. Đ t 1 α0 = α = a0 + 1 , với a0 = [α] ∈ Z, α1 > 1. Vì α ∈ / Q nên α1 không là so nguyên. Khi đó ta có 1 α1 = a1 + 2 , với a1 = [α1] ∈ Z, α2 > 1. Tiep tục quá trình trên đen bước thá n + 1 ta được 1 αn = an + αn+1 , với an = [αn] ∈ Z, αn+1 > 1. Vì α ∈ / Q nên αn+1 không là so hǎu t nên quá trình này kéo dài vô hạn. Do đó α = [a0; a1, . . . , ] là m®t bieu dien của so vô t qua liên phân so vô hạn. Đ t πn = [a0; a1, . . . , an], với moi n = 0, 1, . . . . Khi đó các πn được goi là các liên phân so hũu hạn của liên phân so vô hạn α. (ii) Tà cách tìm các so ai và αi ta suy ra bieu dien của m®t so vô t α = [a0; a1, . . . , ] là duy nhat. Tà Chú ý 1.1.9 ta suy ra h quả quan trong sau: H quả 1.1.10. Mői so vô ty đeu bieu dien m®t cách duy nhat dưới dạng m®t liên phân so vô hạn. Chúng minh. Theo Chú ý 1.1.9 (i) thì moi so vô t đeu có bieu dien thành liên phân so vô hạn. Tiep theo giả sả [a0; a1, . . . , ] = α = [b0; b1, . . . , ] là hai bieu dien của so vô t α. Theo Chú ý 1.1.9 (ii) ta suy ra đieu can cháng minh. Ví dn 1.1.11. Cho α = √ 3 = 1 + 1 . Suy ra a = √ 1 1 = 1 + . Tà đó ta a1 1 có 2 1 3 − 1 a2 Do đó a2 = √ 3 − 1 = 2 + a . 1 1 M®t cách tương tự a3 = √ 3 − 1 = 1 + a . 1 3 a2n = √ 3 − 1 , a2n+1 = √ 2 − 1 với moi n ≥ 1. Vì the √ 3 = [1, 1, 2, 1, 2, 1, 2 . . .]. α α
  • 12. 8 Viết đề tài giá sinh viên – ZALO:0973.287.149-TEAMLUANVAN.COM 3 — − ≥ √ p . . . . q ∀k : α − < . qk . qkqk+1 qkqk 2 k Ví dn 1.1.12. Cho α = √ 2 = 1 + 1 . Suy ra a = √ 1 1 = 2 + . Tà đó ta a1 1 có 1 1 2 − 1 a2 a2 = √ 2 − 1 = 2 + a . M®t cách tương tự an 1 = 2 − 1 với moi n ≥ 1. Vì the √ 2 = [1, 2, 2, 2 . . .]. Tiep theo là m®t so tính chat của liên phân so vô hạn được dùng cho phan cháng minh sau. Chú j 1.1.13. Với hai dãy so nguyên dương {ai}∞ i=0 và bi = 1, i = 0, 1, . . . ta xét các dãy truy hoi sau p0 = a0, p1 = a1p0 + 1, pn = anpn−1 + pn−2, q0 = 1, q1 = a1,n = anqn−1 + qn−2. Khi đó các tính chat sau là đúng: (i) πi = pi , với moi i = 0, 1, . . . . qi (ii) pn và qn là nguyên to cùng nhau, nghĩa là πn là phân so toi giản. (iii) α = pn−1αn + pn−2 , với moi n ≥ 2. (iv) lim k→∞ qn 1αn + qn 2 πk = α. n+1 (v) πn — πn−1 = (−1) , với moi n 1. qn−1qn Bo đe 1.1.14. Cho √ d là so vô tí. Khi đó, ton tại vô so c¾p so nguyên dương (p, q) thóa mãn: (i) . d − . < . . q. . q2 √ (ii) .p2 − dq2. < 1 + 2 d. Chúng minh. (i) Theo tính chat của liên phân so vô hạn ta có pk 1 . qk . qkqk+1 với α ∈ R. Theo cách xác định {qk} ta có qk+1 = akqk + qk−1 > ak−1qk−1 + qk−2 = qk. Vì the pk 1 1 1 ∀k : α − < < = . √ 1
  • 13. 9 Viết đề tài giá sinh viên – ZALO:0973.287.149-TEAMLUANVAN.COM √ p 1 q √ 2 2 y . q. . . q . p + q √ d . = . p − q √ d + 2q √ d . ≤ . q √ d − p . + . 2q √ d . y 2y2 y − y Chon (ii) Th t v y, ta có p = pk ; q = qk ; α = √ d. . d − . < ⇒ 0 < . q √ d − p . < 1 ⇒ q > 0. . . < . 1 + 2q √ d ≤ q + 2 . q √ d . = 1 + . 2 √ d . q. . Suy ra .p − dq . < 1 + 2 d. Ta có đieu can cháng minh. Bo đe 1.1.15. Giả sủ d và n là so nguyên sao cho d > 0 và d không là so chính phương, |n| < √ d. Khi đó neu x2 − dy2 = n thì x là m®t giản phân của liên phân so của √ d với x, y ∈ Z và y > 0. Chúng minh. Đe cháng minh bő đe, ta xét hai trường hợp. Trường hợp thá nhat, với n > 0 ta có x2 − dy2 = n hay (x + y √ d)(x − y √ d) = n, suy ra x − y √ d > 0, v y x > y √ d. Do đó x − √ d > 0. Hơn nǎa, vì 0 < n < √ d nên x √ x − y √ d x2 − dy2 d = = y y y n x + y √ d n = y x + y √ < 2y √ d √ d 1 < 2y2 √ d = 2y2 . Do 0 < x − √ d < 1 x ta suy ra được là giản phân của liên phân so của √ d. Trường hợp thá 2, với n < 0 chia hai ve của x2 − dy2 = n cho −d ta được y2 − 1 x2 = − n . d d n y L p lu n tương tự như trên, neu − d > 0 thì là giản phân của liên phân so x của 1 √ d . Do đó 1 x 1 = là giản phân của liên phân so của y 1 x √ d = √ d. y d Mà q2 y
  • 14. 10 Viết đề tài giá sinh viên – ZALO:0973.287.149-TEAMLUANVAN.COM k k k+1 d − s k+1 = k + k k k + 2aksk − aktk V y 2aksk − a2 tk ∈ Z và tk| d − s2 theo giả thuyet quy nạp nên tk+1 ∈ Z. Bo đe 1.1.16. Cho liên phân so √ d = [a ;a1 , a2 , a3 , ..., an , ...] và các dãy so {sk}, {tk} được xác đ nh bang h thúc: s0 = 0 ; t0 = 1 d − s2 sk+1 = aktk − sk ; tk+1 = k+1 tk với k ≥ 0. Khi đó các phát bieu sau là đúng: (i) sk, tk ∈ Z và tk = / 0; (ii) tk| d − s2 ; sk + √ d (iii) xk = k k ≥ 0 trong đó xk = [a0; a1, ..., ak]. Chúng minh. Ta cháng minh (i) và (ii) bang quy nạp theo k. Với k = 0 thì khȁng định là hien nhiên. Giả sả với k > 0 và sk, tk được xác định như trên thì sk, tk ∈ Z , tk 0 và tk| d − s2 . Ta can cháng minh (i) và (ii) đúng với k + 1. Th t v y, sk+1 = aktk − sk ∈ Z. M t khác tk+1 thuan). Ta có 0, vì neu tk+1 = 0 thì d = s2 ⇒ d là so chính phương (mâu tk+1 = = 2 k+1 tk d − (aktk − sk)2 tk d − s2 2aktksk − a2 t2 tk tk d − s2 2 k Suy ra k tk+1| d − s2 . t k 0 = . t
  • 15. 11 Viết đề tài giá sinh viên – ZALO:0973.287.149-TEAMLUANVAN.COM d − 2 s 0 1 2 3 k k+1 ( sk+1qkpk + qk−1tk+1pk = p , k k k (iii) Ta cháng minh bang quy nạp theo k. Với k = 0, luôn đúng. Giả sả với sk + √ d k > 0 ta có xk = ta có . Ta can cháng minh (iii) đúng với k + 1. Th t v y, tk 1 xk+1 = k = s ak tk + √ d − a t k k k tk = √ d − sk+1 tk sk+1 + √ d k+1 s + √ d Bo đe 1.1.17. Neu pk qk = k+1 tk+1 là giản phân của liên phân so của √ d thì p2 − dq2 = (−1)k+1 tk+1 , k = 0, 1, 2, ... và tk+1 > 0. Chúng minh. Giả sả √ d = [a ; a , a , a , ...a , x ] . Khi đó, ta có √ d = xk+1pk + pk−1 và xk+1 s + √ d = k+1 . V y tk+1 sk+1 + √ d .q + q xk+1qk + qk−1 ! √ d = sk+1 + √ d .p + p . Suy ra tk+1 k k−1 tk+1 k k−1 √ d (sk+1qk + qk−1tk+1 − pk) = sk+1pk + tk+1pk−1 − dqk. Do ve phải là so hǎu tỉ và √ d là so vô tỉ nên sk+1qk + qk−1tk+1 = pk, sk+1pk + tk+1pk−1 = dqk. Lan lượt nhân 2 ve của h thác với pk và −qk ta có 2 k −sk+1pkqk − tk+1pk−1qk = −dq2 . − ( x =
  • 16. 12 Viết đề tài giá sinh viên – ZALO:0973.287.149-TEAMLUANVAN.COM 2 2 ∀ ≥ k k k k k k k 1 d < C®ng ve theo ve ta được p2 − dq2 = tk+1 (qk−1pk − qkpk−1) . M t khác, vì qk−1pk − qkpk−1 = (−1)k+1 nên ta có p2 − dq2 = (−1)k+1 tk+1. Vì p2k q2k < √ d < p2k+1 q2k+1 với moi k ≥ 0 nên neu k chȁn ta có pk < √ d ⇒ p < q √ d ⇒ p2 − dq2 < 0. qk Neu k lẻ ta có √ d < pk k k k k ⇒ q √ d < p ⇒ p − dq > 0. Ta xét bieu thác qk k k k k p2 − dq2 (−1)k+1 tk+1 tk+1 k k = = − (k ≥ 1) . p2 −1 − dq2 −1 (−1)k t tk Neu k lẻ thì (k − 1) chȁn nên p2 − dq2 > 0 ; p2 — dq < 0 ⇒ − tk+1 < 0 ⇒ tk+1 > 0 ∀k ≥ 1. k k k−1 k−1 tk tk Neu k chȁn thì (k − 1) lẻ nên p2 − dq2 < 0 ; p2 — dq > 0 ⇒ − tk+1 < 0 ⇒ tk+1 > 0 ∀k ≥ 1. k k k−1 k−1 tk tk V y neu t0 = 1 > 0 thì d − s2 h√ i √ Mà t2 t1 > 0 ta suy ra t2 > 0. Tương tự ta có tk > 0 k 0 mà tk+1 tk > 0 nên suy ra tk+1 > 0 ∀ k ≥ 0. Bo đe 1.1.18. So tự nhiên n thóa mãn √ d = [a ;a1 ,a2 , ..., an ,...] = [a0 ;a1 ,a2 , ..., an−1 , an] được goi là chieu dài chu kì của liên phân so của √ d. Khi đó t khi n| j. = 1 khi và chí 2 2 0 0 t1 = t0 = d − a2 > 0 a0 = d . j
  • 17. 13 Viết đề tài giá sinh viên – ZALO:0973.287.149-TEAMLUANVAN.COM s t + √ √ t d = s t+ t d, 1 kn+1 Chúng minh. Cho √ d = [a0; a1, a2, ..., an]. Ta có xkn+1 = x1 với k ≥ 0. Khi đó s + √ d s + √ d kn+1 = 1 , tkn+1 t1 khȁng định này tương đương với kn+1 1 1 1 kn+1 kn+1 nghĩa là (tkn+1 − t1) √ d = skn+1t1 − s1tkn+1. Ta có tkn+1 = t1, skn+1 = s1. M t khác, t1 = d − s2 = d − s2 = tkntkn+1 = tknt1. Suy ra tkn = 1. V y ket quả nh n được là tj = 1 , j = kn nên n|j. Ngược lại, cho j là so nguyên dương thỏa mãn tj = 1. Khi đó, tà Bő đe 1.1.16 ta có Vì sj ∈ Z nên ta có s + √ d xj = j = sj tj + √ d. [xj] = h sj + √ d i = sj + h√ d i = sj + a0. Theo cách xác định của xj+1 ta có xj = [xj] + 1 xj+1 = sj + a0 + x 1 . j+1 M t khác, √ d = x 1 = a + và √ d = x − s 1 1 = a + . Suy ra a + = 0 0 x1 1 j j 0 xj+1 0 x1 a0 + xj+1 . Do đó x1 = xj+1. Đieu đó có nghĩa dãy a1, a2, ..., aj (j ∈ Z) l p lại √ trong sự bieu dien liên phân so của dài n chu kì của liên phân so √ d. d. Suy ra j = kn. V y j là b®i của chieu 1.2 Phương trình Diophantine x2 − Dy2 = ±1 Như chúng ta đã biet phương trình Pell là phương trình có dạng x2 − dy2 = n, trong đó d và n là so nguyên cho trước. Khi n = ±1 đã có nhieu công trình liên
  • 18. 14 Viết đề tài giá sinh viên – ZALO:0973.287.149-TEAMLUANVAN.COM quan đen vi c tìm nghi m nguyên của phương trình này. Trong mục này chúng tôi trình bày lại phương pháp giải và cau trúc nghi m của m®t so phương trình Pell đã biet. Các ket quả trong mục này được viet theo các tài li u [4]. 1.2.1 Phương trình Pell dạng x2 − dy2 = 1 Xét phương trình x2 − dy2 = 1 (1.3) trong đó d là so nguyên cho trước. Định lj 1.2.1. Neu d là so chính phương (d = m2 ) thì phương trình (1.3) không có nghi m nguyên dương. Chúng minh. Vì d = m2 , m > 0 nên x2 − m2 y2 = 1, đieu này tương đương với (x − my)(x + my) = 1. Giả sả phương trình (1.3) có nghi m nguyên dương. Khi đó ta có (x − my) ∈ Z và (x + my) ∈ Z≥0. Do đó x + my = 1, x − my = 1. Vì x, y, m là các so nguyên dương nên x = 1 và y = 0. Đieu này là mâu thuan. V y phương trình (1.3) không có nghi m nguyên dương. M nh đe 1.2.2. Neu d là so nguyên âm thì phương trình (1.3) không có nghi m nguyên dương. Chúng minh. Đ t d = −dJ với dJ > 0. Khi đó phương trình (1.3) được viet lại dưới dạng x2 + dJ y2 = 1. Neu dJ = 1, thì x2 + y2 = 1. Suy ra x = 1, y = 0 ho c x = 0, y = 1. Neu dJ > 1, thì x = 1 và y = 0. V y phương trình (1.3) không có nghi m nguyên dương.
  • 19. 15 Viết đề tài giá sinh viên – ZALO:0973.287.149-TEAMLUANVAN.COM √ 2 2 h i √ √ S S x1 ≡ x2(mod |k|) 1 1 2 2 Định lý 1.2.1 và M nh đe 1.2.2 cho ta m®t so đieu ki n đe phương trình trên không có nghi m nguyên dương. Định lý tiep theo chỉ ra đieu ki n can và đủ đe phương trình có nghi m nguyên dương. Định lj 1.2.3. Phương trình (1.3) có nghi m nguyên dương khi và chí khi d là so nguyên dương và không phải là so chính phương. Chúng minh. Theo Định lý 1.2.1 và M nh đe 1.2.2 ta có phương trình (1.3) có nghi m nguyên dương thì d là so nguyên dương và không chính phương. Ngược lại, theo Bő đe 1.1.14 ta có ton tại vô so c p so (x, y) nguyên dương sao cho .x − dy . < 1 + 2 d Suy ra −1 − 2 √ d < x2 − dy2 < 1 + 2 √ d (1.4) Tà tính vô hạn của các c p so nguyên dương (x, y) nên ton tại k ∈ Z thỏa mãn k ∈ −1 − 2 d, 1 + 2 d sao cho có vô so c p so nguyên dương (x, y) thỏa mãn Khi đó ta xét t p x2 − dy2 = k. H = , (x, y) : x2 − dy2 = k, với k ∈ h −1 − 2 √ d, 1 + 2 √ d i, và Hi,j = {(x, y) ∈ H : x ≡ i (mod |k| ) và y ≡ j (mod |k|) , 0 ≤ i, j ≤ |k|} . Vì có vô so (x, y) nên suy ra |H| = +∞. Ta có 0 ≤ i, j ≤ |k| nên so c p (i, j) là hǎu hạn. Tà định nghĩa của hai t p H và Hi,j suy ra H = Hi,j và |H| = | Hi,j| = +∞. Do so c p (i, j) là hǎu hạn nên ton tại (i, j) sao cho |Hi,j| = +∞. Xét Hi,j thỏa mãn |Hi,j| = +∞. Khi đó ton tại (x1, y1) /= (x2, y2) sao cho (xk, yk) ∈ Hi,j (k = 1, 2) và thỏa mãn y1 ≡ y2(mod |k|) x2 − dy2 = x2 − dy2 = k. (1.5)
  • 20. 16 Viết đề tài giá sinh viên – ZALO:0973.287.149-TEAMLUANVAN.COM ( − 1 1 2 2 2 2 1 1 2 2 Ta có (x1 − y1 √ d)(x2 + y2 √ d) = x1x2 − dy1y2 + √ d (x1y2 − x2y1) . (1.6) Theo công thác (1.5) ta có ( x1x2 − dy1y2 ≡ x2 − dy2 ≡ 0 mod (|k|) x1y2 − x2y1 ≡ x2y2 − x2y2 ≡ 0 mod (|k|) . Do đó ton tại (u, v) ∈ Z thỏa mãn x1x2 − dy1y2 = ku, x1y2 − x2y1 = kv. Ket hợp (1.5), (1.6), (1.7) và (1.8) ta được (x1 y1 √ d)(x2 + y2 √ d) = k(u + v √ d), (x1 + y1 √ d)(x2 − y2 √ d) = k(u − v √ d). Nhân hai ve của h phương trình ta có (1.8) (x2 − x2 − u2 − ). Vì (x2 − dy2 )(x2 − dy2 ) = k2 suy ra u2 − dv2 = 1. Cuoi cùng ta can cháng minh u và v là nhǎng so nguyên dương. Th t v y, vì u2 = 1 + dv2 suy ra u ≥ 1. Vì ta chỉ xét nghi m dương nên ta có u > 0. Giả sả v = 0 ta có u2 = 1 suy ra u = 1. V y (x1 − y1 √ d)(x2 + y2 √ d) = k. Theo (1.5) ta có x2 − dy2 = (x1 − y1 √ d)(x1 + y1 √ d) = k, đieu này suy ra 1 1 (x + y √ d) = (x + y √ d). (1.9) Giả sả x — y √ d = 0 ⇒ √ d = x1 và do đó √ d là so hǎu tỉ, đieu này là mâu 1 1 √ y1 √ √ thuan. V y x1 − y1 d /= 0. Tà (1.9) ta có x1 − x2 = (y2 − y1) d. Vì d so vô tỉ và xi, yi ∈ Z+ nên x1 − x2 = 0, y1 − y2 = 0. ( (1.7) dy2 )( dy2 ) = k2 ( dv2 1 1 2 2
  • 21. 17 Viết đề tài giá sinh viên – ZALO:0973.287.149-TEAMLUANVAN.COM ( ( j j j j Khȁng định này tương đương với x1 = x2 y1 = y2 nghĩa là (x1, y1 = (x2, y2). Đieu này mâu thuan và do đó v > 0. V y ton tại c p so nguyên dương (u, v) sao cho u2 − dv2 = 1. Định lý tiep theo cho ta cau trúc nghi m của phương trình thông qua giản phân của liên phân so. Định lj 1.2.4. Cho pk qk là giản phân của liên phân so của √ d với n là chieu dài của nó. Khi đó các phát bieu sau là đúng: (i) Neu n chȁn, khi đó tat cả các nghi m dương của phương trình (1.3) được cho bới: x = pkn−1 y = qkn−1 (k ≥ 1) ; (ii) Neu n lé, khi đó tat cả các nghi m dương của phương trình (1.3) được cho bới: x = p2kn−1 y = q2kn−1 (k ≥ 1) . Chúng minh. Theo Bő đe 1.1.15 ta có x0 , y0 là nghi m của phương trình x2 − dy2 = 1. Trong đó x0 = pj và y0 = qj với pj qj là giản phân của liên phân so của √ d. Theo Bő đe 1.1.17 thì: p2 − dq2 = (−1)j+1 tj+1 j = 0, 1, 2, ... và tj+1 > 0. Do p2 − dq2 = 1 suy ra tj+1 = 1 và j + 1 là so chȁn. Theo bő đe 1.1.17 thì n| (j + 1) suy ra j + 1 = nk ′ k ′ ∈ Z . V y j = k ′ n − 1(k ′ , n ≥ 1). (i) Neu n chȁn và (j + 1) chȁn nên ∀k ′ ≥ 1 ; k ′ = k (k ≥ 1) ⇒ j = kn − 1. V y nghi m của phương trình (1.3) là: x = pkn−1 và y = qkn−1 (k ≥ 1) . (
  • 22. 18 Viết đề tài giá sinh viên – ZALO:0973.287.149-TEAMLUANVAN.COM − (ii) Neu n lẻ và (j + 1) chȁn nên kJ chȁn và đ t kJ = 2k. Khi đó k ≥ 1 suy ra j = 2kn − 1. V y nghi m của phương trình (1.3) là: x = p2kn−1 và y = q2kn−1 với k ≥ 1. Tiep theo ta xét m®t ví dụ minh hoa cho Định lý 1.2.4. Ví dn 1.2.5. Tìm nghi m nhỏ nhat của phương trình x2 − 13y2 = 1. Th t v y, ta có √ 13 = 3; 1, 1, 1, 1, 6 với chieu dài chu kì 5 là so lẻ. Nên ta có các giản phân sau: ak 3 1 1 1 1 6 1 1 1 6 1 pk 1 3 4 7 11 18 119 137 256 393 649 qk 0 1 1 2 3 5 33 38 71 109 180 Ta có tả so và mau so của giản phân p10k−1 là dạng nghi m của phương trình đã cho. Với k = 1 ta có g9 = p9 q9 q10k 1 649 = 180 nên (649, 180) là c p nghi m bé nhat của phương trình đã cho. Đe tìm các c p nghi m còn lại ta lan lượt the k = 2, 3, 4, . . . vào công thác p10k−1 q10k−1 và dựa vào bảng các giản phân. Chú j 1.2.6. Nghi m (x, y) của phương trình Pell (1.3) được goi là nghi m cực tieu neu x < u với (u, v) là nghi m bat kỳ khác của phương trình (1.3). Nghi m cực tieu còn được goi là nghi m cơ bản. Định lý tiep theo cho ta m®t công cụ hǎu hi u đe tìm nghi m của phương trình Pell (1.3) thông qua nghi m cơ bản. Định lj 1.2.7. Cho (a, b) là nghi m cơ bản của phương trình (1.3). Với mői n = 0, 1, 2, ... các dãy {xn} và {yn} được xác đ nh như sau x0 = 1 ; x1 = a ; xn+2 = 2axn+1 − xn, y0 = 0 ; y1 = b ; yn+2 = 2ayn+1 − yn. Khi đó, với moi n ∈ N ta có (xn, yn) là nghi m dương của phương trình (1.3). (
  • 23. 19 Viết đề tài giá sinh viên – ZALO:0973.287.149-TEAMLUANVAN.COM 1 2 n n yn = d1λn + d2λn . Chúng minh. Áp dụng lí thuyet ve dãy so ta có phương trình đ c trưng của dãy trên là λ2 − 2aλ + 1 = 0. (1.10) Phương trình (1.10) có hai nghi m là λ1 = a − √ ∆ ′ và λ2 = a + √ ∆ ′ ∆ ′ = a2 − 1 . Vì (a, b) là m®t c p nghi m của phương trình x2 −dy2 = 1, nên ta có a2 −1 = db2 . V y hai nghi m của phương trình (1.10) là λ1 = a − b √ d và λ2 = a + b √ d. Tà đó theo lý thuyet dãy so, thì: ( xn = c1λn + c2λn , 1 2 Bây giờ ta xác định c1, c2; d1, d2 tà các đieu ki n x0 = 1, x2 = a; y0 = 0, y1 = b. Xét h phương trình sau ( x0 = 1 ⇔ c1 + c2 = 1, √ √ x1 = a Giải h trên ta có c1 a − b d + c2 a + b d = a. xn = và yn = a + b √ d n + a − b √ d n a + b √ d n − a − b √ d n 2 √ d . Vì v y xn và yn là các so nguyên dương. Tà đó ta có xn x + yn — y √ d = a + b √ d n , √ d = a − b √ d n . Vì v y x2 − dy2 = xn + yn √ d xn − yn √ d = a2 − db2 = 1. 2 n n
  • 24. 20 Viết đề tài giá sinh viên – ZALO:0973.287.149-TEAMLUANVAN.COM 1 1 1 1 n n n n 1 − 1 = 0 < r + s √ d − r − s √ d = 2s √ d ⇒ 2 s > 0. n 1 Do đó Do đó (xn, yn) là nghi m nguyên dương của phương trình Pell (1.3) với moi n = 0, 1, 2, ... Định lj 1.2.8. Cho (x1, y1) là nghi m cơ bản của phương trình x2 − dy2 = 1. Với mői nghi m dương (x, y) bat kì của phương trình thì ton tại n ≥ 1 sao cho x + y √ d = x + y √ d = x + y √ d n . n 1 1 Chúng minh. Ta dùng phương pháp cháng minh phản cháng. Giả sả u, v là nghi m dương của phương trình mà không đạt được dạng x1 + y1 √ d . Tác n là u + v √ d x1 + y1√ d n . Giả sả u + v √ d > x1 + y1 √ d n . Vì x + y √ d > 1 nên lũy thàa của x + y √ d sě lớn tùy ý. Đieu đó có nghĩa là: x1 + y1√ d n < u + v √ d < x + y1 √ d n+1 . Suy ra V y xn + yn √ d < u + v √ d < xn + yn √ d x1 + y1 √ d . 1 < u + v √ d xn − yn √ d < x1 + y1 √ d vì x2 − dy2 = 1. So nguyên r, s được xác định bởi r + s √ d = u + v √ d xn − yn √ d , hay r + s √ d = (uxn − dvyn) + (sxn − uyn) √ d. Suy ra r = uxn − dvyn ; s = sxn − uyn. Ta có r2 − ds2 = x2 − dy2 u2 − dv2 = 1. Do đó r, s là nghi m của phương trình (1.3). Ta cháng minh rang r, s > 0. Th t v y, vì 1 < r + s √ d < x + y √ d, (1.11) 1 mà r + s √ d r − s √ d = 1 nên 0 < r−s √ d < 1. Suy ra −1 < − r − s √ d . 1 + 0 = 1 < r + s √ d + r − s √ d = 2r r > 1 1
  • 25. 21 Viết đề tài giá sinh viên – ZALO:0973.287.149-TEAMLUANVAN.COM 1 1 1 1 ( ( ( ( V y r, s là nghi m dương của phương trình (1.3) và x1 < r ; y1 < s. Suy ra x + y √ d < r + s √ d. Đieu này mâu thuan với (1.11). Do đó ton tại so n sao cho moi u, v là nghi m nguyên dương của phương trình (1.3) đeu có dạng: u + v √ d = x + y √ d n . Ví dn 1.2.9. Tìm ba nghi m đau tiên của phương trình: x2 − 23y2 = 1 Th t v y, trước tiên ta xác định nghi m cơ bản của phương trình (*). Bang phương pháp the y = 1, 2, 3, ... vào bieu thác 23y2 + 1 ta có Với y = 1 thì x2 = 23 + 1 = 24 loại. Với y = 2 thì x2 = 23.4 + 1 = 92 loại. Với y = 3 thì x2 = 23.32 + 1 = 208 loại. Với y = 4 thì x2 = 23.42 + 1 = 369 loại. Với y = 5 thì x2 = 23.52 + 1 = 576 nên x = 24. V y nghi m cơ bản là (24, 5). Ta có xn+2 = 2axn+1 − xn yn+2 = 2ayn+1 − yn ⇒ x2 = 2ax1 − x0 y2 = 2ay1 − y0 ⇒ x2 = 2.24.24 − 1 = 1151 y2 = 2.24.5 − 0 = 240 V y nghi m thá hai của phương trình đã cho là (1151, 240). Tương tự, ta tìm nghi m thá ba như trên: x3 = 2ax2 − x1 y3 = 2ay2 − y1 ⇒ x3 = 2.24.1151 − 24 = 55224 y3 = 2.24.240 − 5 = 11515. V y nghi m thá ba của phương trình (*) là (55224, 11515). 1.2.2 Ứng dnng liên phân so √ D vào phương trình Pell x2 − Dy2 = 1 Trong phan này, chúng tôi trình bày lại m®t so ket quả ve áp dụng liên phân so của √ D trong đó D = {k2 + 1, k2 − 1, k2 + 2, k2 − 2, k2 + k, k2 − k}, (với k là so nguyên dương tùy ý) đe mô tả cau trúc nghi m nguyên của phương trình (1.3). Các ket quả trong mục này được viet theo tài li u [4]. (
  • 26. 22 Viết đề tài giá sinh viên – ZALO:0973.287.149-TEAMLUANVAN.COM ( D = − Định lj 1.2.10. Cho k ≥ 1 là so nguyên và D = k2 + 1. Khi đó các phát bieu sau là đúng: (i) Liên phân so của √ D là √ 1; 2 khi k = 1, k; 2k khi k > 1. (ii) (x1, y1) = (2k2 + 1, 2k) là nghi m cơ bản. T¾p {(xn, yn)}, thóa mãn xn = [k; 2k, ..., 2k] (1.12) ` 2n−1 ˛¸ lan x với n ≥ 2 là nghi m của phương trình x2 − (k2 + 1)y2 = 1 và ta có: (a) Hai nghi m liên tiep (xn, yn) và (xn+1, yn+1) thóa mãn xn+1 = (2k2 + 1)xn + (2k3 + 2k)yn, yn+1 = 2kxn + (2k2 + 1)yn, với n ≥ 1. (b) Các nghi m (xn, yn) thóa mãn các moi quan h sau đây xn = (4k2 + 1)(xn−1 + xn−2) − xn−3, yn = (4k2 + 1)(yn−1 + yn−2) − yn−3 với n ≥ 4. Chúng minh. (i) Cho D = k2 + 1. Neu k = 1 ta có √ 2 = [1; 2]. Với k > 1 ta có √ k2 + 1 = k + ( √ k2 + 1 k) = k + 1 1 √ k2 + 1 − k 1 1 = k + √ k2 + 1 + k = k + 2k + ( √ k2 + 1 − k) . V y ta có √ D = [k; 2k]. (ii) Cho √ D = [a0; a1, ..., al] là liên phân so có đ® dài tuan hoàn l. T p A−2 = 0, A−1 = 1, Ak = akAk−1 + Ak−2 và B−2 = 1, B−1 = 0, Bk = akBk−1 + Bk−2 với so nguyên không âm k. Khi đó C = Ak k Bk là giản phân thá k của √ D, và nghi m cơ bản của phương trình (1.3) là (x1, y1) = (Al−1, Bl−1) neu l là so chȁn và (x1, y1) = (A2l−1, B2l−1) neu l là so lẻ. Hơn nǎa, theo cháng minh trên ta có √ D = [k; 2k]. V y A0 = k, A1 = yn
  • 27. 23 Viết đề tài giá sinh viên – ZALO:0973.287.149-TEAMLUANVAN.COM n+1 n n 2k2 + 1, B0 = 1, B1 = 2k. Do đó (x1, y1) = (2k2 + 1, 2k) là nghi m cơ bản. Bây giờ, giả sả rang (xn, yn) là nghi m của phương trình x2 − (k2 + 1)y2 = 1. thì x2 − (k2 + 1)y2 = 1. Áp dụng (1.12) ta có n n xn+1 yn+1 1 = k + 1 2k + xn k + yn (2k2 + 1)x)n + (2k3 + 2k)yn Khi đó ta có = 2kxn . + (2k2 + 1)yn 2 n+1 — (k2 + 1)y2 = [(2k2 + 1)x)n + (2k3 + 2k)yn]2 −(k2 + 1)[2kxn + (2k2 + 1)yn]2 = x2 − (k2 + 1)y2 = 1. V y (xn+1, yn+1) là nghi m của phương trình x2 − (k2 + 1)y2 = 1. (a) Theo cháng minh phan 2 ta có xn+1 = (2k2 + 1)xn + (2k3 + 2k)yn và yn+1 = 2kxn + (2k2 + 1)yn với n ≥ 1. (b) Ta cháng minh xn = (4k2 + 1)(xn−1 + xn−2) − xn−3 (1.13) bang phương pháp quy nạp. Áp dụng (1.12) ta có x1 = 2k2 + 1, x2 = 8k4 + 8k2 + 1, x3 = 32k6 + 48k4 + 18k2 + 1 và x4 = 128k8 + 256k6 + 160k4 + 32k2 + 1. Ta kiem tra (1.13) đúng với n = 4. Th t v y x4 = (4k2 + 1)(x3 + x2) − x1 = (4k2 + 1)(32k6 + 48k4 + 18k2 + 1 + 8k4 + 8k2 + 1) − 2k2 + 1 . = 128k8 + 256k6 + 160k4 + 32k2 + 1. Ta giả sả rang công thác (1.13) đúng với n − 1, tác là xn−1 = (4k2 + 1)(xn−2 + xn−3) − xn−4. Ta phải cháng minh (1.13) đúng với n. Th t v y, theo cháng minh trên ta có xn+1 = (2k2 + 1)xn + (2k3 + 2k)yn. Do đó xn−1 = (2k2 + 1)xn−2 + (2k3 + 2k)yn−2, x
  • 28. 24 Viết đề tài giá sinh viên – ZALO:0973.287.149-TEAMLUANVAN.COM xn−2 = (2k2 + 1)xn−3 + (2k3 + 2k)yn−4, xn−3 = (2k2 + 1)xn−4 + (2k3 + 2k)yn−3. V y công thác (1.13) đúng với moi n ≥ 4. Tiep theo chúng tôi nhac lại m®t so ket quả tương tự cho các trường hợp khác của D mà không trình bày cháng minh của các định lý đó vì các cháng minh được cháng minh như trong cách cháng minh Định lý 1.2.10. Định lj 1.2.11. Cho k ≥ 1 là so nguyên và D = k2 + 2. Khi đó các phát bieu sau là đúng: (i) Liên phân so của √ D là √ D = [k; k, 2k]. (ii) (x1, y1) = (k2 + 1, k) là nghi m cơ bản. T¾p {(xn, yn)}, thóa mãn xn = [k; k; 2k, ..., k, 2k, k] (1.14) ` n−1 ˛¸ lan x với n ≥ 2 là nghi m của phương trình x2 − (k2 + 2)y2 = 1 và ta có: (a) Hai nghi m liên tiep (xn, yn) và (xn+1, yn+1) thóa mãn xn+1 = (k2 + 1)xn + (k3 + 2k)yn, yn+1 = kxn + (k2 + 1)yn với n ≥ 1. (b) Các nghi m (xn, yn) thóa mãn các moi quan h sau đây xn = (2k2 + 1)(xn−1 + xn−2) − xn−3, yn = (2k2 + 1)(yn−1 + yn−2) − yn−3 với n ≥ 4. Trường hợp D = {k2 − 1, k2 − 2, k2 + k, k2 − k} được the hi n trong các định lý sau: Định lj 1.2.12. Cho k ≥ 2 là so nguyên và D = k2 − 1. (i) Liên phân so của √ D là √ D = [k − 1; 1, 2k − 2]. yn
  • 29. 25 Viết đề tài giá sinh viên – ZALO:0973.287.149-TEAMLUANVAN.COM — − − ` ˛¸ x n (ii) (x1, y1) = (k, 1) là nghi m cơ bản. T¾p {(xn, yn)}, trong đó xn = [k; 1; 2k 2, ..., 1, 2k 2, 2k 1] (1.15) ` n−2 ˛¸ lan x với n ≥ 2 là nghi m của phương trình x2 − (k2 − 1)y2 = 1 và ta có: (a) Hai nghi m liên tiep (xn, yn) và (xn+1, yn+1) thóa mãn xn+1 = kxn + (k2 − 1)yn, yn+1 = xn + kyn với n ≥ 1. (b) Các nghi m (xn, yn) thóa mãn các moi quan h sau đây xn = (2k − 1)(xn−1 + xn−2) − xn−3, yn = (2k − 1)(yn−1 + yn−2) − yn−3 với n ≥ 4. Định lj 1.2.13. Cho k ≥ 2 là so nguyên và D = k2 − 2. Khi đó các phát bieu sau là đúng: (i) Liên phân so của √ D là √ D = ( 1; 2 khi k = 2 (ii) (x1, y1) = (k2 − 1, k) là nghi m cơ bản. T¾p {(xn, yn)}, trong đó xn y = [k − 1; 1, k − 2, 1, 2k − 2, ..., 1, k − 2, 1, 2k − 2, 1, k − 1]. (1.16) n−1 lan với n ≥ 2 là nghi m của phương trình x2 − (k2 − 2)y2 = 1 và ta có: (a) Hai nghi m liên tiep (xn, yn) và (xn+1, yn+1) thóa mãn xn+1 = (k2 − 1)xn + (2k3 − 2k)yn, yn+1 = kxn + (k2 − 1)yn với n ≥ 1. (b) Các nghi m (xn, yn) thóa mãn các moi quan h sau đây xn = (2k2 − 3)(xn−1 + xn−2) − xn−3, yn = (2k2 − 3)(yn−1 + yn−2) − yn−3 với n ≥ 4. k − 1; 1, k − 2, 2k − 2 khi k > 2. yn
  • 30. 26 Viết đề tài giá sinh viên – ZALO:0973.287.149-TEAMLUANVAN.COM k; k, 2k khi k > 1. D = ( — − − Định lj 1.2.14. Cho k ≥ 1 là so nguyên và D = k2 + k. Khi đó các phát bieu sau là đúng: (i) Liên phân so của √ D là √ D = ( 1; 2 khi k = 1, (ii) (x1, y1) = (2k + 1, 2) là nghi m cơ bản. T¾p {(xn, yn)}, trong đó xn = [k; k, 2k, ..., k, 2k, 2]. (1.17) ` n−1 ˛¸ lan x với n ≥ 2 là nghi m của phương trình x2 − (k2 + k)y2 = 1 và ta có: (a) Hai nghi m liên tiep (xn, yn) và (xn+1, yn+1) thóa mãn xn+1 = (2k + 1)xn + (2k2 + 2k)yn, yn+1 = 2xn + (2k + 1)yn với n ≥ 1. (b) Các nghi m (xn, yn) thóa mãn các moi quan h sau đây xn = (4k + 1)(xn−1 + xn−2) − xn−3, yn = (4k + 1)(yn−1 + yn−2) − yn−3 với n ≥ 4. Định lj 1.2.15. Cho k ≥ 2 là so nguyên và D = k2 − k. Khi đó các phát bieu sau là đúng (i) Liên phân so của √ D là √ 1; 2 khi k = 2, k − 1; 2, 2k − 2 khi k > 2. (ii) (x1, y1) = (2k − 1, 2) là nghi m cơ bản. T¾p {(xn, yn)}, trong đó xn = [k 1; 2, 2k 2, ..., 2, 2k 2, 2] (1.18) ` n−1 ˛¸ lan x với n ≥ 2 là nghi m của phương trình x2 − (k2 − k)y2 = 1 và ta có: (a) Hai nghi m liên tiep (xn, yn) và (xn+1, yn+1) thóa mãn xn+1 = (2k − 1)xn + (2k2 − 2k)yn, yn+1 = 2xn + (2k − 1)yn với n ≥ 1. (b) Các nghi m (xn, yn) thóa mãn các moi quan h sau đây xn = (4k − 3)(xn−1 + xn−2) − xn−3, yn = (4k − 3)(yn−1 + yn−2) − yn−3 với n ≥ 4. yn yn
  • 31. 27 Viết đề tài giá sinh viên – ZALO:0973.287.149-TEAMLUANVAN.COM 0 0 0 0 0 + 1 .d 1.2.3 Phương trình Pell dạng x2 − dy2 = −1 Trong phan này chúng tôi sě trình bày lại m®t so ket quả ve cau trúc nghi m của phương trình x2 − dy2 = −1 (1.19) trong đó d là so nguyên tùy ý cho trước. Đ c bi t, chúng tôi trình bày lại moi liên h giǎa nghi m của phương trình x2 − dy2 = 1 với nghi m của phương trình (1.19). Định lj 1.2.16. Phương trình (1.19) không có nghi m nguyên dương khi d = m2 với m > 0 là so nguyên (túc là d là so chính phương). Chúng minh. Cho m là so nguyên dương và d = m2 thì phương trình (1.19) có dạng x2 − m2 y2 = −1, khȁng định này tương đương với (my + x) (my − x) = 1. Tà tính nguyên dương của x và y nên suy ra: my + x = 1, my − x = 1. Suy ra x = 0. Đieu này là mâu thuan vì x > 0. V y phương trình Pell (1.19) không có nghi m nguyên dương, khi d là so chính phương. Định lj 1.2.17. Phương trình (1.19) không có nghi m khi d có ước nguyên to p = 4k + 3. Chúng minh. Giả sả khi d có ước nguyên to dạng p = 4k + 3, mà phương trình van có nghi m (x0, y0) : x2 − dy2 = −1. Đieu này tương đương với x2 +1 = dy2 . Tà đó suy ra: x2 . . Vì p = 4k + 3 nên theo lý thuyet chia het suy ra 1 chia het cho p. Đieu này là mâu thuan, v y giả thuyet là sai và phương trình (1.19) không có nghi m trong trường hợp này. (
  • 32. 28 Viết đề tài giá sinh viên – ZALO:0973.287.149-TEAMLUANVAN.COM d ≡ 1 (mod4) Định lj 1.2.18. Cho d là so nguyên to, khi đó phương trình (1.19) có nghi m nguyên dương khi và chí khi d không có dạng 4k + 3. Chúng minh. Giả sả phương trình (1.19) có nghi m. Khi đó theo định lý 1.2.11 thì d không có dạng 4k + 3. Ta cháng minh đieu ngược lại. Giả sả d /= 4k + 3 . Ta xét các trường hợp sau: d ≡ 2 (mod4) d ≡ 0 (mod4) Do d là so nguyên to nên chỉ còn 2 trường hợp: d ≡ 2 (mod4) , d ≡ 1 (mod4) . Trường hợp 1: d ≡ 2 (mod4) hay d có dạng d = 4k + 2 nên d chia het cho 2. Vì d là so nguyên to nên d = 2. V y phương trình (1.19) có nghi m nguyên dương (1, 1). Trường hợp 2: d ≡ 1 (mod4) hay d = 4k + 1. Xét phương trình Pell (1.3) x2 − dy2 = 1 là phương trình liên ket với phương trình (1.19). Goi (a, b) là nghi m dương nhỏ nhat của phương trình (1.3), khi đó ta có: a2 − 1 = db2 . (1.20) Ta xét hai khả năng sau: Neu a chȁn, thì ve trái của (1.20) lẻ và do d = 4k + 1 là so lẻ nên b lẻ. Do đó b2 ≡ 1 (mod4) . Suy ra a2 − 1 ≡ 1.1 (mod4) nên a2 ≡ 2 (mod4) . Đieu này mâu thuan với giả thiet (a chȁn nên a2 ≡ 0 (mod4)). Vì the không xảy ra khả năng này. Neu a lẻ, khi đó l p lu n tương tự như trên ta có b chȁn. Giả sả a = 2a1 + 1, b = 2b1. Thay vào (1.20) ta được: (2a1 + 2) 2a1 = d.4b2 nên (a1 + 1) a1 = db2 . (1.21) 1 1 " (
  • 33. 29 Viết đề tài giá sinh viên – ZALO:0973.287.149-TEAMLUANVAN.COM ( ( ( ( a1 + 1 = dv2 , 0 0 0 0 0 0 0 0 Vì d là so nguyên to và (a1, a1 + 1) = 1 nên tà (1.21) suy ra: a1 = u2 . a1 = du2 a1 + 1 = v2 Trong đó u.v = b1 với u, v là so nguyên dương. Neu a1 + 1 = dv2 a1 = u2 , suy ra u2 −dv2 = −1. Trong trường hợp này (u, v) là nghi m nguyên dương của phương trình (1.19). Neu a1 = du2 a1 + 1 = v2 , suy ra v2 − du2 = 1. Lúc này (v, u) là nghi m của phương trình (1.3). Do (a, b) là nghi m cơ bản của phương trình (1.3) nên ta có v ≥ a. Tà đó: a1 + 1 = v2 ≥ v ≥ a = 2a1 + 1 nên a1 ≥ 2a1. Bat đȁng thác thu được mâu thuan vì a1 là so nguyên dương. Do đó trường hợp này không the xảy ra. Định lj 1.2.19. Goi (a, b) là nghi m cơ bản của phương trình liên ket với phương trình Pell (1.19). Khi đó phương trình Pell (1.19) có nghi m khi và chí khi h có nghi m nguyên dương. a = x2 + dy2 b = 2xy (1.22) Chúng minh. Giả sả (x0, y0) là nghi m nguyên dương của h (1.22). Vì (a, b) là nghi m của phương trình x2 − dy2 = 1 nên a2 − db2 = 1. Tà đó theo h ta có 1 = x2 + dy2 2 − d(2x y )2 nên 1 = x2 − dy2 2 ⇒ x2 − dy2 = ±1. (
  • 34. 30 Viết đề tài giá sinh viên – ZALO:0973.287.149-TEAMLUANVAN.COM 0 0 0 0 0 0 0 0 0 0 0 0 0 0 0 0 0 0 Neu x2 − dy2 = 1 thì (x0, y0) là nghi m nguyên dương của phương trình Pell liên ket. Do (a, b) là nghi m nguyên dương nhỏ nhat của phương trình này nên x0 ≥ a = x2 + dy2 > x0. Đieu này là đieu mâu thuan. Neu x2 − dy2 = −1. Khi đó (x0, y0) là nghi m nguyên dương của phương trình 0 0 (1.19). Ngược lại, giả sả phương trình (1.19) có nghi m nguyên dương. Khi đó goi (x0, y0) là nghi m nguyên dương nhỏ nhat của nó. Ta sě cháng minh rang (x0, y0) chính là nghi m của h (1.22). Th t v y, đ t u = x2 + dy2 ; v = 2x0y0. 0 0 Vì x2 − dy2 = −1 ta có 0 0 u2 − dv2 = x2 + dy2 2 − d(2x y )2 = x2 − dy2 2 = 1. V y (u, v) là m®t nghi m của phương trình Pell liên ket của phương trình (1.19). Tà tính nhỏ nhat của (a, b) suy ra u ≥ a ; v ≥ b. Ta sě cháng minh u = a; v = b khi đó (x0, y0) là nghi m của h (1.22). Giả sả ngược lại, tác là u > a, v > b. Ta có: 0 < a − b √ d < a − b √ d a + b √ d = a2 − db2 = 1 nên 0 < a − b √ d < 1. Suy ra V y a − b √ d x0 + y0 √ d < x0 + y0 √ d. (ax0 − bdy0) + (ay0 − bx0) √ d < x0 + y0 √ d. (1.23) Tà a < u, b < v ta có a + b √ d < u + v √ d = x0 + y0 √ d . 2 Suy ra — (ax0 − bdy0) + (ay0 − bx0) √ d = a + b √ d −x0 + y0 √ d < x + y √ d 2 −x + y √ d = x + y √ d dy2 − x2 . Vì x2 − dy2 = −1 nên 0 0 — (ax0 − bdy0) + (ay0 − bx0) √ d < x0 + y0 √ d dy2 − x2 0 0 = x0 + y0 √ d. (1.24)
  • 35. 31 Viết đề tài giá sinh viên – ZALO:0973.287.149-TEAMLUANVAN.COM 0 √ √ 0 0 0 0 0 0 0 0 0 ⇔ y2 > −b2 . 0 0 0 0 Đ t s = ax0 − bdy0; t = ay0 − bx0. Khi đó (1.23), (1.24) có dạng như sau s + t √ d < x + y √ d, 0 Ta có −s + t √ d < x0 + y0 √ d. s2 − dt2 = (ax0 − bdy0)2 − d(ay0 − bx0)2 = a2 x2 + b2 d2 y2 − da2 y2 − db2 x2 (1.25) = a2 x2 − dy2 + b2 d dy2 − x2 . Do (x0, y0) là nghi m của phương trình (1.19) nên x2 − dy2 = −1. V y tà (1.25) 0 0 ta có các đȁng thác s2 − dt2 = −a2 + db2 = − a2 − db2 . (1.26) Lại do (a, b) là nghi m của phương trình Pell liên ket, nên tà (1.26) ta có s2 − dt2 = −1. Giả sả s = 0 ta suy ra dt2 = 1. Do tính nguyên dương của d và t nên d = t = 1. Đieu đó vô lý vì d không là so chính phương. Do đó s khác 0. Tiep theo ta can cháng minh t > 0. Th t v y: t > 0 ⇔ ay0 − bx0 > 0 ⇔ ay0 > bx0 ⇔ a2 y2 > b2 x2 ⇔ 1 + db2 y2 > b2 dy2 − 1 V y t > 0. Do s khác 0 nên chỉ có hai trường hợp xảy ra: Neu s > 0. Khi đó (s, t) là nghi m nguyên dương của phương trình (1.19), mà (x0, y0) là nghi m dương bé nhat của phương trình này nên: s ≥ x0 và t ≥ y0 nên s + t √ d ≥ x0 + y0 √ d. Tà đó ta suy ra mâu thuan nên không xảy ra trường hợp s > 0. Neu s < 0 thì (−s, t) là nghi m nguyên dương của phương trình x2 − dy2 = −1. Bang cách l p lu n tương tự trường hợp trên, ta dan đen −s+t d ≥ x0 +y0 d. Tà đó ta cũng dan đen mâu thuan. V y giả thuyet cháng minh phản cháng (u, v) /= (a, b) là sai. Đieu đó có nghĩa là (u, v) = (a, b).
  • 36. 32 Viết đề tài giá sinh viên – ZALO:0973.287.149-TEAMLUANVAN.COM 1 2 √ √ 3 2 ⇒ −2b dc = u + 3duv − u a + b d 1 − − 1 1 ( x0 = u; x1 = u3 + 3duv2 ; xn+2 = 2axn+1 − xn, 1 2 b = 2xy Định lj 1.2.20. Cho (a, b) là nghi m cơ bản của phương trình Pell liên ket (1.3) của phương trình (1.19) và h ( a = x2 + dy2 có nghi m (u, v) duy nhat. Xét hai dãy so nguyên dương {xn} và {yn} sau đây: y0 = v; y1 = dv3 + 3u2 v; yn+2 = 2ayn+1 − yn. Khi đó (xn, yn) là nghi m của phương trình (1.19). n = 0, 1, 2, 3... Chúng minh. Theo lý thuyet ve dãy so, thì phương trình đ c trưng của dãy là λ2 − 2aλ + 1 = 0. (1.28) Phương trình (1.28) có hai nghi m là λ1 = a − √ ∆ ′ và λ2 = a + √ ∆ ′ , ∆ ′ = a2 − 1. Do (a, b) là nghi m của phương trình (1.3) nên a2 − db2 = 1, tà đó ta có: ∆ ′ = db2 . V y hai nghi m của phương trình đ c trưng (1.28) là λ1 = a − b √ d và λ2 = a + b √ d. Theo lý thuyet dãy so thì xn = c1λn + c2λn . Bây giờ ta xác định c1; c2 tà đieu 1 2 ki n x0 = u; x1 = u3 + 3duv2 . Xét h phương trình sau: c1 a − b √ c + c = u, d + c2 a + b √ d = u3 + 3duv2 . (1.29) Giải h (1.29) ta có c2 = u − c1 và c a b √ d + (u c ) a + b √ d = u3 + 3duv2 ⇒ c1 a − b √ d − a − b √ d = u3 + 3duv2 − u a + b √ d . ⇒ −2b √ dc1 = u3 + 3duv2 − u u2 + dv2 + 2uv √ d ⇒ −2b √ dc1 = 2duv2 − 2u2 v √ d ⇒ −2uv √ dc1 = uv √ d v √ d − u ⇒ c = u − v √ d ⇒ c = u − u − v √ d = u + v √ d . (1.27) 2 2 2
  • 37. 33 Viết đề tài giá sinh viên – ZALO:0973.287.149-TEAMLUANVAN.COM √ √ 2 √ 2 √ n n − b = 2xy V y x = u − v √ d a − b √ d n + u + v √ d a + b √ d n = u − v d u2 + dv2 − 2uv √ d n + u + v d u2 + dv2 + 2uv √ d n 2 2 u + v d 2n+1 + u v d 2n+1 = ). 2 (1.30) Tương tự ta có yn = u + v √ d 2n+1 − u − v √ d 2n+1 2 √ d . (1.31) Tà (1.30) và (1.31), ta có: xn + yn √ d = u + v √ d 2n+1 , xn — yn √ d = u − v √ d 2n+1 . Nhân đȁng thác trên ta có x2 − dy2 = u2 − dv2 2n+1 . Theo cháng minh của Định lý 1.2.4, thì do (u, v) là nghi m của h (1.27) nên u2 − dv2 = −1. Do đó x2 − dy2 = −1. V y (xn, yn) là nghi m nguyên dương của n n phương trình (1.19). Định lj 1.2.21. Cho (a, b) là nghi m của phương trình Pell liên ket (1.19). H ( a = x2 + dy2 có nghi m duy nhat (u, v). Goi (x, y) là nghi m của phương trình Pell (1.19) Khi đó, ton tại so tự nhiên n sao cho x + y √ d = u + v √ d 2n+1 = x + yn √ d. Chúng minh. Xét so sau đây x + y √ d u + v √ d . n (1.32) n
  • 38. 34 Viết đề tài giá sinh viên – ZALO:0973.287.149-TEAMLUANVAN.COM ( Ta có x + y √ d u + v √ d = (xu + dyv) + (yu + xv) √ d = s + t √ d, trong đó s = xu + dyv, t = yu + xv. M t khác s2 − dt2 = (xu + dyv)2 − d(yu + xv)2 = x2 u2 + d2 y2 v2 − dy2 u2 − dx2 v2 = x2 − dy2 u2 − dv2 = (−1) (−1) = 1. V y (s, t) là nghi m của phương trình (1.3). Do (a, b) là nghi m bé nhat của nó, nên theo Định lý 1.2.8 thì ton tại n ≥ 1 sao cho s + t √ d = a + b √ d n+1 và x + y √ d u + v √ d = a + b √ d n+1 . (1.33) M t khác a + b √ d = u2 + dv2 + 2uv √ d = u + v √ d 2 , thay vào (1.33) ta được x + y √ d u + v √ d = u + v √ d 2n+2 , hay x + y √ d = u + v √ d 2n+1 = x + yn √ d. (1.34) Đieu đó có nghĩa là moi nghi m (x, y) là nghi m của phương trình (1.19) và được bieu dien ở dạng (1.34). Ví dn 1.2.22. Tìm 3 nghi m đau tiên của phương trình x2 − 5y2 = −1. Th t v y, trước tiên ta xác định nghi m cơ bản của phương trình Pell liên ket x2 − 5y2 = 1 là (9, 4). Giải h phương trình sau x2 + 5y2 = 9 2xy = 4 ta được nghi m nguyên duy nhat là (2, 1). V y ta có x0 = 2 ; y0 = 1 đây là nghi m bé nhat của phương trình đã cho. n
  • 39. 35 Viết đề tài giá sinh viên – ZALO:0973.287.149-TEAMLUANVAN.COM ( ( j j j j √ Khi đó x1 = 23 + 3.5.2.12 = 38 y1 = 5.13 + 3.22 .1 = 17 là nghi m thá hai của phương trình. Ta có x2 = 2.9.38 − 2 = 682 y2 = 2.9.17 − 1 = 305 là nghi m thá ba của phương trình. V y (2, 1); (38, 17) và (682, 305) là 3 nghi m đau tiên của phương trình. Định lj 1.2.23. Giả sủ d là so nguyên dương không chính phương, pk là giản qk phân của liên phân so của d với k = 0, 1, 2, 3, ... và n là chieu dài chu kì của liên phân so của √ d. Khi đó các phát bieu sau là đúng: (i) Neu n chȁn thì phương trình Pell (1.19) vô nghi m. (ii) Neu n lé thì tat cả các nghi m của phương trình (1.19) được cho bới công thúc: x = p(2k−1)n−1 y = q(2k−1)n−1 k ≥ 1. Chúng minh. Theo Bő đe 1.1.16 ta có (x0, y0) là nghi m của phương trình (1.19). Trong đó x0 = pj với pj qj là giản phân của liên phân so của √ d. Áp dụng Bő đe 1.1.17 ta có p2 − dq2 = (−1)j+1 tj+1 (j = 0, 1, 2, ...; tj+1 > 0) Do p2 − dq2 = −1 nên j + 1 là so lẻ và tj+1 = 1. Vì n là chieu dài của liên phân so của √ d nên theo Bő đe 1.1.18 ta có n| (j + 1) nên j + 1 = n.j ′ j ′ ∈ Z , suy ra j = n.j ′ − 1 (j ≥ 0) Với n là so lẻ. Do j + 1 so lẻ nên njJ lẻ và vì n lẻ nên jJ lẻ. V y jJ = 2k − 1, k ≥ 1. Tà đó ta có j = n(2k − 1) − 1. V y nghi m của phương trình (1.19) có dạng x = p(2k−1)n−1 y = q(2k−1)n−1 k ≥ 1, ( (
  • 40. 36 Viết đề tài giá sinh viên – ZALO:0973.287.149-TEAMLUANVAN.COM với n là so chȁn. Do j + 1 là so lẻ nên njJ là so lẻ mà n chȁn nên vô lý. Do đó không ton tại j’ thỏa mãn. V y phương trình (1.19) vô nghi m. Ví dn 1.2.24. Tìm hai nghi m đau tiên của phương trình x2 − 13y2 = −1. Với k = 1 thì x0 = p4 = 18 ; y0 = q4 = 5 là nghi m bé nhat của phương trình. Với k = 2 thì x1 = p14 = 23382 ; y1 = q14 = 6485 là nghi m thá hai của phương trình. V y (18, 5) và (23382, 6485) là hai nghi m của phương trình. Định lý tiep theo cho ta m®t moi liên h cau trúc nghi m của hai phương trình Pell liên ket. Định lj 1.2.25. Neu (a, b) là nghi m của phương trình (1.3) và (u, v) là nghi m của phương trình (1.19) thì (au+dbv, av+bu) là nghi m của phương trình (1.19). Ví dn 1.2.26. Ta có (99, 70) là nghi m của phương trình x2 − 2y2 = 1 và (7, 5) là nghi m của phương trình x2 − 2y2 = −1. Khi đó (99.7 + 2.70.5, 99.5 + 70.7) = (1393, 985) là nghi m của phương trình x2 − 2y2 = −1.
  • 41. 37 Viết đề tài giá sinh viên – ZALO:0973.287.149-TEAMLUANVAN.COM Chương 2 Phương trình Diophantine dạng x2 − Dy2 = ±4 Chương này chúng tôi t p trung trình bày lại các ket quả ve cau trúc nghi m nguyên của phương trình x2 − Dy2 = ±4 và áng dụng của phương trình x2 − Dy2 = ±1 và x2 − Dy2 = ±4 vào m®t so bài toán phő thông. Các ket quả của chương này được viet theo tài li u [3]. 2.1 Cau trúc nghi m của ho phương trình x2 − Dy2 = ±4 Xét phương trình Diophantine u2 − Dv2 = ±4, (2.1) trong đó D là so nguyên, không là so chính phương. Giả sả phương trình√ (2.1) có nghi m và cho u, v là hai so nguyên thỏa mãn (2.1). Khi đó √ u + v 2 D được goi là nghi m của phương trình (2.1). Neu x + y 2 D là nghi m của phương trình Diophantine u2 − Dv2 = 4 (2.2) thì so u + v √ D x + y √ D u + v √ D = 1 1 2 2 2 cũng là nghi √ m của phương trình (2.1). Nghi m này được goi là liên ket với nghi m u + v 2 D . T p tat cả các nghi m liên ket với nhau tạo thành m®t lớp nghi m của phương trình (2.1).
  • 42. 38 Viết đề tài giá sinh viên – ZALO:0973.287.149-TEAMLUANVAN.COM ( ( 0 2 Đieu ki n can và đủ đe hai nghi m u + v √ D 2 uJ + vJ √ D , 2 thu®c cùng m®t lớp nghi m là so vuJ − uJ v 2 là so nguyên. Định lj 2.1.1. Phương trình (2.1) ho¾c vô nghi m ho¾c có vô so nghi m. Chúng minh. Ta đi cháng minh rang neu phương trình (2.1) có nghi m thì sě có vô so nghi m. Giả sả (x, y) là m®t nghi m của phương trình (2.1). Goi (a, b) là m®t nghi m của phương trình (1.3), tác là a2 − db2 = 1. Khi đó ta có x2 − dy2 = ±4, a2 − db2 = 1. Nhân tàng ve hai đȁng thác trên, ta nh n được x2 − dy2 a2 − db2 = ±4 ⇔ (xa + dyb)2 − d(xb + ya)2 = ±4. (2.3) Đ t xJ = xa + dyb, yJ = xb + ya. Khi đó thay vào (2.3), ta có xJ2 − dyJ2 = ±4. Tà đó ta có (xJ , yJ ) là nghi m của phương trình (2.1). Rõ ràng x < xJ , y < yJ . Do đó giả sả có nghi m ban đau (x1, y1). Xét h thác xn+1 = xna + dynb, yn+1 = xnb + yna. H thác này theo cháng minh trên cho ta lớp nghi m của phương trình (2.2). Định lj 2.1.2. Giả sủ phương trình Pell (2.1) có nghi m và goi (x0, y0) là nghi m nguyên dương nhó nhat của nó. Khi đó ta có y2 ≤ max ±4b ; 4a2 , d ớ đây (a, b) là nghi m dương nhó nhat của phương trình Pell (1.3). Chúng minh. Đ t u = x0a − dy0b, v = y0a − x0b. ( ( ∓
  • 43. 39 Viết đề tài giá sinh viên – ZALO:0973.287.149-TEAMLUANVAN.COM 0 0 ⇔ − ⇔ ( 0 0 0 0 0 = x2 a2 − db2 − dy2 a2 − db2 0 0 0 0 0 0 0 0 ⇔ x2 1 + db2 > db2 x2 ∓ 4 ⇔ x2 > ∓4db2 ⇔ ±4 + dy0 > −4 a — 1 ⇔ dy0 > ∓4a ⇔ y0 > . d 2 0 0 0 khi đó: u2 − dv2 = (x0a − dy0b)2 − d(y0a − x0b)2 = a2 x2 + d2 b2 y2 − 2abdx0y0 − da2 y2 − db2 x2 + 2abdx0y0 = a2 − db2 x2 − dy2 = ±4. Giả sả ket lu n của bài toán không đúng, tác là y2 > ±4b2 , y2 > max ±4b2 ; 4a2 . d Tà đó, y2 > 4a2 . d Ta sě cháng minh rang u > 0, v > 0. Th t v y, v > 0 y0a x0b > 0 y0a > x0b ⇔ a2 y2 > b2 x2 ⇔ 1 + db2 y2 > b2 ±4 + dy2 ⇔ y2 > ±4b2 . V y v > 0. Tương tự ta có u > 0 ⇔ x0a − dy0b > 0 ⇔ x0a > dy0b ⇔ x2 a2 > d2 b2 y2 0 0 0 2 2 2 2 2 ∓4a2 Như v y, ta có u > 0 và v > 0 mà u2 − dv2 = ±4 nên (u, v) là nghi m của (2.1). Tà h với ȁn là (x0, y0), suy ra u = x0a − dy0b v = −x0b + y0a x0 = au + dbv, y0 = bu + av. Tà đó ta có x0 > u, y0 > v. Như the ta đã xây dựng được nghi m (u, v) của phương trình Pell (2.2), mà nghi m này còn nhỏ hơn (x0, y0). Đieu này là mâu thuan vì (x0, y0) là nghi m bé nhat của (2.1). V y giả thuyet phản cháng sai, nên y2 ≤ max ±4b ; 4a2 . d ∓ 0 ( ∓ ∓
  • 44. 40 Viết đề tài giá sinh viên – ZALO:0973.287.149-TEAMLUANVAN.COM i 0 0 0 1. Neu v0 ≤ max ±4b ; 1 1 Định lj 2.1.3. Giả sủ (2.1) có nghi m và (α1, β1) ; (α2, β2) ; ... ; (αm, βm) là tat cả các nghi m của (2.1) thóa mãn bat đȁng thúc: β2 ≤ max ±4b2 ; ∓4a2 d Xét m dãy {xn,i, yn,i} i = 1, m xác đ nh như sau: x0,i = αi; y0,i = βi xn+1,i = xn,ia + dyn,ib yn+1,i = xn,ib + yn,ia, ớ đây (a, b) là nghi m dương bé nhat của phương trình Pell (1.3) tương úng với (2.1). Khi đó các dãy {xn,i, yn,i} i = 1, m sẽ vét cạn het nghi m của phương trình (2.1). Chúng minh. Theo cháng minh trong Định lý 2.1.1, các so hạng của các dãy trên đeu là nghi m của phương trình Pell (2.1). Ta đi cháng minh đieu ngược lại. Giả sả (u0, v0) là m®t nghi m dương bat kì của (2.1). Ta phải cháng minh sự ton tại i, k sao cho u0 = xk,i ; v0 = yk,i. Chỉ có hai khả năng sau xảy ra 2 2 ∓4a2 sao cho (u0, v0) = (αi, βi) = (x0,i, y0,i). Ta chỉ chon k = 0. 2. Neu v2 > max ±4b2 ; ∓4a2 d . Đ t: u1 = au0 − dbv0 ; v1 = av0 − bu0. Ta thay rang u2 − dv2 = ±4. Ngoài ra, do v2 > ±4b2 và v2 > 4a2 , d nên l p lu n tương tự như cách cháng minh trong định lý 2.1.2 ta có u1 > 0; v1 > 0. Tà đó suy ra (u1, v1) là nghi m của phương trình (2.1). Tà h u1 = au0 − dbv0, v1 = av0 − bu0. Suy ra u0 = au1 + dbv1, v0 = bu1 + av1. d ∓ ( ( . Theo giả thuyet ton tại i ∈ {1, 2, ..., m}
  • 45. 41 Viết đề tài giá sinh viên – ZALO:0973.287.149-TEAMLUANVAN.COM 1 k 2 2 Vì v y suy ra v1 < v0 (u0, v0, u1, v1, a, b, d > 0) . Neu v2 > max ±4b2 ; ∓4a2 d thì bang l p lu n trên ta lại xây dựng được nghi m (u2, v2) của (2.2) với u2 < u1 ; v2 < v1. Quá trình ay cá tiep tục và phải ket thúc ở bước thá k mà sau khi có nghi m (uk, vk) của (2.2), thì v2 ≤ max 4b2 ; 4a2 . d Khi đó, ton tại i ∈ {1, 2, ..., m} : (uk, vk) = (αi, βi) = (x0,i, y0,i). Do uk−1 = auk + dbvk, vk−1 = buk + avk. Nên ta có (u ,v ) = (x , y ) vì ( uk−1 = auk + dbvk = ax0,i + dby0,i = x1,i k−1 k−1 1,i 1,i vk−1 = buk + avk = bx0,i + ay0,i = y1,i (u ,v ) = (x , y ) vì ( uk−2 = auk−1 + dbvk−1 = ax1,i + dby1,i = x2,i k−2 k−2 ... 2,i 2,i vk−2 = buk−1 + avk−1 = bx1,i + ay1,i = y2,i (u0, v0) = (xk,i, yk,i) . V y Định lý đã được cháng minh. Ví dn 2.1.4. Giải phương trình: x2 − 5y2 = −4. Xét phương trình Pell liên ket với nó có dạng: x2 − 5y2 = 1. Phương trình liên ket có nghi m dương nhỏ nhat là (a, b) = (9, 4). Khi đó: max −4.4 ; 4.92 5 4.81 = = 64, 8. 5 So nguyên dương β lớn nhat thỏa mãn β ≤ max 4b2 ; −4a2 d = 64, 8 là β = 8. Xét phương trình x2 − 5y2 = −4, ta có: ( −
  • 46. 42 Viết đề tài giá sinh viên – ZALO:0973.287.149-TEAMLUANVAN.COM i 1. Neu y = 1 thì x = 1, 2. Neu y = 2 thì x = 4, 3. Neu y = 3, 4, 7, 8 thì x không là so nguyên. 4. Neu y = 5 thì x = 11. Như v y bang cách thả trực tiep như the, ta thay phương trình đã cho có ba nghi m (1, 1); (4, 2) và (11, 5) mà thỏa đieu ki n: β2 ≤ max 4b2 ; 4a2 . d Theo định lý, phương trình đã cho có 3 dãy nghi m sau x0,1 = 1; y0,1 = 1; xn+1,1 = 9xn,1 + 20yn,1; yn+1,1 = 4xn,1 + 9yn,1, x0,2 = 4; y0,2 = 2; xn+1,2 = 9xn,2 + 20yn,2; yn+1,2 = 4xn,2 + 9yn,2, x0,3 = 11; y0,3 = 5; xn+1,3 = 9xn,3 + 20yn,3; yn+1,3 = 4xn,3 + 9yn,3. Ba dãy này đã vét het nghi m của phương trình. Định lý tiep theo của mục này cho ta m®t cách hǎu hi u đe tìm nghi m của phương trình (2.1) thông qua m®t nghi m biet trước và nghi m của phương trình Pell (1.3). Định lj 2.1.5. Cho (u, v) là nghi m nguyên dương của phương trình (2.1) và (a, b) là nghi m của phương trình Pell (1.3) liên ket với phương trình (2.1). Khi đó (ua+dbv, ub+va) là nghi m của phương trình (2.1). Các nghi m của phương trình (2.1) khác bi t nhau với sự khác bi t của (u, v) và (a, b). 2.2 Phương trình Diophantine dạng x2 − Dy2 = 4 Xét phương trình Diophantine (2.2) u2 − Dv2 = 4, trong đó D là các so nguyên và D không là so chính phương. −
  • 47. 43 Viết đề tài giá sinh viên – ZALO:0973.287.149-TEAMLUANVAN.COM ! — Dv n n 2n−1 2n−1 ! ! Định lj 2.2.1. Cho (x1, y1) là nghi m cơ bản của phương trình (2.2) và cho un ! = x1 Dy1 !n 1 ! (2.4) vn y1 x1 0 với n ≥ 1. Khi đó nghi m nguyên của phương trình (2.2) là (xn, yn) được cho bới công thúc (x , y ) = un , vn . (2.5) Chúng minh. Ta đi cháng minh định lý bang phương pháp quy nạp theo n. Với n = 1, theo (2.4) ta có (u1, v1) = (x1, y1). Và vì (x1, y1) là nghi m cơ bản của phương trình (2.2). Giả sả xn−1, yn−1 là nghi m của phương trình (2.2), tác là u2 − Dv2 x2 n−1 + y2 n−1 = n−1 n−1 22n−4 = 4. Ta sě cháng minh nghi m của phương trình (2.2) là (xn, yn) được cho bởi công thác (2.5). Th t v y, tà phương trình (2.4) ta có un ! = x1 Dy1 !n 1 ! vn y1 x1 0 = x1 Dy1 ! x1 Dy1 !n−1 1 ! y1 x1 y1 x1 0 = x1 Dy1 y1 x1 un−1 vn−1 Hơn nǎa, = x1un−1 + Dy1vn−1 . y1un−1 + x1vn−1 x2 − Dy2 u2 = n−1 2 n−1 n n 22n−2 (x1un−1 + Dy1vn−1)2 − D(y1nn−1 + x1vn−1)2 22n−2 x2u2 + 2x1un−1Dy1vn−1 + D2 y2 v2 = 1 n−1 22n−2 1 n−1 D(y2 u2 + 2y1un−1x1vn−1 + x2 v2 ) — n−1 22n−2 1 n−1 x2 (u2 − Dv2 ) − Dy2 (u2 − Dv2 ) = 1 n−1 n−1 1 n−1 n−1 22n−2 (x2 − Dy2 )(u2 − Dv2 ) = 1 1 n−1 n−1 . 22n−2 1 =
  • 48. 44 Viết đề tài giá sinh viên – ZALO:0973.287.149-TEAMLUANVAN.COM − − Vì (x1, y1) là nghi m cơ bản của phương trình (2.2) và theo giả thiet quy nạp ta có 2 2 4.2n−2 xn + Dyn = 2n−2 = 4. Do đó, (xn, yn) là nghi m của phương trình (2.2). Vì n là tùy ý nên tat cả các nghi m nguyên của phương trình (2.2) là (xn, yn). H quả 2.2.2. Cho (x1, y1) là nghi m cơ bản của phương trình (2.2) thì x = x1xn−1 + Dy1yn−1 , y n 2 n và = y1xn−1 + x1yn−1 2 xn xn 1 . . = −2y1 với moi n ≥ 2. . yn yn−1 . Chúng minh. Ta có un = x1un−1 + Dy1vn−1, vn = y1un−1 + x1vn−1 và un−1 = 2n−2 xn−1,vn−1 = 2n−2 yn−1. V y ta có bien đői tương đương sau un = x1un−1 + Dy1vn−1, 2n−1 xn = x12n−2 xn−1 + Dy12n−2 yn−1, 2n−1 xn = 2n−2 (x1xn−1 + Dy1yn−1), x = x1xn−1 + Dy1yn−1 , n 2 và vn = y1un−1 + x1vn−1, 2n−1 yn = y12n−2 xn−1 + x12n−2 yn−1, 2n−1 yn = 2n−2 (y1xn−1 + x1yn−1), Hơn nǎa y = y1xn−1 + x1yn−1 . n 2 xn xn 1 . . = xnyn−1 — xn−1yn . yn yn−1 = x1xn−1 + Dy1yn−1 y 2 n−1 — xn−1 y1xn−1 + x1yn−1 2 −y1(x2 − Dy2 ) = n−1 n−1 2 = −4y1 2 = −2y1. .
  • 49. 45 Viết đề tài giá sinh viên – ZALO:0973.287.149-TEAMLUANVAN.COM — Dv 2n+1 2n+1 22n 22n H quả 2.2.3. Cho (x1, y1) là nghi m cơ bản của phương trình (2.2). Khi đó, (xn, yn) thóa mãn các bieu thúc sau xn = (x1 − 1)(xn−1 + xn−2) − xn−3, yn = (x1 − 1)(yn−1 + yn−2) − yn−3. 2.3 Phương trình Diophantine dạng x2 − Dy2 = −4 Xét phương trình Diophantine u2 − Dv2 = −4, (2.6) trong đó D là các so nguyên và D không là so chính phương. Định lj 2.3.1. Cho (x1, y1) là nghi m cơ bản của phương trình (2.6) và cho u2n+1 ! = x1 Dy1 !2n+1 1 ! (2.7) v2n+1 y1 x1 0 với n ≥ 0. Khi đó nghi m nguyên của phương trình (2.6) là (x2n+1, y2n+1) được cho bới công thúc (x , y ) = u2n+1 , v2n+1 . (2.8) Chúng minh. Sả dụng phương pháp quy nạp theo n ta có. Với n = 0, theo (2.7) ta có (u1, v1) = (x1, y1). Và vì (x1, y1) là nghi m cơ bản của phương trình (2.6). Giả sả x2n−1, y2n−1 là nghi m của phương trình (2.6), tác là x2 + y2 u2 = 2n−1 2 2n−1 = −4. 2n−1 2n−1 24(n−1) Chúng ta phải đi cháng minh nghi m của phương trình (2.6) là (x2n+1, y2n+1)
  • 50. 46 Viết đề tài giá sinh viên – ZALO:0973.287.149-TEAMLUANVAN.COM 1 1 1 1 1 1 được cho bởi công thác (2.8). Th t v y, tà phương trình (2.7) ta có u2n+1 ! = x1 Dy1 !2n+1 1 ! v2n+1 y1 x1 0 = x1 Dy1 !2 x Dy1 !2n−1 1 ! y1 x1 y1 x1 0 = x1 Dy1 !2 u2n−1 ! y1 x1 v2n−1 (x2 + Dy2 )u2n−1 + Dx1y1v2n−1 ! 2x1y1u2n−1 + (x2 + Dy2 )v2n−1 Hơn nǎa, u2 − Dv2 2 2n+1 2n+1 = 2n+1 2n+1 22n ((x2 + Dy2 )u2n−1 + Dx1y1v2n−1)2 = 1 1 22n D(2x1y1u2n−1 + (x2 + Dy2 )v2n−1)2 22n (x2 + Dy2 )2 u2 + 4x1u2n−1Dy1v2n−1 + 4D2 x2 y2 v2 = 1 1 2n−1 22n 1 1 2n−1 D(4x2 y2 u2 + 4x1y1(x2 + Dy2 )u2n−1v2n−1 + (x2 + Dy2 )2 v2 ) — 2n−1 1 1 22n 1 1 n−1 (x2 + Dy2 )2 (u2 — Dv2 ) − 4Dx2 y2 (u2 — Dv2 ) = 1 1 2n−1 2n−1 22n 1 1 2n−1 2n−1 (x2 − Dy2 )2 (u2 − Dv2 ) = 1 1 n−1 n−1 . 22n Vì (x1, y1) là nghi m cơ bản của phương trình (2.6) và theo giả thiet quy nạp ta có 2 2 (−4)2 (−4).24n−4 xn−1 + Dyn−1 = 24n = −4. Do đó, (x2n+1, y2n+1) là nghi m của phương trình (2.6). Vì n là tùy ý nên tat cả các nghi m nguyên của phương trình (2.6) là (x2n+1, y2n+1). H quả 2.3.2. Cho (x1, y1) là nghi m cơ bản của phương trình (2.6). Khi đó (x2 + Dy2 )x2n−1 + Dx1y1y2n−1 x2n+1 = 1 1 , 4 2x1y1x2n−1 + (x2 + Dy2 )y2n−1 y2n+1 = 1 1 4 1 1 2 x — Dy = . 1 −
  • 51. 47 Viết đề tài giá sinh viên – ZALO:0973.287.149-TEAMLUANVAN.COM − . − 1 1 1 1 1 1 1 1 1 1 1 1 1 1 1 1 1 1 và x2n+1 x2n 1 . = 2x1y1 với moi n ≥ 1. Chúng minh. Ta có . y2n+1 y2n−1 . u2n+1 = (x2 + Dy2 )u2n−1 + Dx1y1v2n−1, v2n+1 = 2x1y1u2n−1 + (x2 + Dy2 )v2n−1 và u2n−1 = 22n−2 x2n−1,v2n−1 = 22n−2 y2n−1. V y chúng ta có bien đői tương đương sau u2n+1 = (x2 + Dy2 )u2n−1 + Dx1y1v2n−1 22n x2n+1 = (x2 + Dy2 )22n−2 x2n−1 + Dx1y122n−2 y2n−1 22n x2n+1 = 22n−2 ((x2 + Dy2 )x2n−1 + 2Dx1y1y2n−1) (x2 + Dy2 )x2n−1 + Dx1y1y2n−1 x2n+1 = 1 1 , 4 và v2n+1 = 2x1y1u2n−1 + (x2 + Dy2 )v2n−1 22n y2n+1 = 2x1y122n−2 x2n−1 + (x2 + Dy2 )22n−2 x2n−1 22n y2n+1 = 22n−2 (2x1y1x2n−1 + (x2 + Dy2 )y2n−1) 2x1y1x2n−1 + (x2 + Dy2 )y2n−1 Và hơn nǎa y2n+1 = 1 1 . 4 x2n+1 x2n 1 . . = x2n+1 y2n−1 — x2n−1 y2n+1 . y2n+1 y2n−1 . (x2 + Dy2 )x2n−1 + Dx1y1y2n−1 = 1 1 4 y2n−1 −x2n−1 2 x1y1x2n−1 + (x2 + Dy2 )y2n−1 4 −2x1y1(x2 − Dy2 ) = 2n−1 2n−1 4 = −2x1y1(−4) = 2x 4 y1. 1
  • 52. 48 Viết đề tài giá sinh viên – ZALO:0973.287.149-TEAMLUANVAN.COM 1 1 − ≈ − ≈ ≈ 408 408 − H quả 2.3.3. Cho (x1, y1) là nghi m cơ bản của phương trình (2.6). Khi đó, (x2n+1, y2n+1) thóa mãn các bieu thúc sau x2n+1 = (x2 + 1)(x2n−1 + x2n−3) − x2n−5 y2n+1 = (x2 + 1)(y2n−1 + y2n−3) − y2n−5. 2.4 M t so fíng dnng trong toán pho thông 2.4.1 Tìm so nguyên tfi h thfíc ràng bu c Ví dn 2.4.1. Xác định t ∈ Z thỏa mãn 10t + 12 = x2 và 5t + 4 = y2 Th t v y, ta có: x2 − 12 t = 10 y2 4 t = 5 x2 12 = 10 y2 4 2 5 ⇒ x — 12 = 2y − 8 Suy ra x2 − 2y2 = 4. Giải phương trình Pell ta xác định được t. 2.4.2 Xap xỉ hfiu t của căn b c 2 Ta không the viet √ d x y với x, y ∈ Z vì √ d là so vô tỉ. Tuy nhiên ta có x2 − dy2 = ±N ⇒ x 2 N = d + y2 ≈ d. Vì the neu nghi m y của phương trình Pell sao cho y2 càng lớn thì xap xỉ hǎu tỉ của √ d càng tot. Ta xét m®t so ví dụ minh hoa. Ví dn 2.4.2. Nghi m thá tư của phương trình x2 2y2 = 1 là (x, y) = (577, 408) và 577 ≈ 1.4142156. Trong khi đó, √ 2 = 1.4142135. V y 577 ≈ √ d. Ví dn 2.4.3. Cho phương trình Pell x2 − 3y2 = 1. Ta có (1351, 780) là nghi m của phương trình trên. Do đó √ 3 1351 1.7320512. 780 y 2 − ⇒
  • 53. 49 Viết đề tài giá sinh viên – ZALO:0973.287.149-TEAMLUANVAN.COM 4 # ⇔ 2 m2 + m + 1 = n2 2 2 2 2 4 2 2 2.4.3 Tong của nhfing so nguyên liên tiep nhau Ví dn 2.4.4. Cho 1 + 2 = 3; 1 + 2 + ... + 14 = 15 + ... + 20. Tìm k, l thỏa mãn 1 + 2 + . . . + k = (k + 1) + . . . + l. Ta có (k + 1) k 1 + 2 + . . . + k = (k + 1) + . . . + l ⇔ 2 = (k + 1 + l) (l − k) 2 ⇔ 2k2 + 2k = l2 + l ⇔ 2 " k + 1 − 1 # = 1 1 2 l + − 2 4 ⇔ 2 h (2k + 1)2 − 1 i = (2l + 1)2 − 1 ⇔ (2l + 1) − 2(2k + 1) = −1. Đ t x = 2l + 1 và y = 2k + 1 ta được phương trình Pell x2 − 2y2 = −1 với x, y > 0 và x, y là so lẻ. Ta can giải phương trình trên và tà đó tìm ra k và l. Khi đó ta có: 1 + 2 = 3 1 + 2 + . . . + 14 = 15 + 16 + ... + 20 1 + 2 + ... + 84 = 85 + 86 + ... + 119. 2.4.4 Tam giác Pythagoras Ví dn 2.4.5. Cho 32 + 42 = 52 . Ta can tìm m và n sao cho m2 +(m + 1)2 = n2 . Th t v y: m2 + (m + 1)2 = n2 ⇔ 2m2 + 2m + 1 = n2 ⇔ 2 " m + 1 − 1 + 1 = n2 ⇔ (2m + 1)2 − 2n2 = −1. Đ t x = 2m + 1 và y = n ta được phương trình Pell x2 − 2y2 = −1 với x, y > 0 và x lẻ. Ta can giải phương trình trên và tà đó tìm ra m và n. Khi đó ta có:
  • 54. 50 Viết đề tài giá sinh viên – ZALO:0973.287.149-TEAMLUANVAN.COM 2 − x 1 7 41 239 1393 8119 y 1 5 29 169 985 5741 m 0 3 20 119 696 4059 n 0 5 29 169 985 5741 202 + 212 = 292 , 1192 + 1202 = 1692 . Ví dn 2.4.6. Ta can tìm m và n sao cho m2 + n2 = (n + 1)2 . Th t v y, ta có phương trinh tương đương với m2 = 2n + 1. Cho m = 2k + 1 vì m lẻ nên m2 1 n = 2 = 2k2 + 2k. Không can dùng tới phương trình Pell mà ta van có the giải ra dựa vào bảng sau: k 1 2 3 4 m 3 5 7 9 n 4 12 24 40 n + 1 5 13 25 41 Tà bảng trên cho ta thay 32 + 42 = 52 ; 52 + 122 = 132 ; 72 + 242 = 252 ; 92 + 402 = 412 . 2.4.5 Tam giác Heron Tam giác Heron là tam giác có cạnh và di n tích nguyên. Ví dn 2.4.7. Tam giác vuông có cạnh 3, 4, 5 và có di n tích bang 6. Công thác tính di n tích của Hêron với tam giác bat kì có cạnh là a, b, c là S = √ p (p − a) (p − b) (p − c) với p = a + b + c . 3a Tìm tam giác Hêron với các cạnh liên tiep a − 1; a; a + 1. Khi đó p = 2 và S2 = p (p − a) (p − a + 1) (p − a − 1) 3a a = . .a + 2 . a − 2 . 2 2 2 2
  • 55. 51 Viết đề tài giá sinh viên – ZALO:0973.287.149-TEAMLUANVAN.COM − Suy ra (4S)2 = 3a2 a2 4 . Tà đó ta thay rang a là so chȁn. Đ t a = 2x với x ∈ Z ta có S2 = 3x2 x2 − 1 . Vì 3, x2 = 1 nên x2 − 1 = 3y2 (y ∈ Z). Ta có phương trình Pell x2 − 3y2 = 1 (a = 2x; S = 3xy) . Giải phương trình trên ta sě tìm được a và S. Ta có bảng sau: x 2 7 26 97 362 1351 y 2 7 26 97 362 1351 a 4 14 52 154 724 2702 S 6 84 1170 16926 226974 3161340 Tà bảng ket quả trên ta thay: Tam giác có các cạnh là 13, 14, 15 và có di n tích là 84. Tam giác có các cạnh là 51, 52, 53 và có di n tích là 1170. Tam giác có các cạnh là 153, 154, 155 và có di n tích là 16926.
  • 56. 52 Viết đề tài giá sinh viên – ZALO:0973.287.149-TEAMLUANVAN.COM Ket lu n Lu n văn phương trình Diophantine x2 − Dy2 = ±4 trình bày nhǎng van đe chính sau đây: • Trình bày lại m®t so tính chat của liên phân so và liên phân so mở r®ng tà đó áng dụng nghiên cáu tính chat nghi m của phương trình Pell cő đien; • Trình bày lại các ket quả ve công thác nghi m của phương trình Diophan- tine dạng x2 − Dy2 = ±4 trong m®t so trường hợp giải được của phương trình; • Trình bày m®t so áng dụng của phương trình Diophantine trong toán hoc phő thông.
  • 57. 53 Viết đề tài giá sinh viên – ZALO:0973.287.149-TEAMLUANVAN.COM Tài li u tham khảo Tieng Vi t [1] Nguyen Thị My Hạnh (2017), Giải phương trình Diophante y2 = Ax4 + B, Lu n văn Thạc sĩ Toán hoc, Trường Đại hoc Khoa hoc, Đại hoc Thái Nguyên. [2] Đào Thị Thương Hoài (2010), M®t vài van đe ve phương trình Diophante, Lu n văn Thạc sĩ Toán hoc, Trường Đại hoc Khoa hoc, Đại hoc Thái Nguyên. Tieng Anh [3] Ahmet Tekcan (2007), "The Pell equation X2 − D ∗ Y 2 = ±4", Applied Mathematical Sciences, 1(8), pp. 363–369. [4] Ahmet Tekcan (2011), "Continued Fractions Expansion of √ D and Pell Equation x2 − D ∗ y2 = 1", Mathematica Moravica, 15(2), pp. 19—27. [5] B. Stolt (1952), "On the Diophantine equation U2 −D∗V 2 = ±4N", Arkiv för Matematik, 2(2-3), pp. 251–268. [6] B. Stolt (1955), "On the Diophantine equation U2 − D ∗ V 2 = ±4N", part III, Arkiv för Matematik, 3(2), pp. 117–132.